Вы находитесь на странице: 1из 58

Московский физико-технический институт

Физтех-школа прикладной математики и информатики

М Н О Г О М Е Р Н Ы Й А Н А Л И З , И Н Т Е Г РА Л Ы И РЯ Д Ы
БИЛЕТЫ К ЭКЗАМЕНУ
II СЕМЕСТР
Лектор: Редкозубов Вадим Витальевич

Автор: Головко Денис, Фёдор Стуров


Проект на Github

весна 2023
Математический анализ Назад к содержанию

Содержание
1. Преобразование Абеля. Леммы Абеля для последовательностей и интегра-
лов. Несобственные интегралы Римана и их свойства. Критерий Коши. Аб-
солютная и условная сходимости несобственных интегралов. Интегралы от
неотрицательных функций. Признак сравнения и его следствия. Интегралы
от знакопеременных функций. Признаки Дирихле и Абеля. Несобственные
интегралы с несколькими особенностями. 4

2. Числовые ряды и их свойства. Группировка ряда. Критерий Коши. Абсо-


лютная и условная сходимости рядов. Связь сходимости ряда и интеграла от
ступенчатой функции. Ряды с неотрицательными членами. Признак сравне-
ния, интегральный признак. Признаки Коши, Даламбера, Гаусса (б/д). Зна-
копеременные ряды. Признак Лейбница. Признаки Дирихле (б/д) и Абеля
(б/д). Перестановка членов абсолютно сходящегося ряда. Теорема Римана о
перестановке (б/д). Произведение абсолютно сходящихся рядов. 10

3. Поточечная и равномерная сходимость функциональных последовательно-


стей и рядов, супремум-критерий. Арифметические свойства. Критерий Ко-
ши равномерной сходимости. Непрерывность предельной функции и суммы
ряда. Интегрируемость предельной функции и почленное интегрирование ря-
да. Дифференцируемость предельной функции и почленное дифференциро-
вание ряда. Признаки Вейерштрасса, Дирихле, Абеля равномерной сходимо-
сти рядов. Пример ван-дер-Вардена (б/д). 16

4. Степенные ряды. Теорема Коши-Адамара. Радиус и круг сходимости, равно-


мерная сходимость степенных рядов. Теорема Абеля. Дифференцируемость
суммы степенного ряда. Теорема единственности, ряд Тейлора. Пример бес-
конечно дифференцируемой функции, не разлагающейся в степенной ряд.
Достаточное условие разложимости функции в степенной ряд. Ряды Тейло-
ра 𝑒𝑥 , sin 𝑥, cos 𝑥, (1 + 𝑥)𝛼 , ln(1 + 𝑥). 22

5. Метрические и нормированные пространства, 𝑝-нормы на R𝑛 . Топология мет-


рических пространств: открытые и замкнутые множества, их свойства. Пре-
дельные точки. Критерии замкнутости множества. Замыкание множества.
Подпространства метрического пространства, описание открытых множеств
подпространства. Компакты и их свойства. Теорема о секвенциальной ком-
пактности. Описание компактов в R𝑛 . Теорема Больцано-Вейерштрасса. Пол-
ные метрические пространства. Полнота пространств R𝑛 и 𝐵(𝐸). 26

ФПМИ МФТИ, весна 2023 1


Математический анализ Назад к содержанию

6. Предел функции, отображающей метрическое пространство в метрическое


пространство, его свойства. Предел по подмножествам. Равносильные усло-
вия непрерывности. Непрерывность композиции. Критерий непрерывности
через прообразы. Непрерывные функции на компактах. Теорема Вейерштрас-
са. Эквивалентность норм в конечномерных пространствах (б/д). Теорема
Кантора о равномерной непрерывности. Связные множества в метрических
пространствах. Теорема о промежуточном значении. Линейно связные мно-
жества. Линейные отображения из R𝑛 в R𝑚 , операторная норма. 31

7. Дифференцируемость функции из R𝑛 в R𝑚 . Производная по вектору и ее


связь с дифференциалом. Дифференцируемость композиции. Связь диффе-
ренцируемости функции с дифференцируемостью ее координатных функций.
Частные производные, необходимые условия дифференцируемости. Гради-
ент. Матрица Якоби. Достаточные условия дифференцируемости. Частные
производные высших порядков. Независимость смешанной производной от
порядка дифференцирования. Дифференциалы высших порядков и крат-
ная дифференцируемость. Формула Тейлора с остаточным членом в форме
Лагранжа, в форме Пеано (б/д). 36

8. Брусы в R𝑛 и их объем. Представление открытого множества в виде объ-


единения кубов. Алгебры и 𝜎-алгебры, борелевская 𝜎-алгебра. Внешняя мера
Лебега и ее свойства. Измеримые множества, измеримость множеств внеш-
ней меры нуль и полупространств. Теорема Каратеодори: 𝜎-алгебра измери-
мых множеств, мера Лебега и ее счетная аддитивность. Непрерывность меры
Лебега. Измеримость брусов, борелевских имножеств. Критерии измеримо-
сти множества: приближение борелевскими, приближение брусами. Пример
неизмеримого множества. 42

9. Измеримые функции. Согласованность измеримости функций с арифметиче-


скими операциями. Измеримость точных граней и предела последовательно-
сти измеримых функций. Сходимость почти всюду. Простые функции. Тео-
рема о приближении измеримой функции простыми. 48

ФПМИ МФТИ, весна 2023 2


Математический анализ Назад к содержанию

10. Интеграл от неотрицательной простой функции и его свойства. Интеграл от


неотрицательной измеримой функции. Монотонность интеграла по функци-
ям и по множествам. Теорема Леви о монотонной сходимости. Аддитивность
интеграла по функциям. Счетная аддитивность интеграла по множествам.
Неравенство Чебышева. Интеграл Лебега от произвольной измеримой функ-
ции. Интегрируемые функции. Одновременная интегрируемость функции и
ее модуля. Конечность почти всюду интегрируемой функции. Пренебрежение
при интегрировании множествами меры нуль. Монотонность и линейность
интеграла. Теорема Лебега о мажорированной сходимости. Связь интеграла
Лебега и определенного интеграла Римана. Формула суммирования Эйлера
(б/д). Формула Стирлинга. 51

ФПМИ МФТИ, весна 2023 3


Математический анализ Назад к содержанию

1. Преобразование Абеля. Леммы Абеля для последовательностей


и интегралов. Несобственные интегралы Римана и их свойства.
Критерий Коши. Абсолютная и условная сходимости несобствен-
ных интегралов. Интегралы от неотрицательных функций. При-
знак сравнения и его следствия. Интегралы от знакопеременных
функций. Признаки Дирихле и Абеля. Несобственные интегралы
с несколькими особенностями.
Определение
∑︀𝑛 1. Пусть {𝑎𝑛 }, {𝑏𝑛 } — (комлексные) последовательности, 𝑚 ∈ N, и пусть 𝐴𝑛 =
𝑎
𝑘=1 𝑘 для всех 𝑛 ∈ N. Тогда 𝑎𝑘 = 𝐴𝑘 − 𝐴𝑘−1 (𝐴0 = 0), и, значит,
𝑛
∑︁ 𝑛
∑︁ 𝑛
∑︁ 𝑛−1
∑︁
𝑎𝑘 𝑏𝑘 = (𝐴𝑘 − 𝐴𝑘−1 )𝑏𝑘 = 𝐴𝑘 𝑏𝑘 − 𝐴𝑘 𝑏𝑘+1 .
𝑘=𝑚 𝑘=𝑚 𝑘=𝑚 𝑘=𝑚−1

Справедливо преобразование Абеля:


𝑛
∑︁ 𝑛−1
∑︁
𝑎𝑘 𝑏𝑘 = 𝐴𝑛 𝑏𝑛 − 𝐴𝑚−1 𝑏𝑚 − 𝐴𝑘 (𝑏𝑘+1 − 𝑏𝑘 ).
𝑘=𝑚 𝑘=𝑚

Лемма 1 (Абель). Пусть {𝑎𝑛 } — (комплексная) последовательность, {𝑏𝑛 } — монотонная по-


следовательность, и пусть ∀𝑘 |𝐴𝑘 | ⩽ 𝑀 . Тогда:
⃒ ⃒
⃒ ∑︁𝑛 ⃒
𝑎𝑘 𝑏𝑘 ⃒ ⩽ 2𝑀 (|𝑏𝑚 | + |𝑏𝑛 |).
⃒ ⃒

⃒ ⃒
𝑘=𝑚

Доказательство. По монотонности {𝑏𝑛 } знаки 𝑏𝑘+1 − 𝑏𝑘 сохраняются, поэтому:


⃒ ⃒ ⃒ ⃒)︃
𝑛 ⃒ 𝑛−1
(︃
⃒ ∑︁ ⃒ ∑︁ ⃒
𝑎𝑘 𝑏𝑘 ⃒ ⩽ 𝑀 |𝑏𝑛 | + |𝑏𝑚 | + ⃒ (𝑏𝑘+1 − 𝑏𝑘 )⃒ = 𝑀 (|𝑏𝑛 | + |𝑏𝑚 | + |𝑏𝑛 − 𝑏𝑚 |) .
⃒ ⃒ ⃒ ⃒

⃒ ⃒ ⃒ ⃒
𝑘=𝑚 𝑘=𝑚

⃒∫︀ 𝑥 ⃒
Лемма 2 (Абель). Пусть 𝑓 ∈ ℛ[𝑎, 𝑏], 𝑔 монотонна на [𝑎, 𝑏], и пусть ∀𝑥 ∈ [𝑎, 𝑏] ⃒ 𝑎 𝑓 (𝑡)𝑑𝑡⃒ ⩽ 𝑀 .
Тогда: ⃒∫︁ 𝑏 ⃒
⃒ ⃒

⃒ 𝑓 (𝑥)𝑔(𝑥)𝑑𝑥⃒⃒ ⩽ 2𝑀 (|𝑔(𝑎)| + |𝑔(𝑏)|).
𝑎
∫︀ 𝑏
Доказательство. Зафиксируем 𝜀 > 0. Положим 𝐼 = 𝑎 𝑓 (𝑥)𝑑𝑥. Тогда ∃𝛿 > 0 ∀(𝑇, 𝜉) (|𝑇 | < 𝛿 →
|𝜎𝑇 (𝑓, 𝜉) − 𝐼| < 2𝜀 ).
Выберем одно такое разбиение 𝑇 = {𝑥𝑖 }𝑛𝑖=0 .
Пусть 𝑇𝑘 = {𝑥𝑖∫︀}𝑘𝑖=0 — соответствующее разбиение [𝑥0 , 𝑥𝑘 ], 𝑘 = 1, . . . , 𝑛. По критерию Дарбу
𝑥
числа 𝜎𝑇𝑘 (𝑓, 𝜉𝑘 ) и 𝑥0𝑘 𝑓 (𝑥)𝑑𝑥 лежат на отрезке [𝑠𝑇𝑘 (𝑓 ), 𝑆𝑇𝑘 (𝑓 )], и верно 𝑆𝑇𝑘 (𝑓 ) − 𝑠𝑇𝑘 (𝑓 ) ⩽ 𝑆𝑇 (𝑓 ) −
𝑠𝑇 (𝑓 ).
𝜀 𝜀
𝐼− < 𝜎𝑇 (𝑓, 𝜉) < 𝐼 + ,
2 2
𝜀 𝜀
𝐼− ⩽ 𝑠𝑇 (𝑓 ) ⩽ 𝑆𝑇 (𝑓 ) ⩽ 𝐼 + ,
2 2

ФПМИ МФТИ, весна 2023 4


Математический анализ Назад к содержанию

𝑥𝑘
⃒ ∫︁ ⃒
⃒ ⃒
⃒𝜎𝑇 (𝑓, 𝜉𝑘 ) − 𝑓 (𝑥)𝑑𝑥⃒⃒ ⩽ 𝜀.
⃒ 𝑘
𝑥0
∑︀𝑘
Положим 𝐴𝑘 = 𝑖=1 𝑓 (𝑐𝑖 )∆𝑥𝑖 . Тогда 𝐴𝑘 = 𝜎𝑇𝑘 (𝑓, 𝜉𝑘 ) и, значит, из последнего неравенства
|𝐴𝑘 | ⩽ 𝑀 + 𝜀. Применим лемму 1 для 𝑎𝑘 = 𝑓 (𝑐𝑘 )∆𝑥𝑘 , 𝑏𝑘 = 𝑔(𝑐𝑘 ), получим
⃒ ⃒
⃒∑︁𝑛 ⃒
𝑓 (𝑐𝑘 )𝑔(𝑐𝑘 )∆𝑥𝑘 ⃒ ⩽ 2(𝑀 + 𝜀)(|𝑔(𝑐1 )| + |𝑔(𝑐𝑛 )|).
⃒ ⃒

⃒ ⃒
𝑘=1

Неравенство верно для любого набора отмеченных точек, в том числе и 𝑐1 = 𝑎, 𝑐𝑛 = 𝑏. Предель-
ным переходом по мелкости разбиения в случае 𝑐1 = 𝑎, 𝑐𝑛 = 𝑏 получим искомое неравенство.

Определение 2. Функция 𝑓 называется локально интегрируемой по Риману на промежутке


𝐼, если ∀[𝑎, 𝑐] ⊂ 𝐼 ˓→ 𝑓 ∈ ℛ[𝑎, 𝑐].

Определение 3. Пусть −∞ < 𝑎 < 𝑏 ⩽ +∞, и 𝑓 локально интегрируема на [𝑎, 𝑏). Предел
∫︁ 𝑏 ∫︁ 𝑐
𝑓 (𝑥)𝑑𝑥 := lim 𝑓 (𝑥)𝑑𝑥
𝑎 𝑐→𝑏−0 𝑎

называется несобственным интегралом (Римана) от 𝑓 на [𝑎, 𝑏).


∫︀ 𝑏
Если предел существует и конечен, то интеграл 𝑎 𝑓 (𝑥)𝑑𝑥 называется сходящимся, иначе —
расходящимся.

Свойство 1 (принцип локализации). Пусть 𝑓 локально интегрируема на [𝑎, 𝑏), 𝑎* ∈ (𝑎, 𝑏). Тогда
∫︀ 𝑏 ∫︀ 𝑏
интегралы 𝑎* 𝑓 (𝑥)𝑑𝑥 и 𝑎 𝑓 (𝑥)𝑑𝑥 сходятся или расходятся одновременно, и если сходятся, то
∫︁ 𝑏 ∫︁ 𝑎* ∫︁ 𝑏
𝑓 (𝑥)𝑑𝑥 = 𝑓 (𝑥)𝑑𝑥 + 𝑓 (𝑥)𝑑𝑥
𝑎 𝑎 𝑎*

Доказательство. Если 𝑐 ∈ (𝑎, 𝑏), то по свойству аддитивности определённого интеграла верно:


∫︁ 𝑐 ∫︁ 𝑎* ∫︁ 𝑐
𝑓 (𝑥)𝑑𝑥 = 𝑓 (𝑥)𝑑𝑥 + 𝑓 (𝑥)𝑑𝑥.
𝑎 𝑎 𝑎*
∫︀ 𝑐 ∫︀ 𝑏 ∫︀ 𝑐 ∫︀ 𝑏
Поэтому пределы lim𝑐→𝑏−0 𝑎* 𝑓 (𝑥)𝑑𝑥 = 𝑎* 𝑓 (𝑥)𝑑𝑥 и lim𝑐→𝑏−0 𝑎 𝑓 (𝑥)𝑑𝑥 = 𝑎 𝑓 (𝑥)𝑑𝑥 существу-
ют (конечны) одновременно. Равенство 1 получается из равенства для определённых интегралов
переходом к пределу 𝑐 → 𝑏 − 0.

Следующие три свойства доказываются аналогично.


∫︀ 𝑏 ∫︀ 𝑏
Свойство 2 (линейность). Пусть несобственные интегралы 𝑎 𝑓 (𝑥)𝑑𝑥 и 𝑎 𝑔(𝑥)𝑑𝑥 сходятся, и
∫︀ 𝑏
𝛼, 𝛽 ∈ R. Тогда сходится интеграл 𝑎 (𝛼𝑓 (𝑥) + 𝛽𝑔(𝑥)) 𝑑𝑥 и
∫︁ 𝑏 ∫︁ 𝑏 ∫︁ 𝑏
(𝛼𝑓 (𝑥) + 𝛽𝑔(𝑥)) 𝑑𝑥 = 𝛼 𝑓 (𝑥)𝑑𝑥 + 𝛽 𝑔(𝑥)𝑑𝑥.
𝑎 𝑎 𝑎

Свойство 3 (формула Ньютона-Лейбница). Пусть 𝑓 локально интегрируема на [𝑎, 𝑏) и 𝐹 —


первообразная 𝑓 на [𝑎, 𝑏). Тогда
∫︁ 𝑏
𝑓 (𝑥)𝑑𝑥 = 𝐹 (𝑏 − 0) − 𝐹 (𝑎) = 𝐹 |𝑏−0
𝑎 .
𝑎

ФПМИ МФТИ, весна 2023 5


Математический анализ Назад к содержанию

Свойство 4 (интегрирование по частям). Пусть 𝐹, 𝐺 дифференцируемы, а их производные 𝑓, 𝑔


локально интегрируемы на [𝑎, 𝑏). Тогда
∫︁ 𝑏 ∫︁ 𝑏
𝐹 (𝑥)𝑔(𝑥)𝑑𝑥 = 𝐹 (𝑥)𝐺(𝑥)|𝑏−0
𝑎 − 𝐺(𝑥)𝑓 (𝑥)𝑑𝑥.
𝑎 𝑎

Существование двух из трёх конечных пределов влечёт существование третьего и выпол-


нение равенства.

Свойство 5 (замена переменной). Пусть 𝑓 непрерывна на [𝑎, 𝑏), 𝜙 дифференцируема и строго


монотонна на [𝛼, 𝛽), причем 𝜙′ локально интегрируема на [𝛼, 𝛽), 𝜙(𝛼) = 𝑎,
lim 𝜙(𝑡) = 𝑏. Тогда
𝑡→𝛽−0
∫︁ 𝑏 ∫︁ 𝛽
𝑓 (𝑥)𝑑𝑥 = 𝑓 (𝜙(𝑡))𝜙′ (𝑡)𝑑𝑡.
𝑎 𝛼
Если существует один из интегралов, то существует и другой, и равенство выполняется.
∫︀ 𝑐 ∫︀ 𝛾
Доказательство. Рассмотрим частичный интеграл 𝐹 (𝑐) = 𝑎 𝑓 (𝑥)𝑑𝑥 на [𝑎, 𝑏), Φ(𝛾) = 𝛼 𝑓 (𝜙(𝑡))𝜙′ (𝑡)𝑑𝑡.
По свойству замены переменной в определенном интеграле 𝐹 (𝜙(𝛾)) = Φ(𝛾) ∀𝛾 ∈ [𝛼, 𝛽). Пусть (в
∫︀ 𝑏
R) определен интеграл 𝐼 = 𝑎 𝑓 (𝑥)𝑑𝑥. По свойству предела композиции существует lim𝛾→𝛽−0 Φ(𝛾) =
∫︀ 𝛽
lim𝑐→𝑏−0 𝐹 (𝑐) = 𝐼. Следовательно, определен 𝛼 𝑓 (𝜙(𝑡))𝜙′ (𝑡)𝑑𝑡 = 𝐼.
Из условия следует, что существует обратная функция 𝜙−1 и 𝛾 = 𝜙−1 (𝑐) → 𝛽 при 𝑐 → 𝑏 − 0.
Делая соответствующую замену переменной, получим, что lim𝛾→𝛽−0 Φ(𝛾) влечет существование
равного lim𝑐→𝑏−0 𝐹 (𝑐), то есть интеграл в правой части влечет существование интеграла в левой
и их равенство.

Теорема 1 (критерий Коши). Пусть 𝑓 локально интегрируема на [𝑎, 𝑏). Для сходимости ин-
∫︀ 𝑏
теграла 𝑎 𝑓 (𝑥)𝑑𝑥 необходимо и достаточно выполнения условия Коши:
(︂⃒∫︁ 𝜂 ⃒ )︂
⃒ ⃒
∀𝜀 > 0 ∃𝑏𝜀 ∈ [𝑎, 𝑏) ∀𝜉, 𝜂 ∈ (𝑏𝜀 , 𝑏) ⃒⃒ 𝑓 (𝑥)𝑑𝑥⃒⃒ < 𝜀 .
𝜉
∫︀ 𝑥 ∫︀ 𝜂
Доказательство. Положим 𝐹 (𝑥) = 𝑎 𝑓 (𝑡)𝑑𝑡, 𝑥 ∈ [𝑎, 𝑏). Поскольку 𝜉 𝑓 (𝑥)𝑑𝑥 = 𝐹 (𝜂) − 𝐹 (𝜉),
то доказательство утверждения является переформулировкой критерия Коши существования
предела 𝐹 при 𝑥 → 𝑏 − 0.
∫︀ 𝑏
Определение 4. Пусть 𝑓 локально интегрируема на [𝑎, 𝑏). Несобственный интеграл 𝑎 𝑓 (𝑥)𝑑𝑥
∫︀ 𝑏 ∫︀ 𝑏
называется абсолютно сходящимся, если сходится интеграл 𝑎 |𝑓 (𝑥)|𝑑𝑥. Если интеграл 𝑎 𝑓 (𝑥)𝑑𝑥
сходится, но не сходится абсолютно, то он называется условно сходящимся.

Следствие 1. Абсолютно сходящийся интеграл сходится.


∫︀ 𝑏
Доказательство. Зафиксируем
(︁∫︀ 𝜀 > 0.
)︁ Так как 𝑎 |𝑓 (𝑥)|𝑑𝑥 сходится,
⃒∫︀ ⃒ по∫︀критерию Коши ∃𝑏𝜀 ∈
то
𝜂 ⃒ 𝜂 𝜂
[𝑎, 𝑏) ∀[𝜉, 𝜂] ⊂ (𝑏𝜀 , 𝑏) 𝜉 |𝑓 (𝑥)|𝑑𝑥 < 𝜀 . Но тогда тем более ⃒ 𝜉 𝑓 (𝑥)𝑑𝑥⃒ ⩽ 𝜉 |𝑓 (𝑥)|𝑑𝑥 < 𝜀. Следо-

вательно, по критерию Коши, интеграл сходится.

Лемма 3. Пусть 𝑓 локально интегрируема и неотрицательна на [𝑎, 𝑏). Тогда сходимость ин-
∫︀ 𝑏 ∫︀ 𝑥
теграла 𝑎 𝑓 (𝑥)𝑑𝑥 равносильна ограниченности функции 𝐹 (𝑥) = 𝑎 𝑓 (𝑡)𝑑𝑡 на [𝑎, 𝑏).

Доказательство. Функция 𝐹 нестрого возрастает на [𝑎, 𝑏), так как


∫︁ 𝑥2
𝑥1 < 𝑥2 ⇒ 𝐹 (𝑥2 ) − 𝐹 (𝑥1 ) = 𝑓 (𝑡)𝑑𝑡 ⩾ 0.
𝑥1

ФПМИ МФТИ, весна 2023 6


Математический анализ Назад к содержанию

По теореме о пределах монотонной функции существует lim𝑥→𝑏−0 𝐹 (𝑥) = sup[𝑎,𝑏) 𝐹 (𝑥). Отсюда,
учитывая неотрицательность, заключаем, что ограниченность 𝐹 равносильна наличию конечного
предела, то есть сходимости интеграла.

Теорема 2 (признак сравнения). Пусть функции 𝑓, 𝑔 локально интегрируемы на [𝑎, 𝑏), и 0 ⩽


𝑓 ⩽ 𝑔 на [𝑎, 𝑏).
∫︀ 𝑏 ∫︀ 𝑏
1. Если 𝑎 𝑔(𝑥)𝑑𝑥 сходится, то 𝑎 𝑓 (𝑥)𝑑𝑥 сходится.
∫︀ 𝑏 ∫︀ 𝑏
2. Если 𝑎 𝑓 (𝑥)𝑑𝑥 расходится, то 𝑎 𝑔(𝑥)𝑑𝑥 расходится.
∫︀ 𝑥 ∫︀ 𝑥 ∫︀ 𝑏
Доказательство. Для любого 𝑥 ∈∫︀ [𝑎, 𝑏) выполнено 0 ⩽ 𝑎 𝑓 (𝑡)𝑑𝑡 ⩽ 𝑎 𝑔(𝑡)𝑑𝑡. Если 𝑎 𝑔(𝑥)𝑑𝑥
𝑥
сходится, то по лемме 3 функция 𝑎 𝑔(𝑡)𝑑𝑡 ограничена на [𝑎, 𝑏). Но тогда на [𝑎, 𝑏) ограничена и
∫︀ 𝑥 ∫︀ 𝑏
функция 𝑎 𝑓 (𝑡)𝑑𝑡 и, значит, по лемме 3 интеграл 𝑎 𝑓 (𝑥)𝑑𝑥 сходится.
Второе доказываемое утверждение является контрапозицией первого.

Следствие 2. Пусть 𝑓, 𝑔 локально интегрируемы и неотрицательны на [𝑎, 𝑏). Если 𝑓 (𝑥) = 𝑂(𝑔(𝑥)),
то справедливо заключение теоремы 2.
Доказательство. По определению и неотрицательности функции

∃𝐶 > 0 ∃𝑎* ∈ [𝑎, 𝑏) ∀𝑥 ∈ [𝑎* , 𝑏) (𝑓 (𝑥) ⩽ 𝐶𝑔(𝑥)) .


∫︀ 𝑏 ∫︀ 𝑏
Если 𝑎 𝑔(𝑥)𝑑𝑥 сходится, то сходится интеграл 𝑎* 𝐶𝑔(𝑥)𝑑𝑥. Тогда по признаку сравнения сходится
∫︀ 𝑏 ∫︀ 𝑏
интеграл 𝑎* 𝑓 (𝑥)𝑑𝑥 и, значит, сходится интергал 𝑎 𝑓 (𝑥)𝑑𝑥.

Следствие 3. Пусть 𝑓, 𝑔 локально интегрируемы и положительны на [𝑎, 𝑏).


∫︀ 𝑏 ∫︀ 𝑏
Если существует lim𝑥→𝑏−0 𝑓𝑔(𝑥)
(𝑥)
∈ (0, +∞), то интегралы 𝑎 𝑓 (𝑥)𝑑𝑥 и 𝑎 𝑔(𝑥)𝑑𝑥 сходятся или
расходятся одновременно.
Доказательство. По условию также существует lim𝑥→𝑏−0 𝑓𝑔(𝑥)
(𝑥) ∈ (0, +∞). Поскольку существо-
вание конечного предела влечёт ограниченность функции в некоторой окрестности предельной
точки, то утверждение вытекает из следствия 2.

Лемма 4 (метод выделения главной части). Пусть функции 𝑓, 𝑔 локально интегрируемы на


[𝑎, 𝑏).
∫︀ 𝑏 ∫︀ 𝑏 ∫︀ 𝑏
1. Если 𝑎 𝑔(𝑥)𝑑𝑥 сходится, то интегралы 𝑎 𝑓 (𝑥)𝑑𝑥 и 𝑎 (𝑓 (𝑥) + 𝑔(𝑥))𝑑𝑥 сходятся или расхо-
дятся одновременно.
∫︀ 𝑏 ∫︀ 𝑏 ∫︀ 𝑏
2. Если 𝑎 𝑔(𝑥)𝑑𝑥 абсолютно сходится, то интегралы 𝑎 𝑓 (𝑥)𝑑𝑥 и 𝑎 (𝑓 (𝑥) + 𝑔(𝑥))𝑑𝑥 либо од-
новременно расходятся, либо одновременно сходятся условно, либо одновременно сходятся
абсолютно.
Доказательство. Первый пункт вытекает из линейности несобственных интегралов. Одновре-
менная расходимость вытекает из первого пункта по неравенствам |𝑓 +𝑔| ⩽ |𝑓 |+|𝑔|, |𝑓 | ⩽ |𝑓 +𝑔|+|𝑔|
и признаку сравнения.

Теорема 3 (признак Дирихле). Пусть 𝑓, 𝑔 локально интегрируемы на [𝑎, 𝑏), причём


∫︀ 𝑥
1. 𝐹 (𝑥) = 𝛼 𝑓 (𝑡) 𝑑𝑡 ограничена на [𝑎, 𝑏);

2. 𝑔 монотонна на [𝑎, 𝑏);

3. lim 𝑔(𝑥) = 0.
𝑥→𝑏−0

ФПМИ МФТИ, весна 2023 7


Математический анализ Назад к содержанию

∫︀ 𝑏
Тогда несобственный интеграл 𝑓 (𝑥)𝑔(𝑥) 𝑑𝑥 сходится.
𝑎
∫︀ 𝑏
Доказательство. Покажем, что интеграл 𝑎 𝑓 (𝑥)𝑔(𝑥) 𝑑𝑥 удовлетворяет условию Коши.
Пусть |𝐹 | < 𝑀 на [𝑎, 𝑏), тогда для любого 𝜉 ∈ [𝑎, 𝑏) выполнено:
⃒∫︁ 𝑥 ⃒
⃒ ⃒

⃒ 𝑓 (𝑡) 𝑑𝑡⃒⃒ = |𝐹 (𝑥) − 𝐹 (𝜉)| < 2𝑀.
𝜉

Зафиксируем 𝜀 > 0. Тогда, по определению предела для 𝑔(𝑥), ∃𝑏′ ∈ [𝑎, 𝑏) ∀𝑥 ∈ (𝑏′ , 𝑏) |𝑔(𝑥)| < 𝜀
(︀ )︀
8𝑀 .
Тогда по лемме Абеля (1) для любого [𝜉, 𝜂] ⊂ (𝑏′ , 𝑏):
⃒∫︁ 𝜂 ⃒ (︁ 𝜀
⃒ ⃒
⃒ ⩽ 2 · 2𝑀 (|𝑔(𝜉)| + |𝑔(𝜂)|) < 4𝑀 𝜀 )︁
⃒ 𝑓 (𝑡)𝑔(𝑡) 𝑑𝑡 + = 𝜀.

𝜉
⃒ 8𝑀 8𝑀
∫︀ 𝑏
По критерию Коши интеграл 𝑎 𝑓 (𝑥)𝑔(𝑥) 𝑑𝑥 сходится.

Замечание. Условия последней теоремы выполнены, если 𝑓 непрерывна и имеет ограниченную


первообразную на [𝑎, 𝑏), 𝑔 дифференцируема, 𝑔 ′ сохраняет знак на [𝑎, 𝑏), 𝑔(𝑥) → 0 при 𝑥 → 𝑏 − 0.
Теорема 4 (признак Абеля). Пусть 𝑓, 𝑔 локально интегрируемы на [𝑎, 𝑏), причём
∫︀ 𝑏
1. 𝑎 𝑓 (𝑥) 𝑑𝑥 сходится;
2. 𝑔 монотонна на [𝑎, 𝑏);
3. 𝑔 ограничена на [𝑎, 𝑏).
∫︀ 𝑏
Тогда несобственный интеграл 𝑎 𝑓 (𝑥)𝑔(𝑥) 𝑑𝑥 сходится.

Доказательство. Так как 𝑔 монотонна и ограничена на [𝑎, 𝑏), то ∃ lim 𝑔(𝑥) = 𝑐 ∈ R.


𝑥→𝑏−0
∫︀ 𝑏
Функции 𝑓 и 𝑔−𝑐 удовлетворяют условиям признака Дирихле (3), поэтому 𝑎 𝑓 (𝑥) (𝑔(𝑥) − 𝑐) 𝑑𝑥
∫︀ 𝑏 ∫︀ 𝑏 ∫︀ 𝑏
сходится. Тогда 𝑎 𝑓 (𝑥)𝑔(𝑥) 𝑑𝑥 = 𝑎 𝑓 (𝑥)(𝑔(𝑥) − 𝑐) 𝑑𝑥 + 𝑐 𝑎 𝑓 (𝑥) 𝑑𝑥 сходится как сумма сходящихся
интегралов.

Определение 5. Пусть 𝑎, 𝑏 ∈ R, 𝑎 < 𝑏 и функция 𝑓 определена на (𝑎, 𝑏), кроме, быть может,
конечного множества точек.
Точка 𝑐 ∈ (𝑎, 𝑏) называется особенностью 𝑓 , если 𝑓 ̸∈ ℛ[𝛼, 𝛽] для любого [𝛼, 𝛽] ⊂ (𝑎, 𝑏),
𝛼 < 𝑐 < 𝛽.
Точка 𝑏 называется особенностью 𝑓 , если 𝑏 = +∞, или 𝑏 ∈ R и 𝑓 ̸∈ ℛ[𝛼, 𝛽], для любого [𝛼, 𝛽].
Аналогично вводится особенность 𝑎.
Замечание. Если на (𝑐, 𝑑) нет особенностей функции 𝑓 , то 𝑓 локально интегрируема на (𝑐, 𝑑).

Доказательство. Пусть [𝑢, 𝑣] ⊂ (𝑐, 𝑑). По условию ∀𝑥 ∈ (𝑢𝑥 , 𝑣𝑥 ) и 𝑓 ∈ ℛ[𝑢𝑥 , 𝑣𝑥 ].


Тогда {(𝑢𝑥 , 𝑣𝑥 )}𝑥∈[𝑢,𝑣] образуют открытое покрытие [𝑢, 𝑣]. По лемме Гейне-Бореля из открытого
покрытия можно выделить конечное подпокрытие.
Объединяя элементы этого подпокрытия и пользуясь аддитивностью интеграла, заключаем,
что 𝑓 интегрируема на отрезке, содержащем [𝑢, 𝑣], и, значит, 𝑓 локально интегрируема на (𝑐, 𝑑),
так как [𝑢, 𝑣] выбирался произвольным.

Определение 6. Пусть 𝑐1 < 𝑐2 < . . . < 𝑐𝑁 −1 – все особенности функции 𝑓 на (𝑎, 𝑏), 𝑐0 = 𝑎,
𝑐𝑁 = 𝑏. Пусть 𝜉𝑘 ∈ (𝑐𝑘−1 , 𝑐𝑘 ), 𝑘 = 1, . . . , 𝑁 . Несобственным интегралом функции 𝑓 по (𝑎, 𝑏)
называется
𝑁
∫︁ 𝑏 (︃∫︁ )︃
∑︁ 𝜉𝑘 ∫︁ 𝑐𝑘
𝑓 (𝑥)𝑑𝑥 = 𝑓 (𝑥)𝑑𝑥 + 𝑓 (𝑥)𝑑𝑥 ,
𝑎 𝑘=1 𝑐𝑘−1 𝜉𝑘

ФПМИ МФТИ, весна 2023 8


Математический анализ Назад к содержанию

если все интегралы в правой части (понимаются как несобственные) и их сумма имеет смысл в
R. ∫︀ 𝑏
При этом 𝑎 𝑓 (𝑥)𝑑𝑥 называется сходящимся, если все интегралы в правой части сходятся,
иначе – расходящимся.

Замечание. Корректность (независимость от выбора 𝜉𝑘 ) следует из принципа локализации.

ФПМИ МФТИ, весна 2023 9


Математический анализ Назад к содержанию

2. Числовые ряды и их свойства. Группировка ряда. Критерий


Коши. Абсолютная и условная сходимости рядов. Связь сходимо-
сти ряда и интеграла от ступенчатой функции. Ряды с неотри-
цательными членами. Признак сравнения, интегральный признак.
Признаки Коши, Даламбера, Гаусса (б/д). Знакопеременные ряды.
Признак Лейбница. Признаки Дирихле (б/д) и Абеля (б/д). Пере-
становка членов абсолютно сходящегося ряда. Теорема Римана о
перестановке (б/д). Произведение абсолютно сходящихся рядов.
Определение 7. Пусть {𝑎𝑛 } – последовательность действительных (комплексных) чисел. Вы-
ражение
+∞
∑︁
𝑎𝑛 = 𝑎1 + 𝑎2 + . . .
𝑛=1

называется числовым рядом с 𝑛-ым членом 𝑎𝑛 .


Число
𝑁
∑︁
𝑆𝑁 = 𝑎𝑛 = 𝑎1 + . . . + 𝑎𝑁
𝑛=1

называется N-ой частичной суммой ряда 7.


Предел
+∞
∑︁
𝑎𝑛 = lim 𝑆𝑁
𝑁 →+∞
𝑛=1

называется суммой ряда 7. Если предел конечен, то ряд называется сходящимся, иначе – расхо-
дящимся.

Лемма 5 (Принцип локализации). Для каждого 𝑚 ∈ N ряды +∞


∑︀ ∑︀+∞
𝑛=1 𝑎𝑛 и 𝑛=𝑚+1 𝑎𝑛 сходятся
или расходятся одновременно, и если сходятся, то
+∞
∑︁ 𝑚
∑︁ +∞
∑︁
𝑎𝑛 = 𝑎𝑛 + 𝑎𝑛 .
𝑛=1 𝑛=1 𝑛=𝑚+1

Доказательство. Если 𝑁 > 𝑚, то 𝑁


∑︀ ∑︀𝑚 ∑︀𝑁
𝑛=1 𝑎𝑛 = 𝑛=1 𝑎𝑛 + 𝑛=𝑚+1 𝑎𝑛 . Поэтому пределы последо-
∑︀𝑚 ∑︀𝑁
вательностей 𝑛=1 𝑎𝑛 и 𝑛=𝑚+1 𝑎𝑛 при 𝑁 → +∞ существуют (конечны) одновременно.

Замечание. Ряд 𝑟𝑁 = +∞
∑︀
𝑛=𝑁 +1 𝑎𝑛 называется 𝑁 -ым остатком ряда 7.

Принцип локализации можно переформулировать так: если ряд сходится, то сходится и любой
его остаток. И если сходится некоторый остаток ряда, то и весь ряд сходится.
∑︀+∞ ∑︀+∞
Лемма 6 (Линейность).
∑︀+∞ Пусть ряды 𝑛=1 𝑎𝑛 и 𝑛=1 𝑏𝑛 сходятся, и 𝛼, 𝛽 ∈ R (или C), то
сходится и ряд 𝑛=1 (𝛼𝑎𝑛 + 𝛽𝑏𝑛 ), причем верно равенство
+∞
∑︁ +∞
∑︁ +∞
∑︁
(𝛼𝑎𝑛 + 𝛽𝑏𝑛 ) = 𝛼 𝑎𝑛 + 𝛽 𝑏𝑛 .
𝑛=1 𝑛=1 𝑛=1

Доказательство. Вытекает из линейности предела последовательности.

Лемма 7 (Необходимое условие сходимости ряда). Если +∞


∑︀
𝑛=1 𝑎𝑛 сходится, то 𝑎𝑛 → 0.

ФПМИ МФТИ, весна 2023 10


Математический анализ Назад к содержанию

∑︀+∞
Доказательство. Пусть 𝑆 = 𝑛=1 𝑎𝑛 . Так как 𝑎𝑛 = 𝑆𝑛 − 𝑆𝑛−1 (считаем, что 𝑆0 = 0), то 𝑎𝑛 →
(𝑆 − 𝑆) = 0.

Определение 8. Пусть дана строго возрастающая последовательность целых чисел 0 = 𝑛0 <


𝑛1 < 𝑛2 ∑︀
< ...
Ряд +∞
∑︀+∞
𝑘=1 𝑏𝑘 , где 𝑏𝑘 = 𝑎𝑛𝑘−1 +1 + . . . + 𝑎𝑛𝑘 называется группировкой ряда 𝑛=1 𝑎𝑛 .

Лемма 8. 1. Если ряд сходится, то сходится и любая его группировка, причем к той же
сумме.
∑︀𝑛𝑘
2. Пусть ∃𝐿 ∀𝑘 (𝑛𝑘 − 𝑛𝑘−1 ⩽ 𝐿). Если 𝑎𝑛 → 0 и группировка +∞
∑︀
𝑘=1 𝑏𝑘 , где 𝑏𝑘 = 𝑗=𝑛𝑘−1 +1 𝑎𝑗 ,
∑︀+∞
сходится, то сходится и ряд 𝑛=1 𝑎𝑛 , причем к той же сумме.
* – 𝑁 -ую частичную сумму
Доказательство. Пусть 𝑆𝑁 обозначает 𝑁 -ую частичную сумму 7, 𝑆𝑁
группировки.
* =𝑆
1. Пусть 𝑆𝑁 → 𝑆. Так как 𝑆𝑁 *
𝑛𝑁 , то 𝑆𝑁 → 𝑆 как подпоследовательность.

2. Пусть 𝜀 > 0. Выберем такие 𝐾, 𝑀 ∈ N, что ∀𝑘 ⩾ 𝐾 ˓→ |𝑆𝑘* − 𝑆| < 2𝜀 и ∀𝑚 ⩾ 𝑀 ˓→ |𝑎𝑚 | < 𝜀


2𝐿 .
Положим 𝑁 = max{𝑛𝐾 , 𝑀 + 𝐿}. Если 𝑛 ⩾ 𝑁 , то 𝑛𝑘 ⩽ 𝑛 < 𝑛𝑘+1 , где 𝑘 ⩾ 𝐾. Значит,
𝜀 𝜀
|𝑆𝑛 − 𝑆| = |𝑆𝑛𝑘 + 𝑎𝑛𝑘 +1 + · · · + 𝑎𝑛 − 𝑆| ⩽ |𝑆𝑘* − 𝑆| + |𝑎𝑛𝑘 +1 | + · · · + |𝑎𝑛 | < +𝐿 = 𝜀.
2 2𝐿

Применяя критерий Коши для последовательности частичных сумм получаем критерий Коши
сходимости числового ряда.

Теорема 5. Для сходимости ряда +∞


∑︀
𝑛=1 𝑎𝑛 необходимо и достаточно выполнения условия Коши
(︃⃒ 𝑛 ⃒ )︃
⃒ ∑︁ ⃒
∀𝜀 > 0 ∃𝑁 ∈ N ∀𝑛, 𝑚 ∈ N, 𝑁 ⩽ 𝑚 ⩽ 𝑛 ⃒ 𝑎𝑘 ⃒ < 𝜀 .
⃒ ⃒
⃒ ⃒
𝑘=𝑚

9. Ряд +∞
∑︀ ∑︀+∞
Определение
∑︀+∞ 𝑛=1 𝑎𝑛 называется абсолютно сходящимся, если сходится ряд 𝑛=1 |𝑎𝑛 |.
Если ряд 𝑛=1 𝑎𝑛 сходится, но не сходится абсолютно, то он называется условно сходящимся.

Следствие 4. Абсолютно сходящийся ряд сходится.

Доказательство. Для любых 𝑚, 𝑛 ∈ N, 𝑚 ⩽ 𝑛,


⃒ ⃒
𝑛
⃒ ∑︁ ⃒ ∑︁𝑛
𝑎𝑘 ⃒ ⩽ |𝑎𝑘 |.
⃒ ⃒

⃒ ⃒
𝑘=𝑚 𝑘=𝑚
∑︀+∞
Поэтому,
∑︀+∞ если ряд 𝑛=1 |𝑎𝑛 | удовлетворяет условию Коши, то условию Коши удовлетворяет ряд
𝑎
𝑛=1 𝑛 .
∑︀+∞
Замечание. Если ряд 𝑛=1 𝑎𝑛 сходится абсолютно, то
⃒ ⃒
+∞
⃒∑︁ ⃒ ∑︁+∞
𝑎𝑘 ⃒ ⩽ |𝑎𝑘 |.
⃒ ⃒

⃒ ⃒
𝑘=1 𝑘=1
∑︀+∞ ∑︀+∞ ∑︀+∞
Лемма 9. 1. Если 𝑛=1 𝑏𝑛 сходится, то 𝑛=1 (𝑎𝑛 + 𝑏𝑛 ) и 𝑛=1 𝑎𝑛 сходятся или расходятся
одновременно.

ФПМИ МФТИ, весна 2023 11


Математический анализ Назад к содержанию

2. Если +∞
∑︀ ∑︀+∞ ∑︀+∞
𝑛=1 𝑏𝑛 абсолютно сходится, то 𝑛=1 (𝑎𝑛 + 𝑏𝑛 ) и 𝑛=1 𝑎𝑛 либо одновременно расхо-
дятся, либо одновременно сходятся условно, либо одновременно сходятся абсолютно.

Доказательство.

1. Следует из свойства линейности. Для всех 𝑛 ∈ N верно

|𝑎𝑛 + 𝑏𝑛 | ⩽ |𝑎𝑛 | + |𝑏𝑛 |, |𝑎𝑛 | ⩽ |𝑎𝑛 + 𝑏𝑛 | + |𝑏𝑛 |.

Следовательно, по признаку сравнения ряды +∞


∑︀ ∑︀+∞
𝑛=1 (𝑎𝑛 + 𝑏𝑛 ) и 𝑛=1 𝑎𝑛 одновременно абсо-
лютно сходятся.

2. Вытекает из пункта 1.

∫︀ +∞
Лемма 10. Ряд +∞
∑︀
𝑛=1 𝑎𝑛 и 1 𝑓𝑎 (𝑥)𝑑𝑥 сходятся или расходтся одновременно, и если сходятся,
то к одному значению.
∑︀𝑛 ∫︀ 𝑛+1
Доказательство. Пусть 𝑆𝑛 = 𝑘=1 𝑎𝑘 . Так как 𝑆𝑛 = 1 𝑓𝑎 (𝑥)𝑑𝑥, то сходимость интеграла
влечет сходимость ряда. Обратное утверждение следует из оценки
⃒ ∫︁ 𝑥 ⃒
⃒ ⃒
⃒𝑆𝑛 − 𝑓𝑎 (𝑥)𝑑𝑥⃒⃒ ⩽ |𝑎𝑛 | → 0

1

и необходимого условия сходимости ряда.


∑︀∞
Лемма 11. Пусть 𝑎𝑛 ⩾ 0 для всех 𝑛 ∈ N. Тогда сходимость ряда 𝑛=1 𝑎𝑛 равносильна ограни-
ченности последовательности частичных сумм {𝑆𝑛 }.

Доказательство. Все 𝑆𝑛 ⩾ 0 и нестрого возрастают, так как 𝑆𝑛+1 − 𝑆𝑛 = 𝑎𝑛+1 ⩾ 0. Следователь-


но, ∃ lim𝑛→∞ 𝑆𝑛 = sup 𝑆𝑛 .

Теорема 6 (признак сравнения). Пусть 0 ⩽ 𝑎𝑛 ⩽ 𝑏𝑛 для всех 𝑛 ∈ N.

1. Если ряд ∞
∑︀ ∑︀∞
𝑛=1 𝑏𝑛 сходится, то сходится и ряд 𝑛=1 𝑎𝑛 ;

2. Если ряд ∞
∑︀ ∑︀∞
𝑛=1 𝑎𝑛 расходится, то расходится и ряд 𝑛=1 𝑏𝑛 .

Доказательство. Вытекает из леммы (10) и признака сравнения несобственных интегралов.

Теорема 7 (интегральный признак ∑︀сходимости). Пусть ∫︀функция 𝑓 нестрого убывает и неот-


∞ +∞
рицательна на [1, +∞). Тогда ряд 𝑛=1 𝑓 (𝑛) и интеграл 1 𝑓 (𝑥) 𝑑𝑥 сходятся или расходятся
одновременно.

Доказательство. Положим 𝑢, 𝑣 : [1, +∞) → R, 𝑢|[𝑛,𝑛+1) = 𝑓 (𝑛), 𝑣|[𝑛,𝑛+1) = 𝑓 (𝑛 + 1).


Так как∑︀𝑓 нестрого убывает, то 𝑣 ⩽ 𝑓 ⩽ 𝑢 на [1, +∞). ∫︀ +∞
Пусть ∞ 𝑛=1 𝑓 (𝑛) сходится, тогда по лемме (10) сходится 1 𝑢(𝑥) 𝑑𝑥. Следовательно, по при-
∫︀ +∞
знаку сравнения для интегралов 1 𝑓 (𝑥) 𝑑𝑥 также сходится.
∫︀ +∞ ∫︀ +∞
Пусть 1 𝑓 (𝑥) 𝑑𝑥 сходится, ∑︀ тогда по признаку сравнения сходится 1 𝑣(𝑥) 𝑑𝑥. Следователь-
но, по лемме (10) сходится ряд ∞ 𝑛=1 𝑓 (𝑛 + 1).

Теорема 8 (признак Коши). Пусть 𝑎𝑛 ⩾ 0 для всех 𝑛 ∈ N и 𝑞 = lim𝑛→∞ 𝑛 𝑎𝑛 .
∑︀∞
1. Если 𝑞 < 1, то ряд 𝑛=1 𝑎𝑛
сходится;

̸ 0 и ряд ∞
∑︀
2. Если 𝑞 > 1, то 𝑎𝑛 → 𝑛=1 𝑎𝑛 расходится.

ФПМИ МФТИ, весна 2023 12


Математический анализ Назад к содержанию


Доказательство. 1. Пусть 𝑞0 ∈ (𝑞, 1). Выберем 𝑁 так, что sup𝑛⩾𝑁 𝑛 𝑎𝑛 < 𝑞0 при всех 𝑛 ⩾ 𝑁
и, значит, 𝑎𝑛 < 𝑞0𝑛 . Следовательно, ряд сходится по признаку сравнения с геометрическим
рядом.

2. Так как 𝑞 — частичный предел, то ∃{𝑛𝑘 } 𝑛𝑘 𝑎𝑛𝑘 → 𝑞. Поэтому ∃𝑘0 ∀𝑘 ⩾ 𝑘0 (𝑎𝑛𝑘 > 1), следо-
вательно, 𝑎𝑛 ̸→ 0 и ряд расходится.

Теорема 9 (признак Даламбера). Пусть 𝑎𝑛 > 0 для всех 𝑛 ∈ N.


∑︀∞
1. Если 𝑟 = lim𝑛→∞ 𝑎𝑛+1
𝑎𝑛 < 1, то ряд 𝑛=1 𝑎𝑛
сходится;

2. Если 𝑟 = lim𝑛→∞ 𝑎𝑛+1 ̸ 0 и ряд ∞


∑︀
𝑎𝑛 > 1, то 𝑎𝑛 → 𝑛=1 𝑎𝑛 расходится.

𝑎𝑛+1
Доказательство. 1. Пусть 𝑟 ∈ (𝑟, 1). Выберем 𝑁 так, что sup𝑛⩾𝑁 𝑎𝑛 < 𝑟 при всех 𝑛 ⩾ 𝑁 , и,
значит,
∀𝑛 > 𝑁 𝑎𝑛+1 < 𝑟𝑎𝑛 < . . . < 𝑟𝑛+1−𝑁 𝑎𝑁 = 𝑟1−𝑁 𝑎𝑁 𝑟𝑛 ,
и, значит, ряд сходится по признаку сравнения с геометрическим рядом ∞ 𝑛
∑︀
𝑛=1 𝑟 .
(︁ )︁
2. Пусть 𝑟 > 1. Тогда ∃𝑁 inf 𝑛⩾𝑁 𝑎𝑛+1𝑎𝑛 > 1 и, значит, 𝑎𝑛+1 > 𝑎𝑛 > . . . > 𝑎𝑁 > 0 для всех
𝑛 > 𝑁 . Следовательно, 𝑎𝑛 ̸→ 0 и ряд расходится.

Теорема 10 (признак Гаусса). Пусть 𝑎𝑛 > 0 для всех 𝑛 ∈ N и существуют такие 𝑠 > 1 и
ограниченная последовательность {𝛼𝑛 }, что для всех 𝑛 выполнено

𝑎𝑛+1 𝐴 𝛼𝑛
=1− + 𝑠.
𝑎𝑛 𝑛 𝑛
∑︀+∞
Тогда ряд 𝑛=1 𝑎𝑛 сходится при 𝐴 > 1 и расходится иначе.

Теорема
∑︀+∞ 11 (признак Лейбница). Пусть последовательность {𝛼𝑛 } монотонна и 𝛼𝑛 → 0. Тогда
ряд 𝑛=1 (−1)𝑛−1 𝛼𝑛 сходится, причем

|𝑆 − 𝑆𝑛 | ⩽ |𝛼𝑛+1 |.

Доказательство. Сходимость вытекает из признака Дирихле. Докажем ее прямо. Пусть для


определенности {𝛼𝑛 } нестрого убывает, и, значит, все {𝛼𝑛 } ⩾ 0.
𝑆2𝑛+2 − 𝑆2𝑛 = 𝛼2𝑛+1 − 𝛼2𝑛+2 ⩾ 0 ⇒ {𝑆2𝑛 } нестрого возрастает.
𝑆2𝑛+1 − 𝑆2𝑛−1 = −𝛼2𝑛 + 𝛼2𝑛+1 ⩽ 0 ⇒ {𝑆2𝑛−1 } нестрого убывает.
Кроме того, 𝑆2𝑛 − 𝑆2𝑛−1 = −𝛼2𝑛 ⩽ 0. Поэтому для любых 𝑚, 𝑛 ∈ N имеем

𝑆2𝑛 ⩽ 𝑆2𝑘 ⩽ 𝑆2𝑘−1 ⩽ 𝑆2𝑚−1 ,

где 𝑘 = max{𝑚, 𝑛}. Следовательно, последовательности {𝑆2𝑛 } и {𝑆2𝑛−1 } сходятся, 𝑆2𝑛 → 𝑆 ′ ,


𝑆2𝑛−1 → 𝑆 ′′ , и, в частности,
𝑆2𝑛 ⩽ 𝑆 ′ ⩽ 𝑆 ′′ ⩽ 𝑆2𝑛−1 .
Поскольку 𝑆2𝑛 − 𝑆2𝑛−1 = −𝛼2𝑛 → 0, то 𝑆 ′ = 𝑆 ′′ = 𝑆.

Теорема 12 (признак Дирихле). Пусть {𝑎𝑛 } – комплексная последовательность, {𝑏𝑛 } – дей-


ствительная последовательность, причем

1. Последовательность 𝐴𝑁 = 𝑁
∑︀
𝑛=1 𝑎𝑛 ограничена,

ФПМИ МФТИ, весна 2023 13


Математический анализ Назад к содержанию

2. {𝑏𝑛 } монотонна,

3. lim𝑛→+∞ 𝑏𝑛 = 0.
Тогда ряд +∞
∑︀
𝑛=1 𝑎𝑛 𝑏𝑛 сходится.

Теорема 13 (признак Абеля). Пусть {𝑎𝑛 } – комплексная последовательность, {𝑏𝑛 } – действи-


тельная последовательность, причем
1. Ряд +∞
∑︀
𝑛=1 𝑎𝑛 сходится,

2. {𝑏𝑛 } монотонна,

3. {𝑏𝑛 } ограничена.
Тогда ряд +∞
∑︀
𝑛=1 𝑎𝑛 𝑏𝑛 сходится.

Определение 10. Пусть дан ряд +∞


∑︀ ∑︀+∞
∑︀+∞ 𝑛=1 𝑎𝑛 и биекция 𝜙 : N → N. Тогда 𝑛=1 𝑎𝜙(𝑛) называется
перестановкой ряда 𝑛=1 𝑎𝑛 .
∑︀+∞ ∑︀+∞
Теорема 14. Если ряд 𝑛=1 𝑎𝑛 сходится абсолютно, то любая его перестановка 𝑛=1 𝑎𝜙(𝑛)
сходится абсолютно, причем к той же сумме.
Доказательство. Абсолютная сходимость перестановки следует из оценки
max{𝜙(𝑘)}
𝑁
∑︁ 1⩽𝑘⩽𝑁
∑︁ +∞
∑︁
|𝑎𝜙(𝑛) | ⩽ |𝑎𝑛 | ⩽ |𝑎𝑛 | < +∞.
𝑛=1 𝑛=1 𝑛=1

Пусть 𝜀 > 0. Выберем номер 𝑚 так, что +∞


∑︀
𝑛=𝑚+1 |𝑎𝑛 | < 𝜀. Выберем 𝑀 так, что {1, . . . , 𝑚} ⊂
{𝜙(1), . . . , 𝜙(𝑀 )} (достаточно положить −1
⃒∑︀ 𝑀 = max 1⩽𝑗⩽𝑚 𝜙 ⃒ (𝑗)). Тогда для любого 𝑁 ⩾ 𝑀 име-
ем {1, . . . , 𝑚} ⊂ {𝜙(1), . . . , 𝜙(𝑁 )} и ⃒ 𝑛=1 𝑎𝑛 − 𝑛=1 𝑎𝜙(𝑛) ⃒ ⩽ +∞
⃒ +∞ ∑︀𝑁 ∑︀
𝑛=𝑚+1 |𝑎𝑛 | < 𝜀. Таким образом,

частичные суммы перестановки сходятся у сумме исходного ряда.
∑︀+∞
Теорема 15 (Риман). Если ряд с действительными ∑︀+∞членами 𝑛=1 𝑎𝑛 сходится условно, то для
любого 𝐿 ∈ R существует такая перестановка 𝑛=1 𝑎𝜙(𝑛) , что её сумма равна 𝐿.

Теорема 16 (Коши).∑︀Пусть ряды +∞


∑︀ ∑︀+∞
𝑛=1 𝑎𝑛 и 𝑛=1 𝑏𝑛 сходятся абсолютно к 𝐴 и 𝐵 соответ-
+∞
ственно. Тогда ряд 𝑎
𝑛=1 𝑖𝑛 𝑗𝑛 𝑏 из всевозможных попарных произведений, занумерованных в
2
произвольном порядке (то есть, с 𝜙 : N → N , 𝜙(𝑛) = (𝑖𝑛 , 𝑗𝑛 ) – биекция) сходится абсолютно к
𝐴𝐵.

Доказательство. Покажем абсолютную сходимость ряда из произведений:


⎛ ⎞⎛ ⎞
𝑁 max1⩽𝑘⩽𝑁 𝑖𝑘 max1⩽𝑘⩽𝑁 𝑗𝑘 max1⩽𝑘⩽𝑁 𝑖𝑘 max1⩽𝑘⩽𝑁 𝑗𝑘
∑︁ ∑︁ ∑︁ ∑︁ ∑︁
|𝑎𝑖𝑛 𝑏𝑗𝑛 | ⩽ |𝑎𝑖 | · |𝑏𝑗 | = ⎝ |𝑎𝑖 |⎠ ⎝ |𝑏𝑗 |⎠ ⩽
𝑛=1 𝑖=1 𝑗=1 𝑖=1 𝑗=1
(︃ +∞ )︃ ⎛ +∞ ⎞
∑︁ ∑︁
⩽ |𝑎𝑖 | ⎝ |𝑏𝑗 |⎠ .
𝑖=1 𝑗=1

По теореме (14) любая перестановка ряда из произведений сходится к той


∑︀𝑁 ∑︀𝑁же сумме. Рас-
смотрим перестановку «по квадратам» и её частичную сумму 𝑆𝑁 2 = 𝑖=1 𝑗=1 𝑎𝑖 𝑏𝑗 . Так как
(︁∑︀ )︁ (︁∑︀ )︁
𝑁 𝑁
𝑆𝑁 2 = 𝑖=1 𝑎𝑖 𝑗=1 𝑏𝑗 → 𝐴𝐵 и если последовательность сходится, то и любая подпосле-
довательность
∑︀+∞ сходится к тому же пределу, то заключаем, что перестановка «по квадратам», а
значит, и 𝑛=1 𝑎𝑖𝑛 𝑏𝑗𝑛 , имеет сумму 𝐴𝐵.

ФПМИ МФТИ, весна 2023 14


Математический анализ Назад к содержанию

Ряд +∞
∑︀ ∑︀𝑛
Определение
∑︀+∞ 11.
∑︀+∞ 𝑛=1 𝑐𝑛 , где 𝑐𝑛 = 𝑘=1 𝑎𝑘 𝑏𝑛+1−𝑘 , называется произведением по Коши
рядов 𝑛=1 𝑎𝑛 и 𝑛=1 𝑏𝑛 .

Замечание. Если ряды +∞


∑︀ ∑︀+∞
𝑛=1 𝑎𝑛 и 𝑛=1 𝑏𝑛 сходятся абсолютно, то их произведение по Коши
сходится абсолютно к произведению сумм рядов.

ФПМИ МФТИ, весна 2023 15


Математический анализ Назад к содержанию

3. Поточечная и равномерная сходимость функциональных после-


довательностей и рядов, супремум-критерий. Арифметические свой-
ства. Критерий Коши равномерной сходимости. Непрерывность пре-
дельной функции и суммы ряда. Интегрируемость предельной функ-
ции и почленное интегрирование ряда. Дифференцируемость пре-
дельной функции и почленное дифференцирование ряда. Призна-
ки Вейерштрасса, Дирихле, Абеля равномерной сходимости рядов.
Пример ван-дер-Вардена (б/д).
Пусть 𝑓𝑛 , 𝑓 : 𝐸 → R (или C), 𝑛 ∈ N.

Определение 12. Последовательность {𝑓𝑛 } поточечно сходится к 𝑓 на 𝐸, если 𝑓 (𝑥) = lim𝑛→+∞ 𝑓𝑛 (𝑥)
для всех 𝑥 ∈ 𝐸.
Пишут 𝑓𝑛 → 𝑓 на 𝐸 и 𝑓 называют предельной функцией последовательности {𝑓𝑛 }.

Воспользуемся определением предела последовательности. 𝑓𝑛 → 𝑓 на 𝐸 тогда и только тогда,


когда ∀𝑥 ∈ 𝐸 ∀𝜀 > 0 ∃𝑁 ∈ N ∀𝑛 ⩾ 𝑁 (|𝑓𝑛 (𝑥) − 𝑓 (𝑥)| < 𝜀).
Если 𝑁 (𝑥) удаётся выбрать одним для всех 𝑥 ∈ 𝐸 (при фиксированном 𝜀), то приходим к
следующему понятию:

Определение 13. Последовательность {𝑓𝑛 } равномерно сходится к 𝑓 на 𝐸, если

∀𝜀 > 0 ∃𝑁 ∈ N ∀𝑛 ⩾ 𝑁 ∀𝑥 ∈ 𝐸 (|𝑓𝑛 (𝑥) − 𝑓 (𝑥)| < 𝜀) .

Пишут 𝑓𝑛 ⇒ 𝑓 на 𝐸 или 𝑓𝑛 ⇒𝐸 𝑓 при 𝑛 → +∞.

Замечание. Равномерная сходимость, очевидно, влечёт поточечную, но поточечная сходимость


не влечёт равномерную в общем случае.

Лемма 12 (супремум-критерий). 𝑓𝑛 ⇒ 𝑓 на 𝐸 тогда и только тогда, когда lim𝑛→+∞ 𝜌𝑛 = 0, где


𝜌𝑛 = sup𝑥∈𝐸 |𝑓𝑛 (𝑥) − 𝑓 (𝑥)|.

Доказательство.
(︂ )︂
(∀𝑥 ∈ 𝐸 (|𝑓𝑛 (𝑥) − 𝑓 (𝑥)| ⩽ 𝜀)) ⇔ sup |𝑓𝑛 (𝑥) − 𝑓 (𝑥)| ⩽ 𝜀 .
𝑥∈𝐸

∑︀+∞
Рассмотрим функциональный ряд 𝑛=1 𝑓𝑛 , где 𝑓𝑛 : 𝐸 → R (или C)
∑︀+∞
Определение 14. Функциональный ряд 𝑛=1 𝑓𝑛 поточечно сходится ∑︀+∞ на 𝐸, если числовой ряд
∑︀+∞
𝑛=1 𝑓𝑛 (𝑥) сходится для любого 𝑥 ∈ 𝐸. В этом случае 𝑆(𝑥) = 𝑛=1 𝑓𝑛 (𝑥), 𝑥 ∈ 𝐸, называется
суммой ряда +∞
∑︀
𝑓
𝑛=1 𝑛 .
Функциональный ряд +∞
∑︀
∑︀𝑁 𝑛=1 𝑓𝑛 равномерно сходится на 𝐸, если последовательность частич-
ных сумм 𝑆𝑁 = 𝑛=1 𝑓𝑛 равномерно сходится на 𝐸.

Свойство 6 (линейность). 1. Пусть 𝑓𝑛 ⇒ 𝑓 на 𝐸, 𝑔𝑛 ⇒ 𝑔 на 𝐸 и 𝛼, 𝛽 ∈ R (C). Тогда 𝛼𝑓𝑛 +


𝛽𝑔𝑛 ⇒ 𝛼𝑓 + 𝛽𝑔 на 𝐸.

2. Пусть +∞
∑︀ ∑︀+∞ ∑︀+∞
𝑛=1 𝑓𝑛 и 𝑔𝑛 равномерно сходятся
𝑛=1 ∑︀ на 𝐸. Тогда 𝑛=1 𝛼𝑓𝑛 + 𝛽𝑔𝑛 также равно-
мерно сходится на 𝐸 и +∞ +∞ +∞
∑︀ ∑︀
𝑛=1 𝛼𝑓𝑛 + 𝛽𝑔𝑛 = 𝛼 𝑓
𝑛=1 𝑛 + 𝛽 𝑛=1 𝑛 .
𝑔

ФПМИ МФТИ, весна 2023 16


Математический анализ Назад к содержанию

Доказательство. Пусть 𝑥 ∈ 𝐸. По неравенству треугольника

|(𝛼𝑓𝑛 (𝑥) + 𝛽𝑔𝑛 (𝑥)) − (𝛼𝑓 (𝑥) + 𝛽𝑔(𝑥))| ⩽ |𝛼| · |𝑓𝑛 (𝑥) − 𝑓 (𝑥)| + |𝛽| · |𝑔𝑛 (𝑥) − 𝑔(𝑥)|.

Далее по лемме (12).


Второй пункт вытекает из первого применением его к последовательности частичных сумм
ряда.

Свойство 7. Пусть 𝑔 : 𝐸 → R (C) ограничена.

1. Если 𝑓𝑛 ⇒ 𝑓 на 𝐸, то 𝑔𝑓𝑛 ⇒ 𝑔𝑓 на 𝐸.

2. Если +∞
∑︀ ∑︀+∞
𝑛=1 𝑓𝑛 равномерно сходится на 𝐸, то 𝑛=1 𝑔𝑓𝑛 также равномерно сходится на 𝐸 и

+∞
∑︁ +∞
∑︁
𝑔𝑓𝑛 = 𝑔 𝑓𝑛
𝑛=1 𝑛=1

Доказательство.

1. Пусть |𝑔(𝑥)| ⩽ 𝑀 для всех 𝑥 ∈ 𝐸. Тогда

sup |𝑔(𝑥)𝑓𝑛 (𝑥) − 𝑔(𝑥)𝑓 (𝑥)| ⩽ 𝑀 sup |𝑓𝑛 (𝑥) − 𝑓 (𝑥)| .


𝑥∈𝐸 𝑥∈𝐸
⏟ ⏞
→0

Значит, по супремум-критерию (12) 𝑔𝑓𝑛 ⇒ 𝑔𝑓 на 𝐸.

2. Вытекает из пункта 1 применением его к последовательности частичных сумм.

Теорема 17 (критерий Коши равномерной сходимости). Для равномерной сходимости {𝑓𝑛 } на


𝐸 необходимо и достаточно выполнения условия Коши:

∀𝜀 > 0 ∃𝑁 ∈ N ∀𝑛, 𝑚 ⩾ 𝑁 ∀𝑥 ∈ 𝐸 (|𝑓𝑛 (𝑥) − 𝑓𝑚 (𝑥)| < 𝜀) .

Доказательство.
(⇒) Пусть 𝜀 > 0. Из условия равномерной сходимости:
(︁ 𝜀 )︁
∃𝑁 ∈ N ∀𝑛 ⩾ 𝑁 ∀𝑥 ∈ 𝐸 |𝑓𝑛 (𝑥) − 𝑓 (𝑥)| < .
2
Тогда для всех 𝑛, 𝑚 ⩾ 𝑁 и 𝑥 ∈ 𝐸 имеем:
𝜀 𝜀
|𝑓𝑛 (𝑥) − 𝑓𝑚 (𝑥)| ⩽ |𝑓𝑛 (𝑥) − 𝑓 (𝑥)| + |𝑓𝑚 (𝑥) − 𝑓 (𝑥)| < + = 𝜀.
2 2
(⇐) Пусть {𝑓𝑛 } удовлетворяет (17). Тогда для каждого 𝑥 ∈ 𝐸 числовая последовательность
{𝑓𝑛 (𝑥)} фундаментальна и, значит, сходится. Положим 𝑓 (𝑥) = lim𝑛→∞ 𝑓𝑛 (𝑥), 𝑥 ∈ 𝐸. Пусть 𝜀 > 0
и номер 𝑁 из условия (17). Зафиксируем 𝑛 ⩾ 𝑁 в неравенстве и перейдем к пределу при 𝑚 → ∞.
Получим, что |𝑓𝑛 (𝑥) − 𝑓 (𝑥)| ⩽ 𝜀 при всех 𝑛 ⩾ 𝑁 и 𝑥 ∈ 𝐸. Так как 𝜀 > 0 – любое, то 𝑓𝑛 ⇒ 𝑓 на
𝐸.

Теорема 18 (о непрерывности предельной функции). Пусть 𝐸 ⊂ R и 𝑓𝑛 ⇒ 𝑓 на 𝐸. Если все 𝑓𝑛


непрерывны в точке 𝑎 ∈ 𝐸 (на 𝐸), то функция 𝑓 также непрерывна в точке 𝑎 (на 𝐸).

ФПМИ МФТИ, весна 2023 17


Математический анализ Назад к содержанию

Доказательство. Пусть 𝜀 > 0. Из условия равномерной сходимости:


(︁ 𝜀 )︁
∃𝑁 ∀𝑛 ⩾ 𝑁 ∀𝑥 ∈ 𝐸 |𝑓𝑛 (𝑥) − 𝑓 (𝑥)| < .
3
Тогда для 𝑥 ∈ 𝐸:
2
|𝑓 (𝑥) − 𝑓 (𝑎)| ⩽ |𝑓 (𝑥) − 𝑓𝑁 (𝑥)| + |𝑓𝑁 (𝑥) − 𝑓𝑁 (𝑎)| + |𝑓𝑁 (𝑎) − 𝑓 (𝑎)| < |𝑓𝑁 (𝑥) − 𝑓𝑁 (𝑎)| + 𝜀.
3
Так как 𝑓𝑁 непрерывна в точке 𝑎, то
(︁ 𝜀 )︁
∃𝛿 > 0 ∀𝑥 ∈ 𝐵𝛿 (𝑎) ∩ 𝐸 |𝑓𝑁 (𝑥) − 𝑓𝑁 (𝑎)| < .
3
Следовательно, |𝑓 (𝑥) − 𝑓 (𝑎)| < 𝜀 для всех 𝑥 ∈ 𝐵𝛿 (𝑎) ∩ 𝐸.

Следствие 5 (о непрерывности суммы ряда). Пусть ∞


∑︀
𝑛=1 𝑓𝑛 равномерно сходится на 𝐸 ⊂ R и
все 𝑓𝑛 непрерывны в точке 𝑎 ∈ 𝐸. Тогда сумма ряда непрерывна в точке 𝑎 (на 𝐸).

Теорема 19 (об интегрируемости предельной функции). Пусть 𝑓𝑛 ⇒ 𝑓 на [𝑎, 𝑏] и 𝑓𝑛 ∈ ℛ[𝑎, 𝑏] ∀𝑛.


∫︀ 𝑏 ∫︀ 𝑏
Тогда 𝑓 ∈ ℛ[𝑎, 𝑏] и lim𝑛→∞ 𝑎 𝑓𝑛 (𝑥)𝑑𝑥 = 𝑎 𝑓 (𝑥)𝑑𝑥.

Доказательство. Зафиксируем 𝜀 > 0. Из условия равномерной сходимости:


(︂ )︂
𝜀
∃𝑁 ∀𝑛 ⩾ 𝑁 ∀𝑥 ∈ [𝑎, 𝑏] |𝑓𝑛 (𝑥) − 𝑓 (𝑥)| < .
4(𝑏 − 𝑎)

Тогда на [𝑎, 𝑏]:


𝜀 𝜀
𝑓𝑁 (𝑥) − < 𝑓 (𝑥) < 𝑓𝑁 (𝑥) + .
4(𝑏 − 𝑎) 4(𝑏 − 𝑎)
Поскольку 𝑓𝑁 интегрируема, то она ограничена на [𝑎, 𝑏], значит на [𝑎, 𝑏] ограничена 𝑓 .
Пусть 𝑇 – произвольное разбиение [𝑎, 𝑏]. Тогда для верхних сумм Дарбу имеем:
𝜀
𝑆𝑇 (𝑓 ) = 𝑆𝑇 (𝑓 − 𝑓𝑁 + 𝑓𝑁 ) ⩽ 𝑆𝑇 (𝑓 − 𝑓𝑁 ) + 𝑆𝑇 (𝑓𝑁 ) ⩽ + 𝑆𝑇 (𝑓𝑁 ).
4
(так как sup𝐼 (𝑔(𝑥) + ℎ(𝑥)) ⩽ sup𝐼 𝑔(𝑥) + sup𝐼 ℎ(𝑥) при 𝐼 ⊂ [𝑎, 𝑏])
𝜀
Аналогично (︀для нижних сумм Дарбу
𝜀
)︀ 𝑠𝑇 (𝑓 ) ⩾ 𝑠𝑇 (𝑓𝑁 ) − 4 . Так как 𝑓𝑁 ∈ ℛ[𝑎, 𝑏], то существует 𝑇
– разбиение 𝑆𝑇 (𝑓𝑁 ) − 𝑠𝑇 (𝑓𝑁 ) < 2 , для такого 𝑇 имеем

𝑆𝑇 (𝑓 ) − 𝑠𝑇 (𝑓 ) ⩽ 𝜀.

По критерию Дарбу 𝑓 ∈ ℛ[𝑎, 𝑏]. Для 𝑛 ⩾ 𝑁 имеем


⃒∫︁ 𝑏 ∫︁ 𝑏 ⃒ ∫︁ 𝑏
⃒ ⃒ 𝜀
⃒ 𝑓𝑛 (𝑥) 𝑑𝑥 − 𝑓 (𝑥) 𝑑𝑥⃒⃒ ⩽ |𝑓𝑛 (𝑥) − 𝑓 (𝑥)| 𝑑𝑥 ⩽ (𝑏 − 𝑎) · < 𝜀.

𝑎 𝑎 𝑎 (𝑏 − 𝑎)
∫︀ 𝑏 ∫︀ 𝑏
Следовательно, 𝑎 𝑓𝑛 (𝑥) 𝑑𝑥 → 𝑎 𝑓 (𝑥) 𝑑𝑥.

Следствие 6 (о почленном интегрировании ряда). Если ряд ∞


∑︀
𝑛=1 𝑓𝑛 равномерно сходится на
[𝑎, 𝑏], и все функции 𝑓𝑛 ∈ ℛ[𝑎, 𝑏], то сумма ряда интегрируема на [𝑎, 𝑏] и

∫︁ 𝑏 (︃∑︁ ∞ (︂∫︁ 𝑏
)︃ )︂
∑︁
𝑓𝑛 (𝑥) 𝑑𝑥 = 𝑓𝑛 (𝑥)𝑑𝑥 .
𝑎 𝑛=1 𝑛=1 𝑎

ФПМИ МФТИ, весна 2023 18


Математический анализ Назад к содержанию

Замечание. В условиях теоремы (19)


∫︁ 𝑏 ∫︁ 𝑏
lim 𝑓𝑛 (𝑥) 𝑑𝑥 = lim 𝑓𝑛 (𝑥) 𝑑𝑥.
𝑛→∞ 𝑎 𝑎 𝑛→∞

Теорема 20 (о дифференцируемости предельной функции). Если

1. 𝑓𝑛 → 𝑓 на [𝑎, 𝑏];

2. ∀𝑛 ∈ N функция 𝑓𝑛 : [𝑎, 𝑏] → R дифференцируема;

3. 𝑓𝑛′ ⇒ 𝑔 на [𝑎, 𝑏].

Тогда 𝑓 дифференцируема на [𝑎, 𝑏], причем 𝑓 ′ = 𝑔 на [𝑎, 𝑏].

Доказательство. Докажем дифференцируемость функции 𝑓 . Зафиксируем 𝑥. Рассмотрим по-


следовательность {︃
𝑓𝑛 (𝑡)−𝑓𝑛 (𝑥)
𝑡−𝑥 , 𝑡 ̸= 𝑥;
𝜙𝑛 (𝑡) = ′
𝑓𝑛 (𝑥), 𝑡 = 𝑥.
Тогда 𝜙𝑛 → 𝜙 на [𝑎, 𝑏], где {︃
𝑓 (𝑡)−𝑓 (𝑥)
𝑡−𝑥𝑡 ̸= 𝑥;
,
𝜙(𝑡) =
𝑔(𝑥), 𝑡 = 𝑥.
Покажем, что сходимость равномерная. При 𝑡 ̸= 𝑥 по теореме Лагранжа:

(𝑓𝑛 (𝑡) − 𝑓𝑚 (𝑡)) − (𝑓𝑛 (𝑥) − 𝑓𝑚 (𝑥))


𝜙𝑛 (𝑡) − 𝜙𝑚 (𝑡) = = 𝑓𝑛′ (𝜉) − 𝑓𝑚

(𝜉)
𝑡−𝑥
для некоторой точки 𝜉, лежащей между 𝑡 и 𝑥. Поскольку {𝑓𝑛′ } удовлетворяет условию Коши
равномерной сходимости, то {𝜙𝑛 } удовлетворяет условию Коши. Следовательно, {𝜙𝑛 } равномерно
сходится на [𝑎, 𝑏]. Поскольку 𝑓𝑛 дифференцируема в точке 𝑥, то 𝜙𝑛 непрерывна в точке 𝑥. По
теореме (18) 𝜙 непрерывна в точке 𝑥. Тогда lim𝑡→𝑥 𝜙(𝑡) = 𝜙(𝑥), то есть ∃𝑓 ′ (𝑥) = 𝑔(𝑥).

Следствие 7 (о почленном дифференцировании ряда). Пусть


∑︀∞
1. 𝑛=1 𝑓𝑛 сходится поточечно на [𝑎, 𝑏];

2. 𝑓𝑛 : [𝑎, 𝑏] → R дифференцируема ∀𝑛;


∑︀∞ ′
3. 𝑛=1 𝑓𝑛 равномерно сходится на [𝑎, 𝑏].

Тогда сумма ряда дифференцируема и для каждой точки 𝑥 ∈ [𝑎, 𝑏] выполнено


(︃ ∞
)︃′ ∞
∑︁ ∑︁
𝑓𝑛 (𝑥) = 𝑓𝑛′ (𝑥).
𝑛=1 𝑛=1

Теорема 21 (признак Вейерштрасса). Пусть 𝑓𝑛 : 𝐸 → C, 𝑎𝑛 ∈ R ∀𝑛. Пусть

1. ∀𝑛 ∈ N ∀𝑥 ∈ 𝐸 (|𝑓𝑛 (𝑥)| ⩽ 𝑎𝑛 );

2. числовой ряд ∞
∑︀
𝑛=1 𝑎𝑛 сходится.

Тогда функциональный ряд ∞


∑︀
𝑛=1 𝑓𝑛 сходится равномерно и абсолютно на 𝐸.

ФПМИ МФТИ, весна 2023 19


Математический анализ Назад к содержанию

Доказательство. Пусть 𝜀 > 0. Пользуясь критерием Коши как необходимым условием, найдем
𝑁 , что 𝑛𝑘=𝑚 𝑎𝑘 < 𝜀 при всех 𝑛 ⩾ 𝑚 ⩾ 𝑁 . Тогда для таких 𝑛, 𝑚 и всех 𝑥 ∈ 𝐸 справедлива оценка:
∑︀

⃒ ⃒
𝑛
⃒ ∑︁ ⃒ ∑︁𝑛 𝑛
∑︁
𝑓𝑘 (𝑥)⃒ ⩽ |𝑓𝑘 (𝑥)| ⩽ 𝑎𝑘 < 𝜀.
⃒ ⃒

⃒ ⃒
𝑘=𝑚 𝑘=𝑚 𝑘=𝑚
∑︀∞ ∑︀∞
Пользуясь теперь критерием Коши как достаточным условием, получаем, что 𝑛=1 𝑓𝑛 и 𝑛=1 |𝑓𝑛 |
равномерно сходятся на 𝐸.

Определение 15. Последовательность 𝑔𝑛 называется равномерно ограниченной на 𝐸, если най-


дется такое 𝐶 > 0, что |𝑔𝑛 (𝑥)| ⩽ 𝐶 для всех 𝑛 ∈ N и 𝑥 ∈ 𝐸.

Теорема 22 (признак Дирихле). Пусть 𝑎𝑛 : 𝐸 → R (или C), 𝑏𝑛 : 𝐸 → R ∀𝑛 такие, что:

1. 𝐴𝑁 = 𝑁
∑︀
𝑛=1 𝑎𝑛 равномерно ограничена на E;

2. {𝑏𝑛 (𝑥)} монотонна при каждом 𝑥 ∈ 𝐸;

3. 𝑏𝑛 ⇒ 0 на 𝐸.

Тогда ∞
∑︀
𝑛=1 𝑎𝑛 𝑏𝑛 равномерно сходится на 𝐸.

Доказательство. Зафиксируем 𝜀 > 0. Отметим, что при 𝑛 ⩾ 𝑚


⃒ ⃒
𝑛
⃒ ∑︁ ⃒
𝑎𝑘 (𝑥)⃒ = |𝐴𝑛 (𝑥) − 𝐴𝑚−1 (𝑥)| ⩽ 2𝐶
⃒ ⃒

⃒ ⃒
𝑘=𝑚

для всех 𝑥 ∈ 𝐸.
Из равномерной сходимости {𝑏𝑛 } следует, что
(︁ 𝜀 )︁
∃𝑁 ∀𝑛 ⩾ 𝑁 ∀𝑥 ∈ 𝐸 |𝑏𝑛 (𝑥)| < .
8𝐶
Тогда при 𝑛 ⩾ 𝑚 ⩾ 𝑁 и 𝑥 ∈ 𝐸 по лемме Абеля
⃒ ⃒
𝑛
⃒ ∑︁ ⃒
𝑎𝑘 (𝑥)𝑏𝑘 (𝑥)⃒ ⩽ 2 · 2𝐶 (|𝑏𝑚 (𝑥)| + |𝑏𝑛 (𝑥)|) < 𝜀.
⃒ ⃒

⃒ ⃒
𝑘=𝑚
∑︀∞
По критерию Коши 𝑛=1 𝑎𝑛 𝑏𝑛 сходится равномерно на 𝐸.

Теорема 23 (признак Абеля). Пусть 𝑎𝑛 : 𝐸 → R (или C), 𝑏𝑛 : 𝐸 → R, такие, что


∑︀∞
1. 𝑛=1 𝑎𝑛 равномерно сходится на 𝐸;

2. {𝑏𝑛 (𝑥)} монотонна при любом 𝑥 ∈ 𝐸;

3. {𝑏𝑛 } равномерно ограничена на 𝐸.

Тогда ∞
∑︀
𝑛=1 𝑎𝑛 𝑏𝑛 равномерно сходится на 𝐸.

Доказательство. Из равномерной сходимости ряда


(︃⃒ 𝑛 ⃒ )︃
⃒ ∑︁ ⃒ 𝜀
∃𝑁 ∀𝑛, 𝑚 (𝑛 ⩾ 𝑚 ⩾ 𝑁 ) ∀𝑥 ∈ 𝐸 ⃒ 𝑎𝑘 (𝑥)⃒ < .
⃒ ⃒
⃒ ⃒ 4𝐶
𝑘=𝑚

ФПМИ МФТИ, весна 2023 20


Математический анализ Назад к содержанию

Тогда при всех 𝑥 ∈ 𝐸 и 𝑛 ⩾ 𝑚 ⩾ 𝑁 по лемме Абеля


⃒ ⃒
𝑛
⃒ ∑︁ ⃒ 𝜀 𝜀
𝑎𝑘 (𝑥)𝑏𝑘 (𝑥)⃒ ⩽ 2 · (|𝑏𝑚 (𝑥)| + |𝑏𝑛 (𝑥)|) ⩽ 2 · (𝐶 + 𝐶) = 𝜀.
⃒ ⃒
4𝐶 4𝐶

⃒ ⃒
𝑘=𝑚

Доказательство завершается ссылкой на критерий Коши (17).

Пример (ван-дер-Варден). Существует 𝑓 : R → R, непрерывная на R, но не дифференцируемая


ни в одной точке.
Пусть 𝜙 : R → R, 𝜙(𝑥 ± 2) = 𝜙(𝑥), 𝜙|[−1,1] (𝑥) = |𝑥|. Отметим, что если (𝑥, 𝑦) ∩ Z = ∅, то 𝜙
кусочно-линейная с угловым коэффициентом ±1, поэтому

|𝜙(𝑥) − 𝜙(𝑦)| = |𝑥 − 𝑦|.

Положим 𝑓 (𝑥) = ∞ 1 𝑛
∑︀
𝑛=1 𝑓𝑛 (𝑥), 𝑓𝑛 (𝑥) = 4𝑛 𝜙(4 𝑥). Функция 𝑓 непрерывна как сумма равномерно
сходящегося ряда (по признаку Вейерштрасса) из непрерывных функций, но не дифференцирума
ни в одной точке.

ФПМИ МФТИ, весна 2023 21


Математический анализ Назад к содержанию

4. Степенные ряды. Теорема Коши-Адамара. Радиус и круг сходи-


мости, равномерная сходимость степенных рядов. Теорема Абеля.
Дифференцируемость суммы степенного ряда. Теорема единствен-
ности, ряд Тейлора. Пример бесконечно дифференцируемой функ-
ции, не разлагающейся в степенной ряд. Достаточное условие раз-
ложимости функции в степенной ряд. Ряды Тейлора 𝑒𝑥 , sin 𝑥, cos 𝑥,
(1 + 𝑥)𝛼 , ln(1 + 𝑥).
Определение 16. Степенным рядом называется функциональный ряд вида

∑︁
𝑎𝑛 (𝑥 − 𝑥0 )𝑛 ,
𝑛=0

где 𝑎𝑛 , 𝑥0 ∈ R и 𝑥 — действительная переменная, или 𝑎𝑛 , 𝑥0 ∈ C и 𝑥 — комплексная переменная


(комплексный степенной ряд).
1 √
Теорема 24 (Коши-Адамар). Пусть 𝑅 = 𝑛
.
lim𝑛→∞ |𝑎𝑛 |
Тогда:
1. при |𝑥 − 𝑥0 | < 𝑅 ряд (16) сходится, причём абсолютно;

2. при |𝑥 − 𝑥0 | > 𝑅 ряд (16) расходится;

3. если 𝑟 ∈ (0, 𝑅), то ряд (16) равномерно сходится на 𝐵𝑟 (𝑥0 ) = {𝑥 : |𝑥 − 𝑥0 | ⩽ 𝑟}.


Доказательство. Пусть 𝑥 ̸= 𝑥0 , тогда
√︀
𝑛
√︀ |𝑥 − 𝑥0 |
𝑞 := lim |𝑎𝑛 (𝑥 − 𝑥0 )𝑛 | = |𝑥 − 𝑥0 | lim 𝑛 |𝑎𝑛 | = .
𝑛→∞ 𝑛→∞ 𝑅
Если |𝑥 − 𝑥0 | < 𝑅, то 𝑞 < 1 и, значит, по признаку Коши ∞ 𝑛
∑︀
𝑛=0 |𝑎𝑛 (𝑥 − 𝑥0 ) | сходится, то есть,
ряд (16) сходится абсолютно.
Если |𝑥 − 𝑥0 | > 𝑅, то 𝑞 > 1 и, значит, по признаку Коши 𝑛-й член ряда не стремится к нулю,
ряд (16) расходится и абсолютно расходится (то есть, расходится ряд из модулей членов).
Пусть 𝑟 ∈ ∑︀
(0, 𝑅). По доказанному ряд (16) абсолютно сходится в точке 𝑥 = 𝑥0 + 𝑟, то есть
сходится ряд ∞ 𝑛 𝑛 𝑛
𝑛=0 |𝑎𝑛 |𝑟 . Если |𝑥 − 𝑥0 | ⩽ 𝑟, то |𝑎𝑛 (𝑥 − 𝑥0 ) | ⩽ |𝑎𝑛 |𝑟 . Тогда по признаку Вейер-
штрасса ряд (16) равномерно сходится на 𝐵𝑟 (𝑥0 ).

Определение 17. Величина 𝑅 из теоремы (24) называется радиусом сходимости ряда (16).
𝐵𝑅 (𝑥0 ) = {𝑥 : |𝑥 − 𝑥0 | < 𝑅} называется интервалом сходимости (кругом сходимости в
комлексной плоскости).
Из теоремы (24) получаем:
Следствие 8. Пусть для 𝑅 ∈ [0, +∞] выполнено следующее: при |𝑥 − 𝑥0 | < 𝑅 ряд абсолютно
сходится и при |𝑥 − 𝑥0 | > 𝑅 ряд абсолютно расходится, то 𝑅 — радиус сходимости.
Теорема 25 (Абель). Если степенной ряд (16) сходится в точке 𝑥1 ̸= 𝑥0 , то он сходится
равномерно на отрезке с концами 𝑥1 , 𝑥0 .
Доказательство. По условию ряд ∞ 𝑛
∑︀
𝑛=0 𝑎𝑛 (𝑥1 − 𝑥0 ) сходится. Рассмотрим последовательность
{𝑡 𝑛
∑︀∞} : она монотонна при любом 𝑡 ∈ [0, 1] и равномерно ограниченна. По признаку Абеля (23) ряд
𝑛 𝑡𝑛 равномерно сходится на [0, 1]. Сделав замену 𝑡 = 𝑥−𝑥0 , получим, что ряд
𝑎 (𝑥
𝑛=0 𝑛 1 − 𝑥 0 ) 𝑥1 −𝑥0
(16) равномерно сходится на {𝑥 : 𝑥 = 𝑥0 + 𝑡(𝑥1 − 𝑥0 )}.

ФПМИ МФТИ, весна 2023 22


Математический анализ Назад к содержанию

Замечание. Если 𝑥1 ∈ 𝐵𝑅 (𝑥0 ), то предыдущая теорема вытекает из теоремы Коши–Адамара


(24), поэтому интерес представляет случай, когда 𝑥1 лежит на границе круга сходимости.
∑︀+∞ 𝑛−1 также
Лемма 13. Если ряд (16) имеет радиус сходимости 𝑅, то ряд 𝑛=1 𝑛𝑎𝑛 (𝑥 − 𝑥0 )
имеет радиус сходимости 𝑅.
√ √︀ √︀
Доказательство. Так как lim𝑛→+∞ 𝑛 𝑛 = 1, то последовательности √︀ { 𝑛 |𝑎𝑛 |} и√︀{ 𝑛 𝑛|𝑎𝑛 |} имеют
𝑛 𝑛
одинаковое множество частичных пределов, ∑︀+∞|𝑎𝑛 | и lim𝑛→+∞
∑︀+∞ значит lim𝑛𝑛→+∞ 𝑛
𝑛|𝑎𝑛 | равны. То-
гда по формуле Коши–Адамара ряды 𝑛=0 𝑎𝑛 (𝑥 − 𝑥0 ) и 𝑛=1 𝑛𝑎𝑛 (𝑥 − 𝑥0 ) имеют одинаковые
радиусы ∑︀сходимости.
Ряды +∞ 𝑛−1 и
∑︀+∞ 𝑛
𝑛=1 𝑛𝑎𝑛 (𝑥−𝑥0 ) 𝑛=1 𝑛𝑎𝑛 (𝑥−𝑥0 ) отличаются при 𝑥 ̸= 𝑥0 ненулевым множителем
(при 𝑥 = 𝑥0 оба сходятся). Следовательно, эти ряды сходятся одновременно. Тогда, радиусы
сходимости этих рядов также совпадают.

Теорема 26. Если 𝑓 (𝑥) = +∞ 𝑛


∑︀
𝑛=0 𝑎𝑛 (𝑥 − 𝑥0 ) – сумма степенного ряда с радиусом сходимости
𝑅 > 0, то функция 𝑓 бесконечно дифференцируема в 𝐵𝑅 (𝑥0 ), и для всякого 𝑚 ∈ N выполнено:
+∞
∑︁
(𝑚)
𝑓 (𝑥) = 𝑛(𝑛 − 1) · . . . · (𝑛 − 𝑚 + 1)𝑎𝑛 (𝑥 − 𝑥0 )𝑛−𝑚 .
𝑛=𝑚

Доказательство. По лемме (13) при дифференцировании радиус сходимости ряда не меняется,


поэтому нам достаточно доказать утверждение для 𝑚 =∑︀1, после чего применить индукцию.
Пусть 0 < 𝑟 < 𝑅. По теореме (24) исходный ряд и ряд ∞ 𝑛=1 𝑛𝑎𝑛 (𝑥−𝑥0 )
𝑛−1 равномерно сходятся

на отрезке [𝑥0 − 𝑟, 𝑥0 + 𝑟]. Обозначим через 𝑔 сумму продифференцированного ряда. Тогда по


следствию о почленном дифференцировании ряда из теоремы (20) функция 𝑓 дифференцируема
на [𝑥0 − 𝑟, 𝑥0 + 𝑟], причем 𝑓 ′ = 𝑔. Так как 𝑟 ∈ (0, 𝑅) – любое, то равенство выполняется на
(𝑥0 − 𝑅, 𝑥0 + 𝑅).

Следствие 9 (теорема о единственности). Если степенные ряды +∞


∑︀ 𝑛
∑︀+∞
𝑛=0 𝑎𝑛 (𝑥 − 𝑥0 ) и 𝑛=0 𝑏𝑛 (𝑥 −
𝑛
𝑥0 ) сходятся в круге 𝐵𝛿 (𝑥0 ), и их суммы там совпадают, то 𝑎𝑛 = 𝑏𝑛 , 𝑛 = 0, 1, 2, . . .

Следствие ∑︀10. Сумма степенного ряда с радиусом сходимости 𝑅 > 0 имеет первообразную
𝐹 (𝑥) = 𝐶 + +∞ 𝑎𝑛
𝑛=0 𝑛+1 (𝑥 − 𝑥0 )
𝑛+1 при |𝑥 − 𝑥 | < 𝑅.
0

Определение 18. Пусть функция 𝑓 определена в некоторой окрестности точки 𝑥0 и в точке


𝑓 (𝑛) (𝑥0 )
𝑥0 имеет производные любого порядка. Тогда +∞ (𝑥 − 𝑥0 )𝑛 называется рядом Тейлора
∑︀
𝑛=0 𝑛!
функции 𝑓 с центром в точке 𝑥0 . Для 𝑥0 = 0 ряд называют рядом Маклорена.

Пример. Рассмотрим 𝑓 : R → R,
{︃
0, 𝑥 ⩽ 0;
𝑓 (𝑥) = 1
𝑒− 𝑥 , 𝑥 > 0.

Существование производных любого порядка в точке 𝑥 ̸= 0 следует из теоремы о диффе-


1
ренцировании композиции. Более того, 𝑓 (𝑛) (𝑥) = 0 при 𝑥 < 0 и 𝑓 (𝑛) (𝑥) = 𝑝𝑛 ( 𝑥1 )𝑒− 𝑥 , где 𝑝𝑛 (𝑡)
– многочлен степени 2𝑛. последнее утверждение можно установить по индукции: 𝑝0 (𝑡) = 1 и
дифференцирование 𝑓 (𝑛) дает соотношение 𝑝𝑛+1 (𝑡) = 𝑡2 [𝑝𝑛 (𝑡) − 𝑝′𝑛 (𝑡)].
Индукцией по 𝑛 покажем, что 𝑓 (𝑛) (0) = 0. Для 𝑛 = 0 это верно по условию. Если предполо-
жить, что 𝑓 (𝑛) (0) = 0, то (𝑓 (𝑛) )′− (0) = 0 и
1
𝑓 (𝑛) (ℎ) − 𝑓 (𝑛) (0) 𝑝𝑛 ( ℎ1 )𝑒− ℎ 𝑡𝑝𝑛 (𝑡)
(𝑓 (𝑛) )′+ (0) = lim = lim = lim = 0,
ℎ→+0 ℎ ℎ→+0 ℎ 𝑡→+∞ 𝑒𝑡

ФПМИ МФТИ, весна 2023 23


Математический анализ Назад к содержанию

𝑚
поскольку по правилу Лопиталя lim𝑡→+∞ 𝑡𝑒𝑡 = 0 для всех 𝑚 ∈ N0 . Это доказывает, что 𝑓 (𝑛+1) (0) =
0.
Таким образом, ряд Маклорена функции 𝑓 нулевой, но он не сходится к 𝑓 ни в какой окрест-
ности нуля.

Лемма 14 (Достаточное условие представимости функции степенным рядом.). Если на (𝑥0 −


𝜌, 𝑥0 + 𝜌) функция 𝑓 имеет производные всех порядков и
(︂⃒ )︂
⃒ (𝑛) ⃒ 𝐶𝑛!

∃𝐶 > 0 ∀𝑛 ∈ N0 ∀𝑥 ∈ (𝑥0 − 𝜌, 𝑥0 + 𝜌) ⃒𝑓 (𝑥)⃒ ⩽ 𝑛 ,
𝜌
то
+∞ (𝑛)
∑︁ 𝑓 (𝑥0 )
𝑓 (𝑥) = (𝑥 − 𝑥0 )𝑛
𝑛!
𝑛=0

для всех 𝑥 ∈ (𝑥0 − 𝜌, 𝑥0 + 𝜌).


√︁ 1
𝑛 𝑓 (𝑛) (𝑥) 𝐶𝑛 1
Доказательство. Так как 𝑛! ⩽ 𝜌 → 𝜌 , то по формуле Коши-Адамара (24) для 𝑥 ∈
(𝑥0 − 𝜌, 𝑥0 + 𝜌) найдется 𝑐 между 𝑥0 и 𝑥, что
⃒ ⃒ ⃒ ⃒
𝑛
⃒ ∑︁ 𝑓 (𝑘) (𝑥0 ) ⃒ ⃒ 𝑓 (𝑛+1) (𝑐) ⃒
⃒𝑓 (𝑥) − (𝑥 − 𝑥0 )𝑘 ⃒ = ⃒ (𝑥 − 𝑥0 )𝑛+1 ⃒ .
⃒ ⃒ ⃒ ⃒
⃒ 𝑘! ⃒ ⃒ (𝑛 + 1)! ⃒
𝑘=0

Поскольку |𝑓 (𝑛+1) (𝑐)| ⩽ 𝐶, то справедлива оценка:


⃒ ⃒
⃒ 𝑓 (𝑛+1) (𝑐) ⃒ 𝑥 − 𝑥0 ⃒𝑛+1
⃒ ⃒ ⃒
(𝑛+1) ⃒
(𝑥 − 𝑥0 ) ⃒ ⩽ 𝐶 ⃒⃒ → 0,
⃒ ⃒
⃒ (𝑛 + 1)! 𝜌 ⃒

что завершает доказательство.

Следствие 11. Ряды Маклорена функций exp, sin, cos сходятся на R к самим функциям, то есть
∀𝑥 ∈ R :
+∞ 𝑛
𝑥
∑︁ 𝑥
𝑒 = ,
𝑛!
𝑛=0
+∞
∑︁ (−1)𝑛 2𝑛+1
sin(𝑥) = 𝑥 ,
(2𝑛 + 1)!
𝑛=0
+∞
∑︁ (−1)𝑛
cos(𝑥) = 𝑥2𝑛 .
(2𝑛)!
𝑛=0

Доказательство. Все указанные функции бесконечно дифференцируемы на R, причем (𝑒𝑥 )(𝑛) =


𝑒𝑥 , sin(𝑛) (𝑥) = sin(𝑥 + 𝜋𝑛
2 ), cos
(𝑛) (𝑥) = cos(𝑥 + 𝜋𝑛 ).
2
Пусть 𝛿 > 0 и |𝑥| < 𝛿. Тогда (𝑒𝑥 )(𝑛) ⩽ 𝑒𝛿 , | sin(𝑛) (𝑥)| ⩽ 1, | cos(𝑛) (𝑥)| ⩽ 1.
Следовательно, по следствию 10 ряды Маклорена этих функций сходятся к самим функциям
на (−𝛿, 𝛿). Так как 𝛿 > 0 – любое, то предположение верно и на R.
1
= +∞ 𝑛−1 𝑥𝑛−1 при |𝑥| < 1, то по (10)
∑︀
Пример. Так как 1+𝑥 𝑛=1 (−1)

+∞
∑︁ (−1)𝑛−1 𝑥𝑛
ln(1 + 𝑥) = , |𝑥| < 1.
𝑛
𝑛=1

ФПМИ МФТИ, весна 2023 24


Математический анализ Назад к содержанию

Ряд в правой части сходится при 𝑥 = 1, поэтому его сумма непрерывна на (−1, 1] и, значит,
(−1)𝑛−1
равенство имеет место при 𝑥 = 1. Получаем известный нам результат, что +∞
∑︀
𝑛=1 𝑛 = ln 2.
𝛼·(𝛼−1)·...·(𝛼−𝑛+1)
Теорема 27 (биномиальный ряд). Если 𝛼 ̸∈ N0 и 𝐶𝛼𝑛 = 𝑛! , 𝐶𝛼0 = 1, то
+∞
∑︁
(1 + 𝑥)𝛼 = 𝐶𝛼𝑛 𝑥𝑛 , |𝑥| < 1.
𝑛=0

Доказательство. Пусть 𝑓 (𝑥) = (1 + 𝑥)𝛼 . Тогда 𝑓 (𝑛) (𝑥) = 𝛼 · (𝛼 − 1) · . . . · (𝛼 − 𝑛 + 1) · (1 + 𝑥)𝛼−1 и,


𝑓 (𝑛) (0)
значит, 𝑛! = 𝐶𝛼𝑛 . При 𝑥 ̸= 0:

|𝐶𝛼𝑛+1 𝑥𝑛+1 | |𝛼 − 𝑛||𝑥|


lim 𝑛 𝑛
= lim = |𝑥|.
𝑛→+∞ |𝐶𝛼 𝑥 | 𝑛→+∞ 𝑛+1

Если |𝑥| < 1, то ряд абсолютно сходится по признаку Даламбера. Если |𝑥| > 1, то ряд абсо-
лютно расходится по признаку Даламбера. Следовательно, 𝑅сх = 1.
Обозначим 𝑔(𝑥) = +∞ 𝑛 𝑥𝑛 , и покажем, что 𝑔 ≡ 𝑓 на (−1, 1), т.е. (1 + 𝑥)−𝛼 𝑔(𝑥) = 1 при
∑︀
𝐶
𝑛=0 𝛼
𝑥 ∈ (−1, 1). Для этого найдем производную функции (1 + 𝑥)−𝛼 𝑔(𝑥). По теореме (26) имеем
+∞
∑︁ +∞
∑︁
((1 + 𝑥)−𝛼 𝑔(𝑥))′ = (1 + 𝑥)−𝛼 𝑛𝐶𝛼𝑛 𝑥𝑛−1 − 𝛼(1 + 𝑥)−𝛼−1 𝐶𝛼𝑛 𝑥𝑛 =
𝑛=1 𝑛=0
[︃ +∞ +∞ +∞
]︃
∑︁ ∑︁ ∑︁
= (1 + 𝑥)−𝛼−1 𝑛𝐶𝛼𝑛 𝑥𝑛−1 + 𝑛𝐶𝛼𝑛 𝑥𝑛 − 𝛼 𝐶𝛼𝑛 𝑥𝑛 .
𝑛=1 𝑛=0 𝑛=0

В первой сумме произведем замену индекса суммирования. После приведения подобных сла-
гаемых получим
[︃ +∞ +∞
]︃
∑︁ ∑︁
((1 + 𝑥)−𝛼 𝑔(𝑥))′ = (1 + 𝑥)−𝛼−1 (𝑛 + 1)𝐶𝛼𝑛+1 𝑥𝑛 − (𝛼 − 𝑛)𝐶𝛼𝑛 𝑥𝑛 = 0.
𝑛=0 𝑛=0

Отсюда следует, что (1 + 𝑥)−𝛼 𝑔(𝑥)


постоянна на (−1, 1). Из условия 𝑔(0) = 1 получаем, что
−𝛼
(1 + 𝑥) 𝑔(𝑥) = 1 для всех 𝑥 ∈ (−1, 1).

ФПМИ МФТИ, весна 2023 25


Математический анализ Назад к содержанию

5. Метрические и нормированные пространства, 𝑝-нормы на R𝑛 .


Топология метрических пространств: открытые и замкнутые мно-
жества, их свойства. Предельные точки. Критерии замкнутости
множества. Замыкание множества. Подпространства метрическо-
го пространства, описание открытых множеств подпространства.
Компакты и их свойства. Теорема о секвенциальной компактности.
Описание компактов в R𝑛 . Теорема Больцано-Вейерштрасса. Пол-
ные метрические пространства. Полнота пространств R𝑛 и 𝐵(𝐸).
Определение 19. Пусть 𝑋 ̸= ∅. Функция 𝜌 : 𝑋 × 𝑋 → R называется метрикой на 𝑋, если для
любых 𝑥, 𝑦, 𝑧 ∈ 𝑋 выполнено:

1. 𝜌(𝑥, 𝑦) ⩾ 0, 𝜌(𝑥, 𝑦) = 0 ⇔ 𝑥 = 𝑦;
2. 𝜌(𝑥, 𝑦) = 𝜌(𝑦, 𝑥);
3. 𝜌(𝑥, 𝑦) ⩽ 𝜌(𝑥, 𝑧) + 𝜌(𝑧, 𝑦) (неравенство треугольника).

Пара (𝑋, 𝜌) называется метрическим пространством.


В дальнейшем часто под метрическим пространтвом будем понимать само множество 𝑋, пред-
полагая наличие связанной с ним метрики.
Определение 20. Пусть 𝑉 – линейное пространство. Функция ‖ · ‖ : 𝑉 → R называется нормой,
если для любых 𝑥, 𝑦 ∈ 𝑉 и 𝛼 ∈ R выполнено:

1. ‖𝑥‖ ⩾ 0, ‖𝑥‖ = 0 ⇔ 𝑥 = 0;
2. ‖𝛼𝑥‖ = |𝛼|‖𝑥‖;
3. ‖𝑥 + 𝑦‖ ⩽ ‖𝑥‖ + ‖𝑦‖ (неравенство треугольника).

Пара (𝑉, ‖ · ‖) называется нормированным пространством.


Пример. 𝑋 = R𝑛 , 𝑥 = (𝑥1 , . . . , 𝑥𝑛 )𝑇 , 𝑦 = (𝑦1 , . . . , 𝑦𝑛 )𝑇 .
√︀∑︀𝑛 √︀∑︀𝑛
1. |𝑥| = |𝑥 |2 , 𝜌 (𝑥, 𝑦) = 2
𝑖=1 𝑖 2 𝑖=1 |𝑥𝑖 − 𝑦𝑖 | .
1 1
2. ‖𝑥‖𝑝 = ( 𝑛𝑖=1 |𝑥𝑖 |𝑝 ) 𝑝 , 𝜌𝑝 (𝑥, 𝑦) = ( 𝑛𝑖=1 |𝑥𝑖 − 𝑦𝑖 |𝑝 ) 𝑝 , 𝑝 ⩾ 1.
∑︀ ∑︀

3. ‖𝑥‖∞ = max1⩽𝑖⩽𝑛 |𝑥𝑖 |, 𝜌∞ (𝑥, 𝑦) = max1⩽𝑖⩽𝑛 |𝑥𝑖 − 𝑦𝑖 |.

Доказательство. Покажем, что ‖ · ‖𝑝 — норма на R𝑛 .


Проверим сначала, что если ‖𝑥‖𝑝 ⩽ 1, ‖𝑦‖𝑝 ⩽ 1, 𝛼 + 𝛽 = 1, 𝛼 ⩾ 0, 𝛽 ⩾ 0, то ‖𝛼𝑥 + 𝛽𝑦‖𝑝 ⩽ 1.
Функция 𝜙(𝑠) = 𝑠𝑝 — выпуклая на [0, +∞), следовательно |𝛼𝑥𝑖 + 𝛽𝑦𝑖 |𝑝 ⩽ 𝛼|𝑥𝑖 |𝑝 + 𝛽|𝑥𝑖 |𝑝 .
Просуммируем по 𝑖 = 1, . . . , 𝑛. ‖𝛼𝑥 + 𝛽𝑦‖𝑝𝑝 ⩽ 𝛼‖𝑥‖𝑝𝑝 + 𝛽‖𝑦‖𝑝𝑝 ⩽ 𝛼 + 𝛽 = 1.
Пусть 𝑥, 𝑦 произвольны. Если 𝑥 = 0 или 𝑦 = 0, то неравенство выполняется. Будем предпола-
гать, что 𝑥 ̸= 0 и 𝑦 ̸= 0. Покажем, что ‖𝑥 + 𝑦‖𝑝 ⩽ ‖𝑥‖𝑝 + ‖𝑦‖𝑝 (индекс 𝑝 будем опускать)
‖𝑥‖ ‖𝑦‖ 𝑥 𝑦
Введём обозначения 𝛼 = ‖𝑥‖+‖𝑦‖ , 𝛽 = ‖𝑥‖+‖𝑦‖ ,𝑥
ˆ = ‖𝑥‖ , 𝑦ˆ = ‖𝑦‖ . Тогда, учитывая, что ‖𝛼ˆ𝑥+
𝛽 𝑦ˆ‖ ⩽ 1, имеем
⃦ ⃦
⃦ 𝑥 𝑦 ⃦
‖𝑥 + 𝑦‖ = (‖𝑥‖ + ‖𝑦‖) ⃦
⃦ + ⃦ = (‖𝑥‖ + ‖𝑦‖) ‖𝛼ˆ 𝑥 + 𝛽 𝑦ˆ‖ ⩽ ‖𝑥‖ + ‖𝑦‖.
‖𝑥‖ + ‖𝑦‖ ‖𝑥‖ + ‖𝑦‖ ⃦

Проверка, что ‖ · ‖∞ является нормой, легко следует из свойств модуля числа.

ФПМИ МФТИ, весна 2023 26


Математический анализ Назад к содержанию

Определение 21. Пусть (𝑋, 𝜌) — метрическое пространство, 𝑥 ∈ 𝑋.


Множество 𝐵𝑟 (𝑥) = {𝑦 ∈ 𝑋 | 𝜌(𝑦, 𝑥) < 𝑟}, 𝑟 > 0 называется открытым шаром с центром в
точке 𝑥 и радиуса 𝑟.
Множество 𝐵 𝑟 (𝑥) = {𝑦 ∈ 𝑋 | 𝜌(𝑦, 𝑥) ⩽ 𝑟} называется замкнутым шаром с центром в точке 𝑥
и радиуса 𝑟.

Определение 22. Пусть (𝑋, 𝜌) — метрическое пространство, 𝐸 ⊂ 𝑋 и 𝑥 ∈ 𝑋.

1. Точка 𝑥 называется внутренней точкой множества 𝐸, если ∃𝜀 > 0 (𝐵𝜀 (𝑥) ⊂ 𝐸).

2. Точка 𝑥 называется внешней точкой множества 𝐸, если ∃𝜀 > 0 (𝐵𝜀 (𝑥) ⊂ 𝑋 ∖ 𝐸).
Обозначим ext 𝐸 — множество внешних точек 𝐸. Очевидно, ext 𝐸 = int(𝑋 ∖ 𝐸).

3. Точка 𝑥 называется граничной точкой множества 𝐸, если


{︂
𝐵𝜀 (𝑥) ∩ 𝐸 ̸= ∅
∀𝜀 > 0
𝐵𝜀 (𝑥) ∩ (𝑋 ∖ 𝐸) ̸= ∅

Обозначим 𝜕𝐸 — множество граничных точек 𝐸.

Замечание. 𝑋 = int 𝐸 ∪ ext 𝐸 ∪ 𝜕𝐸, причём int 𝐸, ext 𝐸, 𝜕𝐸 попарно не пересекаются.

Определение 23. Множество 𝐺 ⊂ 𝑋 называется открытым, если все точки 𝐺 являются внут-
ренними (то есть 𝐺 = int 𝐺).
Множество 𝐹 ⊂ 𝑋 называется замкнутым, если 𝑋 ∖ 𝐹 открыто.

Аналогично случаю 𝑋 = R доказывается следующая лемма.

Лемма 15. Объединение произвольного семейства открытых множеств и пересечение конеч-


ного семейства открытых множеств являются открытыми множествами.
Объединение конечного семейства замкнутых множеств и пересечение произвольного се-
мейства замкнутых множеств являются замкнутыми множествами.
˚𝜀 (𝑥)∩𝐸 ̸=
Определение 24. Точка 𝑥 называется предельной точкой множества 𝐸, если ∀𝜀 > 0 (𝐵
˚
∅). Здесь и далее 𝐵𝜀 (𝑥) = 𝐵𝜀 (𝑥) ∖ {𝑥}.

Определение 25. Точка 𝑥 называется изолированной точкой множества 𝐸, если 𝑥 ∈ 𝐸 и 𝑥 не


предельная.

Теорема 28. Следующие утверждения эквивалентны:

1. 𝐸 замкнуто;

2. 𝐸 содержит все свои граничные точки;

3. 𝐸 содержит все свои предельные точки;

Доказательство.
(1 ⇒ 2) 𝑥 ∈ 𝑋 ∖ 𝐸 ⇒ ∃𝐵𝜀 (𝑥) ⊂ 𝑋 ∖ 𝐸 ⇒ 𝑥 ̸= 𝜕𝐸 ⇒ 𝜕𝐸 ⊂ 𝐸.
⏟ ⏞
откр.
(2 ⇒ 3) Любая предельная точка является внутренней или граничной, значит 𝐸 содержит все
предельные точки.
˚𝜀 (𝑥) ∩ 𝐸 =
(3 ⇒ 1) Пусть 𝑥 ∈ 𝑋 ∖ 𝐸. Точка 𝑥 не является предельной для 𝐸, т.е. ∃𝜀 > 0 (𝐵
∅) ⇒ 𝐵𝜀 (𝑥) ⊂ 𝑋 ∖ 𝐸. Значит, 𝑋 ∖ 𝐸 открыто.

ФПМИ МФТИ, весна 2023 27


Математический анализ Назад к содержанию

Определение 26. 𝐸 = 𝐸 ∪ 𝜕𝐸 — замыкание 𝐸.

Замечание. 𝑥 ∈ 𝐸 ⇔ ∃{𝑥𝑛 } ⊂ 𝐸 (𝑥𝑛 → 𝑥).

Доказательство. Если 𝑥 ∈ 𝐸 ∪ 𝜕𝐸, то ∀𝜀 > 0 (𝐵𝜀 (𝑥) ∩ 𝐸 ̸= ∅). Выберем точку 𝑥𝑛 ∈ 𝐵 1 (𝑥) ∩ 𝐸.
𝑛
Так как 𝜌(𝑥𝑛 , 𝑥) < 𝑛1 , то 𝑥𝑛 → 𝑥.
Обратно, если 𝑥 ∈ 𝑋 ∖ 𝐸, то 𝑥 – внешнаяя точка 𝐸 и, значит, 𝑥 не может быть пределом
последовательности точек из 𝐸.

Следствие 12. Множество 𝐸 замкнуто ⇔ ∀{𝑥𝑛 }, 𝑥𝑛 ∈ 𝐸 (𝑥𝑛 → 𝑥 ⇒ 𝑥 ∈ 𝐸).

Определение 27. Пусть (𝑋, 𝜌) — метрическое пространство, 𝐸 ⊂ 𝑋, 𝐸 ̸= ∅. Сужение 𝜌|𝐸×𝐸


является метрикой на 𝐸. Пара (𝐸, 𝜌|𝐸×𝐸 ) называется подпространством (𝑋, 𝜌), а функция 𝜌|𝐸×𝐸
— индуцированной метрикой.

Рассмотрим 𝐵𝑟𝐸 (𝑥) = {𝑦 ∈ 𝐸 | 𝜌(𝑥, 𝑦) < 𝑟} = 𝐵𝑟𝑋 (𝑥) ∩ 𝐸.

Лемма 16. Пусть (𝑋, 𝜌) — метрическое пространство, 𝐸 ⊂ 𝑋.

⏟ 𝑈⏞ ⇔∃ ⏟ 𝑉⏞ (𝑈 = 𝑉 ∩ 𝐸).
откр. в 𝐸 откр. в 𝑋

𝐸 (𝑥) ⊂ 𝑈 и, значит, 𝑈 = 𝐸
⋃︀
Доказательство. (⇒) Пусть 𝑈 открыто в 𝐸. Тогда ∀𝑥 ∈ 𝑈 ∃𝐵 𝜀 𝑥 𝑥∈𝑈 𝐵𝜀𝑥 (𝑥).
Положим 𝑉 = 𝑥∈𝑈 𝐵𝜀𝑋𝑥 (𝑥). Тогда 𝑉 открыто в 𝑋 и 𝑉 ∩𝐸 = 𝑥∈𝑈 (𝐵𝜀𝑋𝑥 (𝑥)∩𝐸) = 𝑥∈𝑈 𝐵𝜀𝐸𝑥 (𝑥) = 𝑈 .
⋃︀ ⋃︀ ⋃︀
(⇐) Пусть 𝑥 ∈ 𝑈 и 𝑈 = ⏟ 𝑉⏞ ∩𝐸, тогда 𝑥 ∈ 𝑉 ⇒ ∃𝐵𝜀𝑋 (𝑥) ⊂ 𝑉 ⇒ 𝐵𝜀𝐸 (𝑥) = 𝐵𝜀𝑋 (𝑥) ∩ 𝐸 ⊂
откр. в 𝑋
𝑉 ∩ 𝐸 = 𝑈 , то есть 𝑈 открыто в 𝐸.

Следствие 13.
⏟ 𝑍⏞ ⇔∃ ⏟ 𝐹⏞ (𝑍 = 𝐹 ∩ 𝐸).
замк. в 𝐸 замк. в 𝑋

Определение 28. Пусть 𝑋 — ⋃︀множество, 𝑌 ⊂ 𝑋. Семейство {𝑋𝛼 }𝛼∈𝐴 подмножеств 𝑋 называ-


ется покрытием 𝑌 , если 𝑌 ⊂ 𝛼∈𝐴 𝑋𝛼 .
Если 𝐵 ⊂ 𝐴 и {𝑋𝛼 }𝛼∈𝐵 также является покрытием 𝑌 , то оно называется подпокрытием.

Определение 29. Пусть (𝑋, 𝜌) — метрическое пространство, 𝐾 ⊂ 𝑋. 𝐾 называется компактом


(в 𝑋), если из любого его открытого покрытия {𝐺𝜆 }𝜆∈Λ можно выделить конечное подпокрытие,
то есть ∃𝜆1 , . . . 𝜆𝑚 ∈ Λ (𝐾 ⊂ 𝐺𝜆1 ∪ . . . ∪ 𝐺𝜆𝑚 ).

Лемма 17. Пусть (𝑋, 𝜌) — метрическое пространство, 𝐾 ⊂ 𝑋. Если 𝐾 — компакт, то 𝐾


ограничено и замкнуто в 𝑋.

Доказательство. Пусть 𝑎 ∈ 𝐾. Так как ∞


⋃︀
𝑛=1 𝐵𝑛 (𝑎) = 𝑋, то {𝐵𝑛 (𝑎)}𝑛∈N — открытое покрытие
𝐾. Следовательно, 𝐾 ⊂ 𝐵𝑛1 (𝑎) ∪ . . . ∪ 𝐵𝑛𝑚 (𝑎) = 𝐵𝑁 (𝑎), где 𝑁 = max1⩽𝑖⩽𝑚 {𝑛𝑖 }, и, значит, 𝐾
ограничено. (︁ )︁
Пусть 𝑎 ∈ 𝑋 ∖𝐾. Так как ∞
⋃︀
𝑛=1 𝑋 ∖ 𝐵 1 (𝑎) = 𝑋 ∖{𝑎}, то {𝑋 ∖𝐵 1 (𝑎)}𝑛∈N — открытое покрытие
(︂ )︂ 𝑛 (︁ )︁ 𝑛

𝐾. Следовательно, 𝐾 ⊂ 𝑋 ∖ 𝐵 1 (𝑎) ∪ . . . ∪ 𝑋 ∖ 𝐵 1 (𝑎) = 𝑋 ∖ 𝐵 1 (𝑎), где 𝑁 = max1⩽𝑖⩽𝑚 {𝑛𝑖 }.


𝑛1 𝑛𝑚 𝑁

Тогда 𝐵 1 (𝑎) ⊂ 𝑋 ∖ 𝐾 и, значит, 𝑋 ∖ 𝐾 открыто, а значит, 𝐾 – замкнуто.


𝑁

Лемма 18. Замкнутое подмножество компакта — компакт.

ФПМИ МФТИ, весна 2023 28


Математический анализ Назад к содержанию

Доказательство. Пусть 𝐾 — компакт в 𝑋, ⏟ 𝐹⏞ ⊂ 𝐾. Покажем, что 𝐹 – компакт. Рассмотрим


замк. в 𝑋
открытое покрытие {𝐺𝜆 }𝜆∈Λ множества 𝐹 , тогда {𝐺𝜆 }𝜆∈Λ ∪ {𝑋 ∖ 𝐹 } — открытое покрытие 𝐾,
(︀⋃︀ )︀ 𝐹 ⊂𝐾
так как 𝜆∈Λ 𝐺𝜆 ∪ (𝑋 ∖ 𝐹 ) = 𝑋. Поскольку 𝐾 — компакт, то 𝐾 ⊂ 𝐺𝜆1 ∪ . . . ∪ 𝐺𝜆𝑚 ∪ (𝑋 ∖ 𝐹 ) ⇒
𝐹 ⊂ 𝐺𝜆1 ∪ . . . ∪ 𝐺𝜆𝑚 . Значит, 𝐹 – компакт.

Теорема 29 (о секвенциальной компактности). Пусть (𝑋, 𝜌) — метрическое пространство,


𝐾 ⊂ 𝑋. 𝐾 — компакт тогда и только тогда, когда из любой последовательности элементов
𝐾 можно выделить сходящуюся в 𝐾 подпоследовательность.

Доказательство. (⇒) Пусть ∀𝑛 ∈ N 𝑥𝑛 ∈ 𝐾. Предположим, что из {𝑥𝑛 } нельзя выделить сходя-


ющуюся подпоследовательность в 𝐾. Тогда ∀𝑎 ∈ 𝐾 ∃𝛿𝑎 > 0 ∃𝑁𝑎 ∀𝑛 ⩾ 𝑁𝑎 (𝑥𝑛 ̸∈ 𝐵𝛿𝑎 (𝑎)).
Рассмотрим {𝐵𝛿𝑎 (𝑎)}𝑎∈𝐾 — открытое покрытие 𝐾. Следовательно, 𝐾 ⊂ 𝐵𝛿𝑎1 (𝑎1 )∪. . .∪𝐵𝛿𝑎𝑚 (𝑎𝑚 ).
Положим 𝑁 = max1⩽𝑖⩽𝑚 {𝑁𝑎𝑖 }. Так как 𝑁 ⩾ 𝑁𝑎𝑖 , то 𝑥𝑁 ̸∈ 𝐵𝛿𝑎𝑖 (𝑎𝑖 ) 𝑖 = 1, . . . , 𝑚 ⇒ 𝑥𝑁 ̸∈ 𝐾 —
противоречие.
(⇐) Пусть из любой последовательности элементов 𝐾 можно выделить сходящуюся в 𝐾 под-
последовательность.

1. Покажем, что для любого 𝜀 > 0 𝐾 можно покрыть конечным набором открытых шаров
радиуса 𝜀.
Докажем от противного – пусть нельзя покрыть. Индуктивно построим последовательность
⋃︀𝑛−1
{𝑥𝑛 }, 𝑥1 ∈ 𝐾, 𝑥𝑛 ∈ 𝐾 ∖ 𝑖=1 𝐵𝜀 (𝑥𝑖 ).
По построению 𝜌(𝑥𝑖 , 𝑥𝑗 ) ⩾ 𝜀, и, значит, из {𝑥𝑛 } нельзя выделить сходящуюся подпоследова-
тельность – противоречие.

2. Пусть {𝐺𝜆 }𝜆∈Λ — открытое покрытие 𝐾, тогда ∃𝜀 > 0 ∀𝑥 ∈ 𝐾 ∃𝜆


(︁ ∈ Λ (𝐵𝜀 (𝑥) )︁⊂ 𝐺𝜆 ). Предпо-
ложим, что это не выполняется, тогда ∀𝑛 ∈ N ∃𝑥𝑛 ∈ 𝐾 ∀𝜆 ∈ Λ 𝐵 1 (𝑥𝑛 ) ̸⊂ 𝐺𝜆 .
𝑛

Имеем {𝑥𝑛 } ⊂ 𝐾 ⇒ ∃𝑥𝑛𝑘 → 𝑥 ∈ 𝐾, следовательно, ∃𝜆0 ∈ Λ(𝑥 ∈ 𝐺𝜆0 ) ⇒ ∃𝐵𝛼 (𝑥) ⊂ 𝐺𝜆0 .
⏟ ⏞
откр.
1 𝛼
Выберем 𝑘 так, чтобы 𝑥𝑛𝑘 ∈ 𝐵 (𝑥) и 𝛼
2 𝑛𝑘 < 2. Если 𝑧 ∈ 𝐵 1 (𝑥𝑛𝑘 ) ⇒ 𝜌(𝑧, 𝑥) ⩽ 𝜌(𝑧, 𝑥𝑛𝑘 ) +
𝑛𝑘
𝛼 𝛼
𝜌(𝑥𝑛𝑘 , 𝑥) < 2 + 2 = 𝛼.
Следовательно, 𝑧 ∈ 𝐵𝛼 (𝑥), 𝐵 1 (𝑥𝑛𝑘 ) ⊂ 𝐵𝛼 (𝑥) ⊂ 𝐺𝜆0 — противоречие.
𝑛𝑘

3. Пусть {𝐺𝜆 }𝜆∈Λ – открытое покрытие 𝐾. Тогда по (2):

∃𝜀 > 0 ∀𝑥 ∈ 𝐾 ∃𝜆 ∈ Λ (𝐵𝜀 (𝑥) ⊂ 𝐺𝜆 )

По (1) ∃𝑥1 , 𝑥2 , ..., 𝑥𝑚 ∈ 𝐾, что 𝐾 ⊂ 𝐵𝜀 (𝑥𝑖 ) ∪ ... ∪ 𝐵𝜀 (𝑥𝑚 ) ⊂ 𝐺𝜆1 ∪ ... ∪ 𝐺𝜆𝑚 , где 𝜆𝑖 удовлетворяет
условию 𝐵𝜀 (𝑥𝑖 ) ⊂ 𝐺𝜆𝑖 .
Следовательно, 𝐾 – компакт.

Следствие 14. Множество 𝐾 является компактом в R𝑛 ⇔ 𝐾 ограничено и замкнуто

Доказательство. ⇒ лемма (17).


⇐ Если 𝐾 ограничено, то 𝐾 ⊂ 𝐵𝑟 (𝑥) для некоторой точки 𝑥 = (𝑥1 , . . . , 𝑥𝑛 )𝑇 и 𝑟 > 0. Рассмот-
рим замкнутый брус [𝑥1 − 𝑟, 𝑥1 + 𝑟] × . . . × [𝑥𝑛 − 𝑟, 𝑥𝑛 + 𝑟]. Этот брус содержит 𝐵𝑟 (𝑥), а значит, и
𝐾.
Тогда 𝐾 – компакт по лемме (18).

ФПМИ МФТИ, весна 2023 29


Математический анализ Назад к содержанию

Следствие 15 (теорема Больцано–Вейерштрасса). Из любой ограниченной последовательности


в R𝑛 можно выделить сходящуюся подпоследовательность.

Доказательство. Если последовательность ограничена, то она лежит в некотором замкнутом


шаре. Этот шар – компакт по следствию (14). Осталось применить теорему (29).

Пусть (𝑋, 𝜌) – метрическое пространство.

Определение 30. Последовательность {𝑥𝑛 } в 𝑋 называется фундаментальной, если

∀𝜀 > 0 ∃𝑁 ∀𝑛, 𝑚 ⩾ 𝑁 (𝜌(𝑥𝑛 , 𝑥𝑚 ) < 𝜀).

Лемма 19. Всякая сходящаяся последовательность фундаментальна.

Доказательство. 𝑥𝑛 ∈ 𝑋, 𝑥𝑛 → 𝑎. Пусть 𝜀 > 0, тогда ∃𝑁 ∀𝑛 ⩾ 𝑁 (𝜌(𝑥𝑛 , 𝑎) < 2𝜀 ). Следовательно,


∀𝑛, 𝑚 ⩾ 𝑁 :
𝜌(𝑥𝑛 , 𝑥𝑚 ) ⩽ 𝜌(𝑥𝑛 , 𝑎) + 𝜌(𝑥𝑚 , 𝑎) < 𝜀.

Определение 31. Метрическое пространство называется полным, если всякая фундаментальная


последовательность в нем сходится.

Теорема 30. Евклидово пространство R𝑛 – полное.

Доказательство.
Пусть {𝑥𝑘 } – фундаментальная последовательность в R𝑛 , 𝑥𝑘 = (𝑥1,𝑘 , . . . , 𝑥𝑛,𝑘 )𝑇 . Так как |𝑥𝑖,𝑘 −
𝑥𝑖,𝑚 | ⩽ 𝜌2 (𝑥𝑘 , 𝑥𝑚 ), то из фундаментальности {𝑥𝑘 } следует фундаментальность {𝑥𝑖,𝑘 } в R для
𝑖 = 1, . . . , 𝑛. По критерию Коши √︀∑︀для числовых последовательностей 𝑥𝑖,𝑘 → 𝑎𝑘 ∈ R. Рассмотрим
𝑛
𝑎 = (𝑎1 , . . . , 𝑎𝑛 )𝑇 . 𝜌2 (𝑥𝑘 , 𝑎) = 2 𝑛
𝑖=1 𝑘,𝑖 − 𝑎𝑖 ) → 0 при 𝑘 → ∞. Значит, 𝑥𝑘 → 𝑎 ⇒ R – полное
(𝑥
метрическое пространство.

Пример. 𝐵(𝐸) – линейное пространство всех ограниченных функций 𝑓 : 𝐸 → R.


𝐵(𝐸) является нормированным пространством относительно ‖𝑓 ‖ = sup𝑥∈𝐸 |𝑓 (𝑥)|. Имеем sup |𝑓 (𝑥)+
𝑔(𝑥)| ⩽ sup |𝑓 (𝑥)| + sup |𝑔(𝑥)|. Имеем 𝑓𝑛 → 𝑓 в 𝐵(𝐸) ⇔ ‖𝑓𝑛 − 𝑓 ‖ → 0 ⇔ sup𝑥∈𝐸 |𝑓𝑛 (𝑥) − 𝑓 (𝑥)| →
0 ⇔ 𝑓𝑛 ⇒ 𝑓 на 𝐸.

Теорема 31. 𝐵(𝐸) – полное.

Доказательство. Пусть {𝑓𝑛 } фундаментальна в 𝐵(𝐸), 𝜀 > 0. Тогда

∃𝑁 ∀𝑛, 𝑚 ⩾ 𝑁 (sup |𝑓𝑛 (𝑥) − 𝑓𝑚 (𝑥)| ⩽ 𝜀).


𝑥∈𝐸

По критерию Коши равномерной сходимости ∃𝑓 : 𝑓𝑛 ⇒ 𝑓 на 𝐸. Осталось доказать, что равно-


мерный предел ограниченных функций – ограниченная функция. Для 𝜀 = 1 ∃𝑁 : |𝑓𝑁 (𝑥) − 𝑓 (𝑥)| ⩽
1 ∀𝑥 ∈ 𝐸 ⇒ |𝑓 (𝑥)| ⩽ |𝑓𝑁 (𝑥)| + 1 ⇒ 𝑓 ∈ 𝐵(𝐸) ⇒ 𝐵(𝐸) – полное.

ФПМИ МФТИ, весна 2023 30


Математический анализ Назад к содержанию

6. Предел функции, отображающей метрическое пространство в


метрическое пространство, его свойства. Предел по подмножествам.
Равносильные условия непрерывности. Непрерывность компози-
ции. Критерий непрерывности через прообразы. Непрерывные функ-
ции на компактах. Теорема Вейерштрасса. Эквивалентность норм в
конечномерных пространствах (б/д). Теорема Кантора о равномер-
ной непрерывности. Связные множества в метрических простран-
ствах. Теорема о промежуточном значении. Линейно связные мно-
жества. Линейные отображения из R𝑛 в R𝑚 , операторная норма.
Пусть (𝑋, 𝜌𝑥 ), (𝑌, 𝜌𝑦 ) – метрические пространства, 𝑎 – предельная точка 𝑋, и задана функция
𝑓 : 𝑋 ∖ {𝑎} → 𝑌 .

Определение 32 (Коши). Точка 𝑏 ∈ 𝑌 называется пределом функции 𝑓 в точке 𝑎, если

∀𝜀 > 0 ∃𝛿 > 0 ∀𝑥 ∈ 𝑋(0 < 𝜌𝑋 (𝑥, 𝑎) < 𝛿 ⇒ 𝜌𝑌 (𝑓 (𝑥), 𝑏) < 𝜀)


или, что эквивалентно,
˚𝛿 (𝑎) ⇒ 𝑓 (𝑥) ∈ 𝐵𝜀 (𝑏).
∀𝜀 > 0 ∃𝛿 > 0 ∀𝑥 ∈ 𝑋(𝑥 ∈ 𝐵

Определение 33 (Гейне). Точка 𝑏 ∈ 𝑌 называется пределом функции 𝑓 в точке 𝑎, если

∀{𝑥𝑛 }, 𝑥𝑛 ∈ 𝑋 ∖ {𝑎}(𝑥𝑛 → 𝑎 ⇒ 𝑓 (𝑥𝑛 ) → 𝑏).

Как и в случае числовых функций, доказывается равносильность определений по Коши и по


Гейне, поэтому в обоих случаях пишут lim𝑥→𝑎 𝑓 (𝑥) = 𝑏, или 𝑓 (𝑥) → 𝑏 при 𝑥 → 𝑎.

Свойство 8 (единственность). Если lim𝑥→𝑎 𝑓 (𝑥) = 𝑏 и lim𝑥→𝑎 𝑓 (𝑥) = 𝑐, то 𝑏 = 𝑐.

Доказательство. Пусть 𝑥𝑛 → 𝑎 и 𝑥𝑛 ̸= 𝑎. По определению Гейне 𝑓 (𝑥𝑛 ) → 𝑏 и 𝑓 (𝑥𝑛 ) → 𝑐. Так как


последовательность в метрическом пространстве имеет не более одного предела, то 𝑏 = 𝑐.

Свойство 9. 𝑓, 𝑔 : 𝑋 ∖ {𝑎} → R и lim𝑥→𝑎 𝑓 (𝑥) = 𝑏, lim𝑥→𝑎 𝑔(𝑥) = 𝑐. Тогда lim𝑥→𝑎 (𝑓 (𝑥) + 𝑔(𝑥)) =
𝑏 + 𝑐, lim𝑥→𝑎 𝑓 (𝑥)𝑔(𝑥) = 𝑏𝑐.

Доказательство. 𝑥𝑛 ∈ 𝑋 ∖ {𝑎}, 𝑥𝑛 → 𝑎 ⇒ 𝑓 (𝑥𝑛 ) → 𝑏, 𝑔(𝑥𝑛 ) → 𝑐 ⇒ 𝑓 (𝑥𝑛 ) + 𝑔(𝑥𝑛 ) → 𝑏 + 𝑐,


𝑓 (𝑥𝑛 )𝑔(𝑥𝑛 ) → 𝑏𝑐. Утверждение следует по определению Гейне.

В дальнейшем, говоря о «пределе по подможеству», всегда будем иметь в виду подпростран-


ство с индуцированной метрикой.

Свойство 10 (предел по подмножеству). Пусть 𝐸 ⊂ 𝑋, 𝑎 – предельная точка множества 𝐸.


Если lim𝑥→𝑎 𝑓 (𝑥) = 𝑏, то lim𝑥→𝑎 (𝑓 |𝐸 )(𝑥) = 𝑏.

Доказательство. Пусть 𝑥𝑛 ⊂ 𝐸, 𝑥𝑛 → 𝑎 и 𝑥𝑛 ̸= 𝑎. Тогда (𝑓 |𝐸 )(𝑥𝑛 ) = 𝑓 (𝑥𝑛 ) → 𝑏. По определению


Гейне lim𝑥→𝑎 (𝑓 |𝐸 )(𝑥) = 𝑏.
˚𝛿 (𝑎))
Свойство 11 (локальная ограниченность). Если существует lim𝑥→𝑎 𝑓 (𝑥), то ∃𝛿 > 0 : 𝑓 (𝐵
ограничено.

Доказательство. Достаточно положить в определении Коши 𝜀 = 1.

ФПМИ МФТИ, весна 2023 31


Математический анализ Назад к содержанию

Определение 34. Функция 𝑓 непрерывна в точке 𝑎 ∈ 𝑋, если

∀𝜀 > 0 ∃𝛿 > 0 ∀𝑥 ∈ 𝑋 (𝜌𝑋 (𝑥, 𝑎) < 𝛿 ⇒ 𝜌𝑌 (𝑓 (𝑥), 𝑓 (𝑎)) < 𝜀)

или, что эквивалентно,

∀𝜀 > 0 ∃𝛿 > 0 ∀𝑥 ∈ 𝑋 (𝑥 ∈ 𝐵𝛿 (𝑎) ⇒ 𝑓 (𝑥) ∈ 𝐵𝜀 (𝑓 (𝑎))) .

Лемма 20. Пусть 𝑓 : 𝑋 → 𝑌 , 𝑎 ∈ 𝑋. Следующие условия эквивалентны:


1. функция 𝑓 непрерывна в точке 𝑎;
2. ∀{𝑥𝑛 }, 𝑥𝑛 ∈ 𝑋 (𝑥𝑛 → 𝑎 ⇒ 𝑓 (𝑥𝑛 ) → 𝑓 (𝑎));
3. 𝑎 – изолированная точка множества 𝑋 или 𝑎 – предельная точка 𝑋 и lim𝑥→𝑎 𝑓 (𝑥) = 𝑓 (𝑎).
Доказательство. (1) ⇒ (2) Выберем 𝜀 > 0 и соответствующее 𝛿 > 0 из определения непрерыв-
ности. Если 𝑥𝑛 → 𝑎 (в 𝑋), то существует такой номер 𝑁 , что 𝜌𝑋 (𝑥𝑛 , 𝑎) < 𝛿 при всех 𝑛 ⩾ 𝑁 , но
тогда 𝜌𝑌 (𝑓 (𝑥𝑛 ), 𝑓 (𝑎)) < 𝜀 при 𝑛 ⩾ 𝑁 . Это означает, что 𝑓 (𝑥𝑛 ) → 𝑓 (𝑎).
(2) ⇒ (3) Если 𝑎 – предельная точка 𝑋, то из условия lim𝑥→𝑎 𝑓 (𝑥) = 𝑓 (𝑎) по определению
Гейне.
(3) ⇒ (1) Если 𝑎 изолирована, то 𝐵𝛿0 (𝑎) ∩ 𝑋 = {𝑎} для некоторого 𝛿0 > 0. Тогда для любого
𝜀 > 0 определение непрерывности в точке 𝑎 выполняется при 𝛿 = 𝛿0 . Пусть 𝑎 предельная для 𝑋.
По определению предела по Коши ∀𝜀 > 0 ∃𝛿 > 0 ∀𝑥 ∈ 𝐸 (0 < 𝜌𝑋 (𝑥, 𝑎) < 𝛿 ⇒ 𝜌𝑌 (𝑓 (𝑥), 𝑓 (𝑎)) < 𝜀).
Но последняя импликация верна и для 𝑥 = 𝑎. Значит, функция 𝑓 непрерывна в точке 𝑎.

Теорема 32 (о непрерывности композиции). Пусть (𝑋, 𝜌𝑋 ), (𝑌, 𝜌𝑌 ) и (𝑍, 𝜌𝑍 ) – метрические


пространства. Если функция 𝑓 : 𝑋 → 𝑌 непрерывна в точке 𝑎 ∈ 𝑋, и функция 𝑔 : 𝑌 → 𝑍
непрерывна в точке 𝑓 (𝑎) ∈ 𝑌 , то их композиция 𝑔 ∘ 𝑓 : 𝑋 → 𝑍 непрерывна в точке 𝑎.
Доказательство. Пусть 𝑥𝑛 → 𝑎, тогда 𝑓 (𝑥𝑛 ) → 𝑓 (𝑎) и, значит, 𝑔(𝑓 (𝑥𝑛 )) → 𝑔(𝑓 (𝑎)).

Теорема 33 (критерий непрерывности). Функция 𝑓 : 𝑋 → 𝑌 непрерывна ⇔ для любого откры-


того 𝑉 ⊂ 𝑌 множество 𝑓 −1 (𝑉 ) открыто в 𝑋.
Доказательство. (⇒) Пусть 𝑉 открыто в 𝑌 . Если 𝑥 ∈ 𝑓 −1 (𝑉 ), то 𝑓 (𝑥) ∈ 𝑉 и, значит, существует
такое 𝜀 > 0, что 𝐵𝜀 (𝑓 (𝑥)) ⊂ 𝑉 . Функция 𝑓 непрерывна в точке 𝑥, поэтому найдется такое 𝛿 > 0,
что 𝑓 (𝐵𝛿 (𝑥)) ⊂ 𝐵𝜀 (𝑓 (𝑥)). Отсюда следует, что 𝐵𝛿 (𝑥) ⊂ 𝑓 −1 (𝑉 ) ⇒ 𝑓 −1 (𝑉 ) открыто в 𝑋.
(⇐) Пусть 𝑥 ∈ 𝑋, и 𝜀 > 0. Шар 𝐵𝜀 (𝑓 (𝑥)) открыт в 𝑌 , поэтому множество 𝑓 −1 (𝐵𝜀 (𝑓 (𝑥)))
открыто в 𝑋 и, значит, существует 𝛿 > 0, что 𝐵𝛿 (𝑥) ⊂ 𝑓 −1 (𝐵𝜀 (𝑓 (𝑥))), или 𝑓 (𝐵𝛿 (𝑥)) ⊂ 𝐵𝜀 (𝑓 (𝑥)).
Так как 𝜀 > 0 – любое, то 𝑓 непрерывна в точке 𝑥.

Следствие 16. Функция 𝑓 : 𝑋 → 𝑌 непрерывна на 𝑋 ⇔ для каждого замкнутого множества


𝐹 ⊂ 𝑌 множество 𝑓 −1 (𝐹 ) замкнуто в 𝑋.
Доказательство. Следует из теоремы в силу равенства 𝑋 ∖ 𝑓 −1 (𝐹 ) = 𝑓 −1 (𝑌 ∖ 𝐹 ), верного для
любого 𝐹 ⊂ 𝑌 .

Теорема 34. Если функция 𝑓 : 𝐾 → 𝑌 непрерывна, и 𝐾 компакт, то 𝑓 (𝐾) – компакт в 𝑌 .


Доказательство. Пусть {𝐺𝜆 }𝜆∈Λ – открытое покрытие 𝑓 (𝐾). Если 𝑥 ∈ 𝐾, то существует такое
𝜆0 ∈ Λ, что 𝑓 (𝑥) ∈ 𝐺𝜆0 и, значит, 𝑥 ∈ 𝑓 −1 (𝐺𝜆0 ). Следовательно, семейство {𝑓 −1 (𝐺𝜆 )}𝜆∈Λ обра-
зует открытое покрытие 𝐾. Это покрытие открыто по критерию непрерывности. Поскольку 𝐾
компакт, то 𝐾 ⊂ 𝑓 −1 (𝐺𝜆1 ) ∪ . . . ∪ 𝑓 −1 (𝐺𝜆𝑚 ).
Покажем, что 𝑓 (𝐾) ⊂ 𝐺𝜆1 ∪ . . . ∪ 𝐺𝜆𝑚 . Действительно, если 𝑦 ∈ 𝑓 (𝐾), то 𝑦 = 𝑓 (𝑥) для
некоторого 𝑥 ∈ 𝐾. Найдем такое 𝑘, что 𝑥 ∈ 𝑓 −1 (𝐺𝜆𝑘 ), тогда, в свою очередь, 𝑦 = 𝑓 (𝑥) ∈ 𝐺𝜆𝑘 .
Следовательно, 𝑓 (𝐾) – компакт.

ФПМИ МФТИ, весна 2023 32


Математический анализ Назад к содержанию

Следствие 17 (теорема Вейерштрасса). Если функция 𝑓 : 𝐾 → R непрерывна, и 𝐾 компакт, то


существуют точки 𝑥𝑚 , 𝑥𝑀 ∈ 𝐾, такие что 𝑓 (𝑥𝑀 ) = sup 𝑓 (𝑥) и 𝑓 (𝑥𝑚 ) = inf 𝑓 (𝑥).
𝑥∈𝐾 𝑥∈𝐾

Доказательство. 𝑓 (𝐾) — компакт в R, следовательно, 𝑓 (𝐾) замкнуто и ограничено.


Так как 𝑓 (𝐾) ограничено, то 𝑀 = sup𝐾 𝑓 (𝑥) ∈ R. 𝑀 — граничная точка 𝑓 (𝐾), следовательно,
𝑀 ∈ 𝑓 (𝐾) и, значит, ∃𝑥𝑀 ∈ 𝐾 𝑓 (𝑥) = 𝑀 .
Доказательство для inf 𝐾 𝑓 аналогично.

Определение 35. Пусть 𝑉 – линейное пространство, ‖ · ‖, ‖ · ‖* нормы на 𝑉 . Нормы ‖ · ‖ и ‖ · ‖*


называются эквивалентными, если существуют такие 𝛼 > 0 и 𝛽 > 0, что

∀𝑥 ∈ 𝑉 (𝛼‖𝑥‖ ⩽ ‖𝑥‖* ⩽ 𝛽‖𝑥‖) .

Следствие 18. На конечномерном пространстве 𝑉 все нормы эквивалентны.

Определение 36. Функция 𝑓 : 𝑋 → 𝑌 называется равномерно непрерывной (на 𝑋), если

∀𝜀 > 0 ∃𝛿 > 0 ∀𝑥, 𝑥′ ∈ 𝑋 (𝜌𝑋 (𝑥, 𝑥′ ) < 𝛿 ⇒ 𝜌𝑌 (𝑓 (𝑥), 𝑓 (𝑥′ )) < 𝜀).

Теорема 35 (Кантор). Если функция 𝑓 : 𝐾 → 𝑌 непрерывна, и 𝐾 компакт, то 𝑓 равномерно


непрерывна.

Доказательство. Пусть 𝜀 > 0. По определению непрерывности


(︁ 𝜀 )︁
∀𝑎 ∈ 𝐾 ∃𝛿𝑎 > 0 ∀𝑥 ∈ 𝑋 𝜌𝑋 (𝑥, 𝑎) < 𝛿𝑎 ⇒ 𝜌𝑌 (𝑓 (𝑥), 𝑓 (𝑎)) < ,
2
Семейство {𝐵 𝛿𝑎 }𝑎∈𝐾 — открытое покрытие 𝐾. Так как 𝐾 — компакт, то 𝐾 ⊂ 𝐵 𝛿𝑎1 (𝑎1 ) ∪ . . . ∪
2 2
𝐵 𝛿𝑎𝑚 (𝑎𝑚 ).
2 {︁ }︁
𝛿
Положим 𝛿 = min1⩽𝑖⩽𝑚 2𝑎𝑖 . Покажем, что 𝛿 будет удовлетворять определению равномерной
непрерывности для 𝜀.
Пусть 𝜌𝐾 (𝑥, 𝑥′ ) < 𝛿. Найдётся 𝑖, 1 ⩽ 𝑖 ⩽ 𝑚, что 𝑥 ∈ 𝐵 𝛿𝑎𝑖 (𝑎𝑖 ). Тогда
2

𝛿𝑎𝑖 𝛿𝑎
𝜌𝐾 (𝑥′ , 𝑎𝑖 ) ⩽ 𝜌𝐾 (𝑥′ , 𝑥) + 𝜌𝐾 (𝑥, 𝑎𝑖 ) < + 𝑖 = 𝛿𝑎𝑖 ,
2 2
и, значит, 𝑥, 𝑥′ ∈ 𝐵𝛿𝑎𝑖 (𝑎𝑖 ). Поэтому
𝜀 𝜀
𝜌𝑌 (𝑓 (𝑥), 𝑓 (𝑥′ )) ⩽ 𝜌𝑌 (𝑓 (𝑥), 𝑓 (𝑎𝑖 )) + 𝜌𝑌 (𝑓 (𝑎𝑖 ), 𝑓 (𝑥′ )) < + = 𝜀.
2 2

Определение 37. Метрическое пространство 𝑋 называется несвязным, если существуют непу-


стые открытые 𝑈, 𝑉 ⊂ 𝑋, что 𝑋 = 𝑈 ∪ 𝑉 и 𝑈 ∩ 𝑉 = ∅.
Метрическое пространство 𝑋 называется связным, если оно не является несвязным.
Множество 𝐸 ⊂ 𝑋 называется несвязным (связным), если оно несвязно (связно) как подпро-
странство 𝑋.

Замечание. Согласно устройству открытых множеств подпространства получаем, что 𝐸 ⊂ 𝑋


несвязно, если существуют открытые 𝑈, 𝑉 ⊂ 𝑋, такие что 𝐸 ⊂ 𝑈 ∪ 𝑉 и 𝐸 ∩ 𝑈 ̸= ∅, 𝐸 ∩ 𝑉 ̸= ∅,
𝑈 ∩ 𝑉 ∩ 𝐸 = ∅.

Покажем, что 𝑈 и 𝑉 можно всегда выбрать непересекающимися.

ФПМИ МФТИ, весна 2023 33


Математический анализ Назад к содержанию

Лемма 21. Множество 𝐸 ⊂ 𝑋 несвязно ⇔ существуют открытые 𝑈, 𝑉 ⊂ 𝑋, такие что


𝐸 ⊂ 𝑈 ∪ 𝑉 и 𝐸 ∩ 𝑈 ̸= ∅, 𝐸 ∩ 𝑉 ̸= ∅, 𝑈 ∩ 𝑉 = ∅.

Доказательство. Достаточность очевидна. Для доказательства необходимости предположим,


что множество 𝐸 несвязно. Тогда существуют непустые открытые 𝑈𝐸 , 𝑉𝐸 ⊂ 𝐸, такие что 𝐸 =
𝑈𝐸 ∪ 𝑉𝐸 , 𝑈𝐸 ∩ 𝑉𝐸 = ∅.
Для каждого 𝑥 ∈ 𝑈𝐸 найдется такое 𝛿𝑥 > 0, что 𝐵𝛿𝑥 (𝑥) ∩ 𝐸 ⊂ 𝑈𝐸 и, значит, 𝐵𝛿𝑥 (𝑥) ∩ 𝑉𝐸 = ∅.
Аналогично, для каждого
⋃︀ 𝑦 ∈ 𝑉𝐸 найдется
⋃︀ такое 𝛿𝑦 > 0, что 𝐵𝛿𝑦 (𝑦) ∩ 𝐸 ⊂ 𝑉𝐸 и 𝐵𝛿𝑦 (𝑦) ∩ 𝑈𝐸 = ∅.
Положим 𝑈 = 𝐵 𝛿𝑥 (𝑥), 𝑉 = 𝐵 𝛿𝑦 (𝑦). Если существует 𝑧 ∈ 𝑈 ∩ 𝑉 , то 𝑧 ∈ 𝐵 𝛿𝑥 (𝑥) и
𝑥∈𝑈𝐸 2 𝑦∈𝑉𝐸 2 2

𝑧 ∈ 𝐵 𝛿𝑦 (𝑦) для некоторых 𝑥 ∈ 𝑈𝐸 и 𝑦 ∈ 𝑉𝐸 , тогда


2

𝛿𝑥 + 𝛿𝑦
𝜌(𝑥, 𝑦) ⩽ 𝜌(𝑥, 𝑧) + 𝜌(𝑧, 𝑦) < ⩽ 𝑚𝑎𝑥{𝛿𝑥 , 𝛿𝑦 }.
2
Если 𝑚𝑎𝑥{𝛿𝑥 , 𝛿𝑦 } = 𝛿𝑥 , то 𝑦 ∈ 𝐵𝛿𝑥 (𝑥); если же 𝑚𝑎𝑥{𝛿𝑥 , 𝛿𝑦 } = 𝛿𝑦 , то 𝑥 ∈ 𝐵𝛿𝑦 (𝑦). Обе эти ситуации
невозможны. Следовательно, 𝑈 ∩ 𝑉 = ∅.

Теорема 36. Множество 𝐼 ⊂ R связно ⇔ 𝐼 – промежуток.

Доказательство. (⇒) Если 𝐼 не является промежутком, то существуют 𝑥, 𝑦 ∈ 𝐼 и 𝑧 ∈ R, такие


что 𝑥 < 𝑧 < 𝑦 и 𝑧 ̸∈ 𝐼. Рассмотрим (−∞, 𝑧) ∩ 𝐼 и (𝑧, +∞) ∩ 𝐼. Это непустые (содержат соответ-
ственно точки 𝑥, 𝑦), непересекающиеся, открытые в 𝐼 множества, объединение которых совпадает
с 𝐼. Значит, множество 𝐼 несвязно.
(⇐) Предположим, что промежуток 𝐼 не является связным множеством. Тогда найдутся от-
крытые (в R) множества 𝑈 и 𝑉 , такие что 𝐼 ⊂ 𝑈 ∪ 𝑉 , 𝐼 ∩ 𝑈 ̸= ∅, 𝐼 ∩ 𝑉 ̸= ∅ и 𝑈 ∩ 𝑉 ∩ 𝐼 ̸= ∅. Пусть
𝑥 ∈ 𝐼 ∩ 𝑈 и 𝑦 ∈ 𝐼 ∩ 𝑉 . Без ограничения общности можно считать, что 𝑥 < 𝑦 (тогда [𝑥,𝑦] ⊂ 𝐼).
Положим 𝑆 = {𝑧 ∈ [𝑥, 𝑦] : 𝑧 ∈ 𝑈 }. Так как 𝑆 не пусто и ограничено, то существует 𝑐 = sup 𝑆.
В силу замкнутости отрезка 𝑐 ∈ [𝑥, 𝑦]. Отрезок [𝑥, 𝑦] ⊂ 𝐼 ⊂ 𝑈 ∪ 𝑉 , поэтому 𝑐 ∈ 𝑈 или 𝑐 ∈ 𝑉 .
Если 𝑐 ∈ 𝑈 , то 𝑐 ̸= 𝑦, и значит, найдется 𝜀 > 0, что полуинтервал [𝑐, 𝑐 + 𝜀) лежит одновременно
в 𝑈 и [𝑥, 𝑦]. Но тогда [𝑐, 𝑐 + 𝜀) ⊂ 𝑆, что противоречит 𝑐 = sup 𝑆.
Если 𝑐 ∈ 𝑉 , то 𝑐 ̸= 𝑥, и значит, найдется 𝜀 > 0, что полуинтервал (𝑐 − 𝜀, 𝑐] лежит одновременно
в 𝑉 и [𝑥, 𝑦]. В частности, отрезок [𝑐 − 2𝜀 , 𝑐] не пересекается с 𝑆, что противоречит 𝑐 = sup 𝑆.
Значит, 𝐼 связно.

Теорема 37. Если функция 𝑓 : 𝑆 → 𝑌 непрерывна, и множество 𝑆 связно, то множество 𝑓 (𝑆)


связно в 𝑌 .

Доказательство. Предположим, что 𝑓 (𝑆) несвязно, тогда существют открытые в 𝑌 множества


𝑈 и 𝑉 , такие что 𝑓 (𝑆) ⊂ 𝑈 ∪ 𝑉 , 𝑓 (𝑆) ∩ 𝑈 ̸= ∅, 𝑓 (𝑆) ∩ 𝑉 ̸= ∅ и 𝑓 (𝑆) ∩ 𝑈 ∩ 𝑉 = ∅. Множества
𝑓 −1 (𝑈 ) и 𝑓 −1 (𝑉 ) не пусты, не пересекаются, открыты в 𝑆 (по критерию непрерывности) и 𝑆 =
𝑓 −1 (𝑈 ) ∪ 𝑓 −1 (𝑉 ) (так как 𝑈, 𝑉 образуют покрытие 𝑓 (𝑆)). Это противоречит связности 𝑆.

Следствие 19 (Теорема о промежуточных значениях). Если функция 𝑓 : 𝑆 → R непрерывна,


и множество 𝑆 связно, то 𝑓 принимает все промежуточные значения (то есть если 𝑢, 𝑣 ∈ 𝑓 (𝑆) и
𝑢 < 𝑣, то [𝑢, 𝑣] ⊂ 𝑓 (𝑆)).

Доказательство. По теореме (37) множество 𝑓 (𝑆) связно в R и, значит, по теореме (36) является
промежутком.

Определение 38. Открытое связное множество в метрическом пространстве называется обла-


стью.

Выделим класс множеств, для которых проверка связности осуществляется несколько проще.

ФПМИ МФТИ, весна 2023 34


Математический анализ Назад к содержанию

Определение 39. Метрическое пространство 𝑋 называется линейно связным, если для любых
точек 𝑥, 𝑦 ∈ 𝑋 существует такая непрерывная функция 𝛾 : [0, 1] → 𝑋, что 𝛾(0) = 𝑥, 𝛾(1) = 𝑦.

Теорема 38. Всякое линейно связное метрическое пространство связно.

Доказательство. Предположим, что линейно связное пространство 𝑋 несвязно. Тогда найдутся


непустые открытые множества 𝑈 и 𝑉 , такие что 𝑋 = 𝑈 ∪ 𝑉 и 𝑈 ∩ 𝑉 = ∅. Пусть 𝑥 ∈ 𝑈 и
𝑦 ∈ 𝑉 . Так как 𝑋 линейно связно, то существует непрерывная функция 𝛾 : [0, 1] → 𝑋, такая
что 𝛾(0) = 𝑥 и 𝛾(1) = 𝑦. Тогда 𝛾 −1 (𝑈 ) и 𝛾 −1 (𝑉 ) не пусты, не пересекаются, открыты в [0, 1], и
[0, 1] = 𝛾 −1 (𝑈 ) ∪ 𝛾 −1 (𝑉 ), что невозможно, так как отрезок [0, 1] связен.

Лемма 22. Связное открытое множество 𝐸 в нормированном пространстве линейно связно.

Доказательство. Пусть 𝑥 ∈ 𝐸. Рассмотрим множество 𝑈 тех точек 𝑦, которые можно соединить


с 𝑥 кривой, то есть существует непрерывная функция 𝛾 : [0, 1] → 𝐸, что 𝛾(0) = 𝑥, 𝛾(1) = 𝑦.
Покажем, что 𝑈 открыто. Для 𝑦 ∈ 𝑈 в силу открытости 𝐸 найдется такое 𝜀 > 0, что 𝐵𝜀 (𝑦) ⊂ 𝐸.
Любая пара точек в шаре может быть соединена открезком: для 𝑧 ∈ 𝐵𝜀 (𝑦) рассмотрим 𝜎 : [0, 1] →
𝐵𝜀 (𝑦), 𝜎(𝑡) = (1 − 𝑡)𝑦 + 𝑡𝑧. Тогда кривая
{︃
𝛾(2𝑡), 0 ⩽ 𝑡 ⩽ 21 ,
𝛾 ∘ 𝜎(𝑡) =
𝜎(2𝑡 − 1), 12 ⩽ 𝑡 ⩽ 1,

соединяет 𝑥 и 𝑧, поэтому 𝐵𝜀 (𝑦) ⊂ 𝑈 . Аналогично устанавливается, что 𝐸 ∖ 𝑈 открыто. В силу


связности 𝐸 ∖ 𝑈 пусто, то есть 𝐸 = 𝑈 .

Определение 40. Отображение 𝐿 называется линейным, если ∀𝑥1 , 𝑥2 ∈ 𝑋 и ∀𝛼1 , 𝛼2 ∈ R выпол-


нено 𝐿(𝛼1 𝑥1 + 𝛼2 𝑥2 ) = 𝛼1 𝐿(𝑥1 ) + 𝛼2 𝐿(𝑥2 ).

Определение 41. Для 𝐿 ∈ ℒ(𝑋, 𝑌 ) определим ‖𝐿‖ = sup ‖𝐿(𝑥)‖


‖𝑥‖ .
𝑥̸=0

Замечание. ‖𝐿‖ ∈ R. По определению супремума ‖𝐿(𝑥)‖ ⩽ ‖𝐿‖‖𝑥‖ для всех 𝑥 ∈ 𝑋, и для всякого
𝜀 > 0 найдется такое 𝑥𝜀 ∈ 𝑋, что ‖𝐿(𝑥)‖ > (‖𝐿‖ − 𝜀)‖𝑥𝜀 ‖. Это означает, что ‖𝐿‖ – наименьшее из
чисел 𝐶 > 0, таких что ‖𝐿(𝑥)‖ ⩽ 𝐶‖𝑥‖ для всех 𝑥 ∈ 𝑋.
Нетрудно проверить, что (ℒ(𝑋, 𝑌 ), ‖ · ‖) является нормированным пространством, причем
‖𝐿2 𝐿1 ‖ ⩽ ‖𝐿2 ‖‖𝐿1 ‖.

ФПМИ МФТИ, весна 2023 35


Математический анализ Назад к содержанию

7. Дифференцируемость функции из R𝑛 в R𝑚 . Производная по век-


тору и ее связь с дифференциалом. Дифференцируемость ком-
позиции. Связь дифференцируемости функции с дифференциру-
емостью ее координатных функций. Частные производные, необ-
ходимые условия дифференцируемости. Градиент. Матрица Яко-
би. Достаточные условия дифференцируемости. Частные произ-
водные высших порядков. Независимость смешанной производной
от порядка дифференцирования. Дифференциалы высших поряд-
ков и кратная дифференцируемость. Формула Тейлора с остаточ-
ным членом в форме Лагранжа, в форме Пеано (б/д).
Пусть 𝑈 ⊂ R𝑛 , 𝑈 – открытое и задана функция 𝑓 : 𝑈 → R𝑛 .

Определение 42. Функция 𝑓 называется дифференцируемой в точке 𝑎, если существует такое


непрерывное линейное отображение 𝐿𝑎 : 𝑋 → 𝑌 , что

𝑓 (𝑎 + ℎ) = 𝑓 (𝑎) + 𝐿𝑎 (ℎ) + 𝛼(ℎ)‖ℎ‖,

для некоторой функции 𝛼, такой что 𝛼(ℎ) → 0.

Замечание. Формула (42) не определяет значение 𝛼 в нуле. В дальнейшем будем считать, что
𝛼(0) = 0 и, значит, функция 𝛼 непрерывна в нуле.
Формулу (42) можно написать в виде

𝑓 (𝑎 + ℎ) = 𝑓 (𝑎) + 𝑑𝑓𝑎 (ℎ) + 𝑜(‖ℎ‖), ℎ → 0.

Линейное отображение 𝐿𝑎 называется дифференциалом 𝑓 в точке 𝑎 и обозначается 𝑑𝑓𝑎 .

Определение 43. Пусть 𝑣 ∈ R𝑛 и функция 𝑓 определена на множестве {𝑎 + 𝑡𝑣 : |𝑡| < 𝛿} для


некоторого 𝛿 > 0. Предел
𝑓 (𝑎 + 𝑡𝑣) − 𝑓 (𝑎)
lim ,
𝑡→0 𝑡
если этот предел существует, называется производной 𝑓 по вектору v в точке 𝑎 и обозначается
𝜕𝑓 ′
𝜕𝑣 (𝑎) (а также 𝑓𝑣 (𝑎) и 𝜕𝑣 𝑓 (𝑎)).
𝜕𝑓
Теорема 39. Если 𝑓 : 𝑈 → R𝑛 дифференцируема в точке 𝑎, 𝑣 ∈ R𝑛 , то существует 𝜕𝑣 (𝑎) =
𝑑𝑓𝑎 (𝑣).

Доказательство. Для 𝑣 = 0 утверждение верно. Пусть 𝑣 ̸= 0. Выберем 𝛿 > 0 так, что 𝐵𝛿 (𝑎) ⊂ 𝑈 .
𝛿
Тогда для всех 𝑡 ∈ R с |𝑡| < |𝑣| , получим

𝑓 (𝑎 + 𝑡𝑣) = 𝑓 (𝑎) + 𝑑𝑓𝑎 (𝑡𝑣) + 𝛼(𝑡𝑣)‖𝑡𝑣‖.

В силу линейности 𝑑𝑓𝑎 (𝑡𝑣) = 𝑡𝑑𝑓𝑎 (𝑣). Далее, по непрерывности 𝛼 в 0 имеем 𝛼(𝑡𝑣) → 0 при 𝑡 → 0,
поэтому
𝜕𝑓 𝑓 (𝑎 + 𝑡𝑣) − 𝑓 (𝑎)
(𝑎) = lim = lim(𝑑𝑓𝑎 (𝑣) ± 𝛼(𝑡𝑣)‖𝑣‖) = 𝑑𝑓𝑎 (𝑣).
𝜕𝑣 𝑡→0 𝑡 𝑡→0

Теорема 40 (дифференцирование композиции). Пусть ⏟ 𝑈⏞ ⊂ R𝑛 , ⏟ 𝑉⏞ ⊂ R𝑚 .


откр. откр.

ФПМИ МФТИ, весна 2023 36


Математический анализ Назад к содержанию

Если 𝑓 : 𝑈 → R𝑚 дифференцируема в точке 𝑎, 𝑔 : 𝑉 → R𝑘 дифференцируема в точке 𝑓 (𝑎),


𝑓 (𝑈 ) ⊂ 𝑉 , то композиция 𝑔 ∘ 𝑓 : 𝑈 → R𝑘 дифференцируема в точке 𝑎 и

𝑑(𝑔 ∘ 𝑓 )𝑎 = 𝑑𝑔𝑓 (𝑎) ∘ 𝑑𝑓𝑎 .

Доказательство. Положим 𝑏 = 𝑓 (𝑎). По определению

𝑓 (𝑎 + ℎ) = 𝑓 (𝑎) + 𝑑𝑓𝑎 (ℎ) + 𝛼(ℎ)‖ℎ‖, ℎ → 0 ⇒ 𝛼(ℎ) → 0,


𝑔(𝑏 + 𝑢) = 𝑔(𝑏) + 𝑑𝑔𝑏 (𝑢) + 𝛽(𝑢)‖𝑢‖, 𝑢 → 0 ⇒ 𝛽(𝑢) → 0,

Подставим вместо 𝑢 во второе равенство выражение κ(ℎ) = 𝑑𝑓𝑎 (ℎ) + 𝛼(ℎ)‖ℎ‖.

𝑔(𝑓 (𝑎 + ℎ)) = 𝑔(𝑏 + κ(ℎ)) = 𝑔(𝑏) + 𝑑𝑔𝑏 (𝑑𝑓𝑎 (ℎ) + 𝛼(ℎ)‖ℎ‖) + 𝛽(κ(ℎ))‖κ(ℎ)‖ =
= 𝑔(𝑏) + 𝑑𝑔𝑏 (𝑑𝑓𝑎 (ℎ)) + 𝑑𝑔𝑏 (𝛼(ℎ)) · ‖ℎ‖ + 𝛽(κ(ℎ)) · ‖κ(ℎ)‖ =
‖κ(ℎ)‖
= 𝑔(𝑏) + 𝑑𝑔𝑏 (𝑑𝑓𝑎 (ℎ)) + 𝛾(ℎ)‖ℎ‖, 𝛾(ℎ) = 𝑑𝑔𝑏 (𝛼(ℎ)) + 𝛽(κ(ℎ)) .
‖ℎ‖

По теореме о непрерывности композиции 𝑑𝑔𝑏 (𝛼(ℎ)) и 𝛽(κ(ℎ)) непрерывны при ℎ = 0 со значе-


нием 0. Также ∃𝐶 ⩾ 0 (‖𝑑𝑓𝑎 (ℎ)‖ ⩽ 𝐶‖ℎ‖). Следовательно, ‖κ(ℎ)‖
‖ℎ‖ ограничена в некоторой проколо-
той окрестности ℎ = 0 и, значит, 𝛾(ℎ) — бесконечно малая при ℎ → 0 (как сумма двух бесконечно
малых).

Пусть 𝑈 ⊂ R𝑛 открыто, и функция 𝑓 : 𝑈 → R𝑚 , 𝑓 = (𝑓1 , . . . , 𝑓𝑚 )𝑇 .

Лемма 23. Функция 𝑓 дифференцируема в точке 𝑎 тогда и только тогда, когда все координат-
ные функции 𝑓𝑖 дифференцируемы в точке 𝑎.

Доказательство. Пусть 𝑓 дифференцируема в точке 𝑎. Распишем формулу (1) покоординатно:

𝑓𝑖 (𝑎 + ℎ) = 𝑓𝑖 + 𝐿𝑖 (ℎ) + 𝛼𝑖 (ℎ)|ℎ|.

Координатные функции 𝐿𝑖 дифференциала 𝐿𝑎 линейны, а условие "𝛼(ℎ) → 0 при ℎ → 0" равно-


сильно "𝛼𝑖 (ℎ) → 0 при ℎ → 0" , где 𝑖 = 1, . . . , 𝑚, поэтому функция 𝑓𝑖 дифференцируема в точке
𝑎 и ее дифференциал 𝑑(𝑓𝑖 )𝑎 = 𝐿𝑖 .
Обратно, если все функции 𝑓𝑖 дифференцируемы, то верна и формула (1) с 𝐿𝑎 = (𝐿1 , . . . , 𝐿𝑚 )𝑇
и 𝛼 = (𝛼1 , . . . , 𝛼𝑚 )𝑇 .
𝜕𝑓 𝑓 (𝑎+𝑡𝑒𝑘 )−𝑓 (𝑎)
Определение 44. Производная по вектору 𝑒𝑘 в точке 𝑎, т.е. 𝜕𝑒𝑘 (𝑎) = lim𝑡→0 𝑡 , на-
𝜕𝑓
зывается частной производной функции 𝑓 по переменной 𝑥𝑘 в точке 𝑎 и обозначается 𝜕𝑥𝑘 (𝑎) (а
также 𝑓𝑥′ 𝑘 (𝑎) и 𝜕𝑘 𝑓 (𝑎)).

Из теоремы 1 получим необходимое условие дифференцируемости.

Следствие 20. Если 𝑓 : 𝑈 → R дифференцируема ∑︀𝑛 𝜕𝑓в точке 𝑎, то она имеет𝑛 в этой точке частные
𝜕𝑓
производные 𝜕𝑥 𝑘
(𝑎), 𝑘 = 1, . . . , 𝑛, и 𝑑𝑓 𝑎 (ℎ) = 𝑘=1 𝜕𝑥𝑘 (𝑎)ℎ𝑘 для всех ℎ ∈ R .

𝜕𝑓
Доказательство. По теореме (39) существуют 𝜕𝑥 𝑘
(𝑎) = 𝑑𝑓𝑎 (𝑒𝑘 ), следовательно, в силу линейно-
сти (︃ 𝑛 𝑛 𝑛
)︃
∑︁ ∑︁ ∑︁ 𝜕𝑓
𝑑𝑓𝑎 (ℎ) = 𝑑𝑓𝑎 ℎ𝑘 𝑒𝑘 = ℎ𝑘 𝑑𝑓𝑎 (𝑒𝑘 ) = (𝑎)ℎ𝑘 .
𝜕𝑥𝑘
𝑘=1 𝑘=1 𝑘=1

ФПМИ МФТИ, весна 2023 37


Математический анализ Назад к содержанию

𝜕𝑓 𝜕𝑓
Определение 45. Вектор ( 𝜕𝑥 1
(𝑎), . . . , 𝜕𝑥𝑛
(𝑎))𝑇 называется градиентом функции 𝑓 в точке 𝑎 и
обозначается 𝑔𝑟𝑎𝑑𝑓 (𝑎) или ∇𝑓 (𝑎).

Следствие 21. Пусть 𝑓 дифференцируема в точке 𝑎, и 𝑔𝑟𝑎𝑑𝑓 (𝑎) ̸= 0. Тогда для любого 𝑣 ∈ R𝑛
с |𝑣| = 1 выполнено ⃒ ⃒
⃒ 𝜕𝑓 ⃒
⃒ (𝑎)⃒ ⩽ |𝑔𝑟𝑎𝑑𝑓 (𝑎)|,
⃒ 𝜕𝑣 ⃒
𝑔𝑟𝑎𝑑𝑓 (𝑎)
причем равенство достигается лишь при 𝑣 = ± |𝑔𝑟𝑎𝑑𝑓 (𝑎)| .

𝜕𝑓
Доказательство.
⃒ ⃒ Так как 𝜕𝑣 (𝑎) = 𝑑𝑓𝑎 (𝑣) = (𝑔𝑟𝑎𝑑𝑓 (𝑎), 𝑣), то по неравенству Коши-Буняковского-
Шварца ⃒ 𝜕𝑓
𝜕𝑣 (𝑎)⃒ ⩽ |𝑔𝑟𝑎𝑑𝑓 (𝑎)| · |𝑣| = |𝑔𝑟𝑎𝑑𝑓 (𝑎)|, причем равенство достигается лишь в случае кол-
⃒ ⃒
𝑔𝑟𝑎𝑑𝑓 (𝑎)
линеарности 𝑔𝑟𝑎𝑑𝑓 (𝑎) и 𝑣, то есть 𝑣 = ± |𝑔𝑟𝑎𝑑𝑓 (𝑎)| .

Поскольку действие линейного отображения из R𝑛 в R𝑚 на вектор есть умножение этого


вектора слева на матрицу, поэтому найдется такая матрица 𝐷𝑓𝑎 размера 𝑚×𝑛, что 𝑑𝑓𝑎 (ℎ) = 𝐷𝑓𝑎 ·ℎ
для всех ℎ ∈ R𝑛 .

Определение 46. Матрица 𝐷𝑓𝑎 называется матрицей Якоби функции 𝑓 в точке 𝑎.

Замечание. По лемме 1 следует, что 𝑑𝑓 (ℎ) = (𝑑𝑓1 (ℎ), . . . , 𝑑𝑓𝑚 (ℎ))𝑇 , поэтому 𝑖𝑗-й элемент матрицы
𝜕𝑓𝑖
Якоби в точке 𝑎 равен значению 𝑑(𝑓𝑖 )𝑎 (𝑒𝑗 ), то есть 𝜕𝑥 𝑗
(𝑎). Таким образом, строками матрицы
Якоби являются градиенты ее координатных функций в этой точке.

Теорема 41 (Достаточное условие дифференцируемости). Пусть 𝑓 : 𝑈 ⊂ R𝑛 → R, точка 𝑎 ∈ 𝑈 .


𝜕𝑓
Если все частные производные 𝜕𝑥𝑘
определены в окрестности а и непрерывны в точке 𝑎, то 𝑓
дифференцируема в точке 𝑎.
𝜕𝑓
Доказательство. Пусть все определены в 𝐵𝑟 (𝑎) ⊂ 𝑈 . Рассмотрим ℎ = (ℎ1 , . . . , ℎ𝑛 )𝑇 с |ℎ| < 𝑟,
𝜕𝑥𝑘
и определим точки 𝑥0 = 𝑎, 𝑥𝑘 = 𝑎 + 𝑘𝑗=1 ℎ𝑗 𝑒𝑗 . Тогда приращение
∑︀

𝑛
∑︁ 𝑛
∑︁
𝑓 (𝑎 + ℎ) − 𝑓 (𝑎) = (𝑓 (𝑥𝑘 ) − 𝑓 (𝑥𝑘−1 )) = (𝑓 (𝑥𝑘−1 + ℎ𝑘 𝑒𝑘 ) − 𝑓 (𝑥𝑘−1 )).
𝑘=1 𝑘=1

Функция 𝑔(𝑡) = 𝑓 (𝑥𝑘−1 + 𝑡𝑒𝑘 ) − 𝑓 (𝑥𝑘−1 ) на отрезке с концами 0 и ℎ𝑘 (при ℎ𝑘 ̸= 0) имеет


𝜕𝑓
производную 𝑔 ′ (𝑡) = 𝜕𝑥 𝑘
(𝑥𝑘−1 + 𝑡𝑒𝑘 ). По теореме Лагранжа о среднем 𝑔(ℎ𝑘 ) − 𝑔(0) = 𝑔 ′ (𝜉𝑘 )ℎ𝑘 для
некоторого 𝜉𝑘 между 0 и ℎ𝑘 . Положим 𝑐𝑘 (ℎ) = 𝑥𝑘−1 +𝜉𝑘 𝑒𝑘 , тогда последнее равенство перепишется
𝜕𝑓
в виде 𝑓 (𝑥𝑘 ) − 𝑓 (𝑥𝑘−1 ) = 𝜕𝑥 𝑘
(𝑐𝑘 )ℎ𝑘 , причем 𝑐𝑘 → 𝑎 при ℎ → 0. Поэтому
𝑛 𝑛 (︂ )︂
∑︁ 𝜕𝑓 ∑︁ 𝜕𝑓 𝜕𝑓
𝑓 (𝑎 + ℎ) − 𝑓 (𝑎) − (𝑎)ℎ𝑘 = (𝑐𝑘 ) − (𝑎) ℎ𝑘 =
𝜕𝑥𝑘 𝜕𝑥𝑘 𝜕𝑥𝑘
𝑘=1 𝑘=1

𝑛 (︂ )︂
∑︁ 𝜕𝑓 𝜕𝑓 ℎ𝑘
= (𝑐𝑘 ) − (𝑎) |ℎ| =: 𝛼(ℎ)|ℎ|.
𝜕𝑥𝑘 𝜕𝑥𝑘 |ℎ|
𝑘=1
𝜕𝑓
В силу непрерывности 𝜕𝑥 𝑘
в точке 𝑎 и неравенства |ℎ𝑘 | ⩽ |ℎ| функция 𝛼(ℎ) → 0 при ℎ → 0.
Следовательно, 𝑓 дифференцируема в точке 𝑎.

Пусть ⏟ 𝑈⏞ ⊂ R𝑛 , 𝑓 : 𝑈 → R, 𝑘 ∈ N.
откр.

ФПМИ МФТИ, весна 2023 38


Математический анализ Назад к содержанию

Определение 47. Частной производной нулевого порядка в точке 𝑎 называют 𝑓 (𝑎).


𝑘−1 𝑓
Если частная производная 𝜕𝑥𝑖 𝜕 ...𝜕𝑥 𝑖1
𝑘 − 1-го порядка определена в некоторой окрестности
𝑘−1
точки 𝑎 и меет в точке 𝑎 частную производную по 𝑥𝑖𝑘 , то

𝜕𝑘𝑓 𝜕 𝑘−1 𝑓
(︂ )︂⃒
𝜕 ⃒
:= ⃒
𝜕𝑥𝑖𝑘 𝜕𝑥𝑖𝑘−1 . . . 𝜕𝑥𝑖1 𝜕𝑥𝑖𝑘 𝜕𝑥𝑖𝑘−1 . . . 𝜕𝑥𝑖1 ⃒𝑥=𝑎

называется частной производной 𝑘-го порядка функции 𝑓 в точке 𝑎.


𝜕𝑓 𝜕𝑓
Теорема 42 (Юнг). Пусть ⏟ 𝑈⏞ ⊂ R2 , 𝑓 : 𝑈 → R. Если частные производные 𝜕𝑥 и 𝜕𝑦 определены
откр.
в некоторой окрестности точки (𝑎, 𝑏) и дифференцируемы в точке (𝑎, 𝑏), то

𝜕2𝑓 𝜕2𝑓
(𝑎, 𝑏) = (𝑎, 𝑏).
𝜕𝑦𝜕𝑥 𝜕𝑥𝜕𝑦
𝜕𝑓 𝜕𝑓
Доказательство. Выберем окрестность 𝐵𝛿 (𝑎, 𝑏), в которой определены 𝜕𝑥 и 𝜕𝑦 . Рассмотрим вы-
ражение

∆(𝑡) = 𝑓 (𝑎 + 𝑡, 𝑏 + 𝑡) − 𝑓 (𝑎 + 𝑡, 𝑏) − 𝑓 (𝑎, 𝑏 + 𝑡) + 𝑓 (𝑎, 𝑏), 0 < |𝑡| < 𝛿.


Функция 𝑔(𝑠) = 𝑓 (𝑎 + 𝑠, 𝑏 + 𝑡) − 𝑓 (𝑎 + 𝑠, 𝑏) на отрезке с концами 0 и 𝑡 имеет производную
𝑔 ′ (𝑠) = 𝜕𝑓 𝜕𝑓 ′
𝜕𝑥 (𝑎 + 𝑠, 𝑏 + 𝑡) − 𝜕𝑥 (𝑎 + 𝑠, 𝑏). По теореме Лагранжа 𝑔(𝑡) − 𝑔(0) = 𝑔 (𝜉)𝑡 для некоторого 𝜉
между 0 и 𝑡. Тогда в силу равенства ∆(𝑡) = 𝑔(𝑡) − 𝑔(0) и дифференцируемости 𝜕𝑓 𝜕𝑥 имеем

𝜕𝑓 𝜕𝑓
∆(𝑡) = 𝑔 ′ (𝜉)𝑡 =
(𝑎 + 𝜉, 𝑏 + 𝑡)𝑡 − (𝑎 + 𝜉, 𝑏)𝑡 =
𝜕𝑥 𝜕𝑥
𝜕2𝑓 𝜕2𝑓 𝜕2𝑓
(︂ )︂ (︂ )︂
𝜕𝑓 √︀ 𝜕𝑓
= (𝑎, 𝑏) + 2 (𝑎, 𝑏)𝜉 + (𝑎, 𝑏)𝑡 + 𝛼(𝑡) 𝜉 2 + 𝑡2 𝑡 − (𝑎, 𝑏) + 2 (𝑎, 𝑏)𝜉 + 𝛽(𝑡)|𝜉| 𝑡 =
𝜕𝑥 𝜕 𝑥 𝜕𝑦𝜕𝑥 𝜕𝑥 𝜕 𝑥
(︃ √︂ )︃
𝜕2𝑓 𝜉2 |𝜉| 2
= (𝑎, 𝑏) ± 𝛼(𝑡) 1 + 2 ± 𝛽(𝑡) 𝑡 ,
𝜕𝑦𝜕𝑥 𝑡 |𝑡|
Δ(𝑡) 𝜕2𝑓
где 𝛼(𝑡) → 0, 𝛽(𝑡) → 0 при 𝑡 → 0. Следовательно, существует lim𝑡→0 𝑡2
= 𝜕𝑦𝜕𝑥 (𝑎, 𝑏).
Δ(𝑡) 𝜕2𝑓
Аналогично lim𝑡→0 𝑡2
= 𝜕𝑥𝜕𝑦 (𝑎, 𝑏), что и доказывает теорему.

Распространим теорему на случай 𝑛 переменных.

Следствие 22. Пусть 𝑘 ∈ N, 𝑘 ⩾ 2. Если все частные производные до порядка 𝑘 − 2 дифферен-


цируемы в некоторой окрестности точки 𝑎, а все частные производные порядка 𝑘 − 1 дифферен-
цируемы в точке 𝑎, то

𝜕𝑘𝑓 𝜕𝑘𝑓
(𝑎) = (𝑎)
𝜕𝑥𝑖𝑘 . . . 𝜕𝑥𝑖1 𝜕𝑥𝑗𝑘 . . . 𝜕𝑥𝑗1
при условии, что списки (𝑖1 , . . . , 𝑖𝑘 ) и (𝑗1 , . . . , 𝑗𝑘 ) отличаются лишь порядком.

Доказательство. Индукция по 𝑘. Пусть 𝑘 = 2. Положим 𝑥𝑟 = 𝑎𝑟 , 𝑟 ̸= 𝑖1 ,𝑖2 , тогда имеем функцию


двух переменных 𝑥𝑖1 и 𝑥𝑖2 , и равенство вытекает по теореме Юнга (42).
Пусть 𝑘 > 2. Можно считать, что список (𝑗1 , . . . , 𝑗𝑘 ) получен из (𝑖1 , . . . , 𝑖𝑘 ) с помощью одной
транспозиции, то есть обменом 𝑖𝑟 и 𝑖𝑟−1 .
𝑟−2 𝑓
Рассмотрим 𝑔 = 𝜕𝑥𝑖 𝜕 ...𝜕𝑥 𝑖
. По теореме Юнга в окрестности точки 𝑎 имеет место равенство
𝑟−2 1
𝜕2𝑔 𝜕2𝑔 𝜕2𝑔 𝜕2𝑔
𝜕𝑥𝑖𝑟 𝜕𝑥𝑖𝑟−1 = 𝜕𝑥𝑖𝑟−1 𝜕𝑥𝑖𝑟 . При 𝑟 = 𝑘 имеем 𝜕𝑥𝑖𝑟 𝜕𝑥𝑖𝑟−1 (𝑎) = 𝜕𝑥𝑖𝑟−1 𝜕𝑥𝑖𝑟 (𝑎), что лишь формой записи

ФПМИ МФТИ, весна 2023 39


Математический анализ Назад к содержанию

отличается от требуемого равенства; при 𝑟 < 𝑘 еще надо продифференцировать по переменным


𝑥𝑖𝑟+1 , . . . , 𝑥𝑖𝑘 и подставить 𝑥 = 𝑎.

Дифференциалы высших порядков определяются индуктивно.


Пусть 𝑈 ⊂ R𝑛 открыто.

Определение 48. Положим 𝑑1 𝑓 = 𝑑𝑓 . Пусть 𝑘 ∈ N, 𝑘 ⩾ 2. Пусть 𝑑𝑘−1 𝑓 определен в некоторой


окрестности точки 𝑎 и дифференцируем в точке 𝑎, то 𝑑𝑘 𝑓𝑎 := 𝑑(𝑑𝑘−1 𝑓 )𝑎 , понимаемый как 𝑘-
линейное отображение, называется дифференциалом 𝑘-го порядка функции 𝑓 в точке 𝑎. При
этом функция 𝑓 называется 𝑘 раз дифференцируемой в точке 𝑎.

Лемма 24. Дифференциал 𝑑𝑘 𝑓 симметричен, то есть на наборах 𝑘 векторов, отличающихся


лишь порядком, принимает одинаковые значения.

Доказательство. Достаточно установить совпадение на наборах векторов стандартного базиса


и воспользуемся линейностью.
𝑘𝑓
Покажем по индукции, что 𝑑𝑘 𝑓𝑎 (𝑒𝑖1 , . . . , 𝑒𝑖𝑘 ) = 𝜕𝑥𝑖 𝜕...𝜕𝑥 𝑖
(𝑎). При 𝑘 = 1 это следует из тео-
𝑘 1
𝜕 𝑘−1 𝑓
ремы 1 и определения частной производной. Если равенство верно для 𝑘 − 1, то 𝜕𝑥𝑖𝑘−1 ...𝜕𝑥𝑖1 =
𝑑𝑘−1 𝑓 (𝑒 𝑖1 , . . . , 𝑒𝑖𝑘−1 ) дифференцируема в точке 𝑎. Следовательно,

𝜕 𝑘−1 𝑓 𝜕 𝑘−1 𝑓 𝜕𝑘𝑓


(︂ )︂ (︂ )︂
𝑘 𝜕
𝑑 𝑓𝑎 (𝑒𝑖1 ,...,𝑒𝑖𝑘 ) = 𝑑 (𝑒𝑖𝑘 ) = |𝑥=𝑎 = (𝑎).
𝜕𝑥𝑖𝑘−1 . . . 𝜕𝑥𝑖1 𝑎
𝜕𝑥𝑖𝑘 𝜕𝑥𝑖𝑘−1 . . . 𝜕𝑥𝑖1 𝜕𝑥𝑖𝑘 . . . 𝜕𝑥𝑖1

Симметричность 𝑑𝑘 𝑓 на наборах базисных векторов теперь вытекает из следствия теоремы


Юнга (42).

Следствие 23. Функция 𝑓 дифференцируема 𝑘 раз в точке 𝑎, тогда и только тогда, когда все
частные производные до порядка 𝑘 − 2 дифференцируемы в некоторой окрестности точки 𝑎, а
все частные производные порядка 𝑘 − 1 дифференцируемы в точке 𝑎.

Теорема 43 (формула Тейлора с остаточным членом в форме Лагранжа). Пусть 𝑓 : ⏟ 𝑈⏞ → R


откр.
дифференцируема (𝑝 + 1) раз на 𝑈 . Если 𝑎 ∈ 𝑈 , ℎ ∈ R𝑛 , такие что [𝑎, 𝑎 + ℎ] ⊂ 𝑈 , то ∃Θ ∈ (0, 1),
что
𝑝
∑︁ 1 𝑘 1
𝑓 (𝑎 + ℎ) = 𝑓 (𝑎) + 𝑑 𝑓𝑎 (ℎ) + 𝑑𝑝+1 𝑓𝑎+Θℎ (ℎ).
𝑘! (𝑝 + 1)!
𝑘=1

Доказательство. [𝑎, 𝑎 + ℎ] = {𝑎 + 𝑡ℎ | 𝑡 ∈ [0, 1]} — отрезок с концами 𝑎 и 𝑎 + ℎ.


Рассмотрим функцию 𝑔(𝑡) = 𝑓 (𝑎 + 𝑡ℎ), определённую на интервале, содержащем [0, 1]. Так
как 𝑡 ↦→ ⏟ 𝑎⏞ + ⏟𝑡ℎ⏞ ⇒ ∀𝜏 ∈ R 𝑑(𝑎 + 𝑡ℎ)𝑡 (𝜏 ) = 𝜏 ℎ. Тогда по теореме о дифференцировании
пост. линейн.
композиции
𝑑𝑔𝑡 (𝜏 ) = 𝑑𝑓𝑎+𝑡ℎ (𝜏 ℎ).
По индукции
𝑑𝑘 𝑔𝑡 (𝜏 ) = 𝑑𝑘 𝑓𝑎+𝑡ℎ (𝜏 ℎ) 𝑘 = 1, . . . , 𝑝 + 1.
𝜏 =1
Имеем 𝑑𝑘 𝑔𝑡 (𝜏 ) = 𝑔 (𝑘) (𝑡)𝜏 𝑘 ⇒ 𝑔 (𝑘) (𝑡) = 𝑑𝑘 𝑓𝑎+𝑡ℎ (ℎ), 𝑘 = 1, . . . , 𝑝 + 1.
По формуле Тейлора с остаточным членом в форме Лагранжа
𝑝
∑︁ 𝑔 (𝑘) (0) 𝑔 (𝑝+1) (𝜃𝑡 ) 𝑝+1
𝑔(𝑡) = 𝑔(0) + 𝑡𝑘 + 𝑡 .
𝑘! (𝑝 + 1)!
𝑘=1

ФПМИ МФТИ, весна 2023 40


Математический анализ Назад к содержанию

При 𝑡 = 1 и 𝜃 = 𝜃1 получаем искомую формулу.

Теорема 44 (остаточный член в форме Пеано). Если функция 𝑓 : ⏟ 𝑈⏞ → R дифференцируема


откр.
𝑝 раз в точке 𝑎, то
𝑝
∑︁ 1 𝑘
𝑓 (𝑎 + 𝑥) = 𝑓 (𝑎) + 𝑑 𝑓𝑎 (ℎ) + 𝑜(|ℎ|𝑝 ), ℎ → 0.
𝑘!
𝑘=1

ФПМИ МФТИ, весна 2023 41


Математический анализ Назад к содержанию

8. Брусы в R𝑛 и их объем. Представление открытого множества


в виде объединения кубов. Алгебры и 𝜎-алгебры, борелевская 𝜎-
алгебра. Внешняя мера Лебега и ее свойства. Измеримые множе-
ства, измеримость множеств внешней меры нуль и полупространств.
Теорема Каратеодори: 𝜎-алгебра измеримых множеств, мера Лебе-
га и ее счетная аддитивность. Непрерывность меры Лебега. Из-
меримость брусов, борелевских имножеств. Критерии измеримо-
сти множества: приближение борелевскими, приближение брусами.
Пример неизмеримого множества.
Определение 49. Брусом в R𝑛 называется множество вида 𝐵 = 𝐼1 × . . . × 𝐼𝑛 , где 𝐼𝑘 – ограничен-
ный промежуток. Если 𝑎𝑘 ⩽ 𝑏𝑘 – концы 𝐼𝑘 , то |𝐵| = (𝑏1 − 𝑎1 ) · . . . · (𝑏𝑛 − 𝑎𝑛 ) называется объемом
бруса 𝐵.
Если хотя бы один из промежутков 𝐼𝑘 вырожденный, то брус 𝐵 называется вырожденным, в
частности, ∅ — вырожденный брус. Объём вырожденного бруса равен 0.
Если все 𝐼𝑘 – отрезки, то брус называется замкнутым.
Если все 𝐼𝑘 – интервалы, то брус называется открытым.

Свойство 12. Если 𝐵, 𝐵1 , . . . , 𝐵𝑚 — брусы и 𝐵 ⊂ 𝑚


⋃︀ ∑︀𝑚
𝑖=1 𝐵𝑖 , то |𝐵| ⩽ 𝑖=1 |𝐵𝑖 |.

Доказательство. Если 𝐼 ⊂ R — ограниченный промежуток, то

|𝐼| − 1 ⩽ #(𝐼 ∩ Z) ⩽ |𝐼| + 1,


𝑁 |𝐼| − 1 ⩽ #(𝑁 𝐼 ∩ Z) ⩽ 𝑁 |𝐼| + 1,
(︂ )︂
1 1 1 1
|𝐼| − ⩽ # 𝐼 ∩ Z ⩽ |𝐼| + ,
𝑁 𝑁 𝑁 𝑁
(︂ )︂
1 1
|𝐼| = lim # 𝐼∩ Z .
𝑁 →∞ 𝑁 𝑁

Пусть 𝐵 = 𝐼1 × . . . × 𝐼𝑛 , тогда
𝑛 (︂ )︂ (︂ )︂
l 1 1 1 1 𝑛
|𝐵| = lim # 𝐼𝑗 ∩ Z = lim # 𝐵∩ Z .
𝑁 →∞ 𝑁 𝑁 𝑁 →∞ 𝑁 𝑛 𝑁
𝑗=1
⋃︀𝑚
Если 𝐵 ⊂ 𝑖=1 𝐵𝑖 , то
(︂ )︂ 𝑛 (︂ )︂
1 1 𝑛 1 ∑︁ 1 𝑛
# 𝐵∩ Z ⩽ 𝑛 # 𝐵𝑖 ∩ Z .
𝑁𝑛 𝑁 𝑁 𝑁
𝑖=1

Предельный переход 𝑁 → ∞ завершает доказательство.

Свойство 13. Для любого бруса 𝐵 и 𝜀 > 0 найдутся замкнутый брус 𝐵 ′ и открытый брус 𝐵 𝑜 ,
так что 𝐵 ′ ⊂ 𝐵 ⊂ 𝐵 𝑜 и |𝐵 ′ | > |𝐵| − 𝜀, |𝐵 𝑜 | < |𝐵| + 𝜀.

Доказательство. Пусть 𝐵 = 𝐼1 × . . . × 𝐼𝑛 , где 𝐼𝑘 — ограниченный промежуток с концами 𝑎𝑘 ⩽ 𝑏𝑘 .


Если |𝐵| > 0, то положим

𝐵𝛿′ = [𝑎1 + 𝛿, 𝑏1 − 𝛿] × . . . × [𝑎𝑛 + 𝛿, 𝑏𝑛 − 𝛿]


𝐵𝛿𝑜 = (𝑎1 − 𝛿, 𝑏1 + 𝛿) × . . . × (𝑎𝑛 − 𝛿, 𝑏𝑛 + 𝛿)

ФПМИ МФТИ, весна 2023 42


Математический анализ Назад к содержанию

Так как |𝐵𝛿′ |, |𝐵𝛿𝑜 | → |𝐵| при 𝛿 → +0, то искомые брусы существуют и определяются выбором
𝛿. Если же 𝐵 – вырожденный брус, то положим 𝐵 ′ = ∅, 𝐵𝛿𝑜 как выше.

Лемма 25. Каждое непустое открытое множество 𝑈 в R𝑛 представимо в виде счетного


объединения непересекающихся кубов (брусов, у которых длины ребер равны).
[︁ )︁
Доказательство. Куб 2𝑘𝑚1 ; 𝑘21𝑚 +1
× . . . × 2𝑘𝑚
[︀ 𝑛 𝑘𝑛 +1 )︀
; 2𝑚 , где 𝑘𝑖 ∈ Z, 𝑚 ⩾ 0, будем называть двоичным
𝑚-го ранга.
Обозначим через 𝐴0 множество всех кубов ранга 0, содержащихся в 𝑈 . Если множества
𝐴0 , . . . , 𝐴𝑚−1 уже определены, то обозначим через 𝐴𝑚 множество всех кубов ⋃︀ ранга 𝑚, содер-

жащихся в 𝑈 и не лежащих ни в одном кубе из 𝐴0 , . . . , 𝐴𝑚−1 . Положим ⋃︀ 𝐴 = 𝑚=0 𝐴𝑚 . Тогда
𝐴 – счетное множество непересекающихся кубов. Покажем, что 𝑈 = 𝑄∈𝐴 𝑄. Пусть 𝑥 ∈ 𝑈 .

Ввиду открытости 𝑈 существует шар 𝐵𝑟 (𝑥) ⊂ 𝑈 . Если 𝑚 таково, что 2𝑚𝑛 ⩽ 𝑟, то содержа-
щий точку 𝑥 куб 𝑄𝑚 (𝑥) ранга 𝑚 удовлетворяет включению 𝑄𝑚 (𝑥) ⊂ 𝐵𝑟 (𝑥) и, значит, множество
{𝑚 ∈ N0 : 𝑄𝑚 (𝑥) ⊂ 𝑈 } непусто. Обозначим через 𝑚0 его минимум.
⋃︀ Тогда 𝑄𝑚 (𝑥) ̸⊂ 𝑈 при 𝑚 < 𝑚0 ,
а 𝑄𝑚0 (𝑥) ⊂ 𝑈 . Следовательно, 𝑄𝑚0 (𝑥) ∈ 𝐴𝑚0 и поэтому 𝑥 ∈ 𝑄∈𝐴 𝑄. Учитывая, что обратное
включение очевидно, равенство установлено.

Определение 50. Семейство 𝒜 ⊂ 𝒫(R𝑛 ) называется алгеброй, если

1. ∅ ∈ 𝒜;

2. если 𝐸 ∈ 𝒜, то 𝐸 𝑐 = R𝑛 ∖ 𝐸 ∈ 𝒜;

3. если 𝐸, 𝐹 ∈ 𝒜, то 𝐸 ∪ 𝐹 ∈ 𝒜.

Алгебра 𝒜 называется 𝜎-алгеброй, если выполнено условие


⋃︀∞
3’. если 𝐸𝑘 ∈ 𝒜, 𝑘 ∈ N, то 𝑘=1 𝐸𝑘 ∈ 𝒜.

Пример.

1. 𝜎-алгебра, содержащая все одноэлементные множества, также содержит все не более чем
счетные множества и множества, дополнение к которым не более чем счетно.

2. ℬ(R𝑛 ) – минимальная по включению 𝜎-алгебра, содержащая все открытые множества (боре-


левская 𝜎-алгебра). Чтобы установить существование ℬ(R𝑛 ), необходимо рассмотреть пере-
сечение всех 𝜎-алгебр, содержащие открытые множества.

Определение 51. Внешней мерой Лебега множества 𝐸 ⊂ R𝑛 называется величина


{︃ ∞ ∞
}︃
∑︁ ⋃︁
𝜇* (𝐸) = inf |𝐵𝑖 | : 𝐸 ⊂ 𝐵𝑖 ,
𝑖=1 𝑖=1

где инфимум берется по всем счетным наборам {𝐵𝑖 }, покрывающих 𝐸.


Очевидно, 0 ⩽ 𝜇* (𝐸) ⩽ +∞.

Теорема 45. Внешняя мера обладает следующими свойствами

1. если 𝐸 ⊂ 𝐹 , то 𝜇* (𝐸) ⩽ 𝜇* (𝐹 ) (монотонность);

2. если 𝐸 = ∞
⋃︀ *
∑︀∞ *
𝑘=1 𝐸𝑘 , то 𝜇 (𝐸) ⩽ 𝑘=1 𝜇 (𝐸𝑘 ) (счетная полуаддитивность);

3. 𝜇* (𝑅) = |𝑅| для любого бруса 𝑅 (нормировка).

ФПМИ МФТИ, весна 2023 43


Математический анализ Назад к содержанию

Доказательство. Докажем пункт 2. Будем предполагать, что 𝜇* (𝐸) < +∞, иначе утверждение
очевидно. Зафиксируем 𝜀 > 0 и рассмотрим семейство брусов {𝐵𝑖,𝑘 }∞𝑖=1 , образующее покрытие
𝐸𝑘 , такие что

∑︁ 𝜀
|𝐵𝑖,𝑘 | < 𝜇* (𝐸𝑘 ) + 𝑘 .
2
𝑖=1
⋃︀∞
Семейство {𝐵𝑖,𝑘 }∞
𝑖,𝑘=1 образуют покрытие 𝐸 = 𝑘=1 𝐸𝑘 и

+∞ ∑︁
+∞ +∞ (︁ +∞
*
∑︁ ∑︁
* 𝜀 )︁ ∑︁ *
𝜇 (𝐸) ⩽ |𝐵𝑖,𝑘 | ⩽ 𝜇 (𝐸𝑘 ) + 𝑘 = 𝜇 (𝐸𝑘 ) + 𝜀.
2
𝑘=1 𝑖=1 𝑘=1 𝑘=1

Так как 𝜀 > 0 – любое, то пункт 2 установлен.


Докажем пункт 3. Так как {𝑅} – покрытие 𝑅 брусом, то 𝜇* (𝑅) ⩽ |𝑅|. Покажем, что 𝜇* (𝑅) ⩾
|𝑅|.
Сначала для случая, когда 𝑅 – замкнуто. Нам достаточно показать, что |𝑅| ⩽ ∞
∑︀
𝑖=1 |𝐵𝑖 | для
всякого покрытия 𝑅 брусами 𝐵𝑖 . Зафиксируем 𝜀 > 0. Тогда по свойству брусов (13) ∃ 𝐵𝑖𝑜 ⊃ 𝐵𝑖
⏟ ⏞
отк. брус
и |𝐵𝑖𝑜 | < |𝐵𝑖 | + 2𝜀𝑖 . Так как 𝑅 ⊂ ∞
⋃︀
𝑖=1 и 𝑅 – компакт, то по свойству брусов (12)

𝑁 𝑁 ∞ (︁ ∞
⋃︁ ∑︁ ∑︁ 𝜀 )︁ ∑︁
𝑅⊂ 𝐵𝑖𝑜 ⇒ |𝑅| ⩽ |𝐵𝑖𝑜 | ⇒ |𝑅| ⩽ |𝐵𝑖 | + = |𝐵𝑖 | + 𝜀.
2𝑖
𝑖=1 𝑖=1 𝑖=1 𝑖=1

Так как 𝜀 > 0 – любое, то |𝑅| ⩽ ∞


∑︀
𝑖=1 |𝐵𝑖 |.

Пусть 𝑅 – произвольный брус. Тогда для 𝜀 > 0 по свойству (13) ∃ ⏟𝑅⏞ ⊂ 𝑅 (|𝑅′ | > |𝑅| − 𝜀).
замк. брус
Тогда
𝜇* (𝑅) ⩾ 𝜇* (𝑅′ ) = |𝑅′ | > |𝑅| − 𝜀.
Так как 𝜀 > 0 – любое, то 𝜇* (𝑅) ⩾ |𝑅|.

Определение 52. Множество 𝐸 ⊂ R𝑛 называется измеримым (по Лебегу), если для любого
𝐴 ⊂ R𝑛 : 𝜇* (𝐴) = 𝜇* (𝐴 ∩ 𝐸) + 𝜇* (𝐴 ∩ 𝐸 𝑐 ).

Пример. Если 𝜇* (𝐸) = 0, то 𝐸 измеримо.


Действительно, 𝜇* (𝐴 ∩ 𝐸) ⩽ 𝜇* (𝐸) = 0, 𝜇* (𝐴 ∩ 𝐸) ⩽ 𝜇* (𝐴) из монотонности 𝜇* . Тогда 𝜇* (𝐴) ⩾
𝜇 (𝐴 ∩ 𝐸) + 𝜇* (𝐴 ∩ 𝐸 𝑐 ).
*

Пример. Для всякого 𝑎 ∈ R и 𝑘 ∈ {1, . . . , 𝑛} полупространство 𝐻 = 𝐻𝑎,𝑘 = {𝑥 = (𝑥1 , . . . , 𝑥𝑛 )𝑇 :


𝑥𝑘 < 𝑎} измеримо.
Рассмотрим 𝐴 ⊂ R𝑛 и произвольное покрытие {𝐵𝑖 }∞ 𝑖=1 . Брусами определим

𝐵𝑖1 = 𝐵𝑖 ∩ 𝐻, 𝐵𝑖2 = 𝐵𝑖 ∩ 𝐻 𝑐 .

Тогда 𝐵𝑖1 , 𝐵𝑖2 – брусы. {𝐵𝑖1 ∩ 𝐻}∞ 2 𝑐 ∞ 𝑐


𝑖=1 – покрытие 𝐴 ∩ 𝐻. {𝐵𝑖 ∩ 𝐻 }𝑖=1 – покрытие 𝐴 ∩ 𝐻 .


∑︁ ∞
∑︁ ∞
∑︁
|𝐵𝑖 | = |𝐵𝑖1 | + |𝐵𝑖2 | ⩾ 𝜇* (𝐴 ∩ 𝐻) + 𝜇* (𝐴 ∩ 𝐻 𝑐 ).
𝑖=1 𝑖=1 𝑖=1

Следовательно, 𝜇* (𝐴) ⩾ 𝜇* (𝐴 ∩ 𝐻) + 𝜇* (𝐴 ∩ 𝐻 𝑐 ).
Аналогичное утверждение верно и для других неравенств между 𝑥𝑘 и 𝑎.

Теорема 46 (Каратеодори). Совокупность ℳ всех измеримых множеств в R𝑛 образует 𝜎-


алгебру. Сужение 𝜇* |ℳ счетно аддитивно.

ФПМИ МФТИ, весна 2023 44


Математический анализ Назад к содержанию

Доказательство. ∅ ∈ ℳ, 𝐸 ∈ ℳ ⇒ 𝐸 𝑐 ∈ ℳ.
1. Пусть 𝐸, 𝐹 ∈ ℳ. Покажем, что 𝐸 ∪ 𝐹 ∈ ℳ.
Пусть 𝐴 ⊂ R𝑛 , тогда

𝜇* (𝐴∩(𝐸 ∪𝐹 ))+𝜇* (𝐴∩(𝐸 ∪𝐹 )𝑐 ) = 𝜇* (𝐴∩(𝐸 ∪𝐹 )∩𝐸)+𝜇* (𝐴∩(𝐸 ∪𝐹 )∩𝐸 𝑐 )+𝜇* (𝐴∩(𝐸 ∪𝐹 )𝑐 ) =

= 𝜇* (𝐴 ∩ 𝐸) + 𝜇* (𝐴 ∩ 𝐸 𝑐 ∩ 𝐹 ) + 𝜇* (𝐴 ∩ 𝐸 𝑐 ∩ 𝐹 𝑐 ) = 𝜇* (𝐴 ∩ 𝐸) + 𝜇* (𝐴 ∩ 𝐸 𝑐 ) = 𝜇* (𝐴).

2. Пусть {𝐸𝑘 } ⊂ ℳ, причем 𝐸𝑖 ∩ 𝐸𝑗 = ∅ при 𝑖 ̸= 𝑗. Покажем, что 𝐹 = ∞


⋃︀
𝑘=1 𝐸𝑘 ∈ ℳ.
⋃︀𝑛
Положим 𝐹𝑛 = 𝑘=1 𝐸𝑘 . Если 𝐴 ⊂ 𝑋, то

𝜇* (𝐴 ∩ 𝐹𝑛 ) = 𝜇* (𝐴 ∩ 𝐹𝑛 ∩ 𝐸𝑛 ) + 𝜇* (𝐴 ∩ 𝐹𝑛 ∩ 𝐸𝑛𝑐 ) = 𝜇* (𝐴 ∩ 𝐸𝑛 ) + 𝜇* (𝐴 ∩ 𝐹𝑛−1 ).

Продолжая процесс, получим 𝜇* (𝐴 ∩ 𝐹𝑛 ) = 𝑛𝑘=1 𝜇* (𝐴 ∩ 𝐸𝑘 ).


∑︀

Поскольку 𝐹𝑛 ∈ ℳ, то
𝑛
∑︁
* * *
𝜇 (𝐴) = 𝜇 (𝐴 ∩ 𝐹𝑛 ) + 𝜇 (𝐴 ∩ 𝐹𝑛𝑐 ) ⩾ 𝜇* (𝐴 ∩ 𝐸𝑘 ) + 𝜇* (𝐴 ∩ 𝐹 𝑐 ).
𝑘=1
∑︀∞
Переходя к пределу при 𝑛 → ∞, получим 𝜇* (𝐴) ⩾ 𝑘=1 𝜇
* (𝐴 ∩ 𝐸𝑘 ) + 𝜇* (𝐴 ∩ 𝐹 𝑐 ). Откуда по
свойству счетной полуаддитивности

∑︁
𝜇* (𝐴) ⩾ 𝜇* (𝐴 ∩ 𝐸𝑘 ) + 𝜇* (𝐴 ∩ 𝐹𝑐 ) ⩾ 𝜇* (𝐴 ∩ 𝐹 ) + 𝜇* (𝐴 ∩ 𝐹 𝑐 ) ⩾ 𝜇* (𝐴).
𝑘=1

Это доказывает, что 𝐹 ∈ ℳ. Если еще положить 𝐴 = 𝐹 , то 𝜇* (𝐹 ) = ∞ *


∑︀
𝑘=1 𝜇 (𝐸𝑘 ).

3. Пусть {𝐴𝑘 } ⊂ ℳ. Покажем, что 𝐴 = ∞


⋃︀
𝑘=1 𝐴𝑘 ∈ ℳ.
Положим 𝐸1 = 𝐴1 , 𝐸𝑘 = 𝐴𝑘 ∖ 𝑖<𝑘 𝐸𝑖 . Тогда 𝐸𝑘 попарно не пересекаются, и 𝐴 = ∞
⋃︀ ⋃︀
𝑘=1 𝐸𝑘 ∈ ℳ
по предыдущему пункту.

Определение 53. 𝜇 = 𝜇* |ℳ – мера Лебега.


Теорема 47 (непрерывность меры). 1. 𝐴𝑖 ∈ ℳ, 𝐴1 ⊂ 𝐴2 ⊂ . . ., 𝐴 = ∞
⋃︀
𝑖=1 𝐴𝑖 . Тогда 𝜇(𝐴) =
lim𝑖→∞ 𝜇(𝐴𝑖 ) (непрерывность снизу).
2. 𝐴𝑖 ∈ ℳ, 𝐴1 ⊃ 𝐴2 ⊃ . . ., 𝐴 = ∞
⋂︀
𝑖=1 𝐴𝑖 , 𝜇(𝐴1 ) < ∞. Тогда 𝜇(𝐴) = lim𝑖→∞ 𝜇(𝐴𝑖 ) (непрерывность
сверху).

⋃︀𝑚 1. Положим 𝐵1 = 𝐴1 , 𝐵𝑖 = 𝐴𝑖 ∖ 𝐴𝑖−1 . Тогда 𝐵𝑖 ∈ ℳ, 𝐵𝑖 ∩ 𝐵𝑗 = ∅ при 𝑖 ̸= 𝑗, и


Доказательство.
⋃︀𝑚
𝑖=1 𝐵𝑖 = 𝑖=1 𝐴𝑖 для всех 𝑚 ∈ N ∪ ∞. Поэтому
(︃ ∞ )︃ ∞ 𝑚
⋃︁ ∑︁ ∑︁
𝜇(𝐴) = 𝜇 𝐵𝑖 = 𝜇(𝐵𝑖 ) = lim 𝜇(𝐵𝑖 ) = lim 𝜇(𝐴𝑚 ).
𝑚→∞ 𝑚→∞
𝑖=1 𝑖=1 𝑖=1

2. Рассмотрим 𝐴1 ∖𝐴𝑖 . Применим прошлый пункт к этим множествам. Тогда ∞


⋃︀
𝑖=1 (𝐴∖𝐴𝑖 ) = 𝐴1 ∖𝐴
и
𝜇(𝐴1 ) − 𝜇(𝐴) = 𝜇(𝐴1 ∖ 𝐴) = lim 𝜇(𝐴1 ∖ 𝐴𝑚 ) = 𝜇(𝐴1 ) − lim 𝜇(𝐴𝑚 ).
𝑚→∞ 𝑚→∞

Осталось из обоих частей вычесть 𝜇(𝐴1 ) и изменить знак.

ФПМИ МФТИ, весна 2023 45


Математический анализ Назад к содержанию

Лемма 26. ℬ(R𝑛 ) ⊂ ℳ.


Доказательство. Брус измерим, так как его можно записать в виде пересечения конечного числа
подпространств (измеримы по примеру ). По лемме 25 тогда всякое открытое множество изме-
римо.

Определение 54. Счетное пересечение открытых множеств называется множествами типа 𝐺𝛿 .


Счетное объединение замкнутых множеств называется множествами типа 𝐹𝜎 .
Замечание. Множества типа 𝐺𝛿 и 𝐹𝜎 являются борелевскими.
Теорема 48 (критерий измеримости). Множество 𝐸 измеримо ⇔ существует множество Ω
типа 𝐺𝛿 , что 𝐸 ⊂ Ω и 𝜇(Ω ∖ 𝐸) = 0.
Доказательство. Докажем первое утверждение.
⋂︀∞(⇒) Для каждого 𝑘 ∈ N найдется такое открытое 𝐺𝑘 ⊃ 𝐸 с 𝜇(𝐺𝑘 ∖ 𝐸) ⩽ 𝑘1 . Положим Ω =
1
𝑘=1 𝐺𝑘 , тогда 𝐸 ⊂ Ω, и 𝜇(Ω ∖ 𝐸) ⩽ 𝜇(𝐺𝑘 ∖ 𝐸) ⩽ 𝑘 , откуда 𝜇(Ω ∖ 𝐸) = 0.
(⇐) Поскольку 𝐸 = Ω ∖ (Ω ∖ 𝐸) есть разность двух измеримых множеств, то 𝐸 измеримо.

Замечание. Множество 𝐸 измеримо ⇔ существует множество ∆ типа 𝐹𝜎 , что ∆ ⊂ 𝐸 и 𝜇(𝐸∖∆) =


0.
Теорема 49 (критерий измеримости). Пусть 𝜇* (𝐸)⋃︀< ∞. Множество 𝐸 измеримо ⇔ суще-
ствуют брусы 𝐵1 , . . . , 𝐵𝑁 , такие что ∀𝜀 > 0 𝜇* (𝐸△ 𝑁
𝑘=1 𝐵𝑘 ) < 𝜀.

Доказательство. (⇒) Зафиксируем 𝜀 > 0. Если 𝐸 измеримо, то ∃ {𝐵𝑘 }∞ , такие что 𝐸 ⊂


⏟ ⏞𝑘=1
брусы
⋃︀∞ ∑︀+∞ * (𝐸) + 𝜀 . Так как ряд
∑︀+∞ (︀∑︀+∞ 𝜀
)︀
𝐵
𝑘=1 𝑘 и 𝑘=1 |𝐵𝑘 | < 𝜇 2 𝑘=1 |𝐵 𝑘 | сходится, то ∃𝑁 𝑘=𝑁 +1 |𝐵𝑘 | < 2 .
Положим 𝐶 = 𝑁
⋃︀
𝑘=1 𝐵𝑘 . Тогда:
(︃ +∞ )︃ (︃+∞ )︃
⋃︁ ⋃︁
𝜇* (𝐸△𝐶) ⩽ 𝜇* (𝐸 ∖ 𝐶) + 𝜇* (𝐶 ∖ 𝐸) ⩽ 𝜇* 𝐵𝑘 + 𝜇* 𝐵𝑘 ∖ 𝐸 ⩽
𝑘=𝑁 +1 𝑘=1

+∞ +∞
∑︁ ∑︁ 𝜀 𝜀
⩽ |𝐵𝑘 | + |𝐵𝑘 | − 𝜇* (𝐸) < + = 𝜀.
2 2
𝑘=𝑁 +1 𝑘=1

(⇐) Пусть 𝜇* (𝐸△𝐶) < 𝜀. Тогда тем более 𝜇* (𝐸 ∖ 𝐶) < 𝜀 и 𝜇* (𝐶 ∖ 𝐸) < 𝜀. Поскольку 𝐸 ⊂
𝐶 ∪ (𝐸 ∖ 𝐶) и 𝐸 𝑐 ⊂ 𝐶 𝑐 ∪ (𝐶 ∖ 𝐸), то для любого 𝐴 ⊂ R𝑛 имеем

𝜇* (𝐴 ∩ 𝐸) + 𝜇* (𝐴 ∩ 𝐸 𝑐 ) ⩽ 𝜇* (𝐴 ∩ 𝐶) + 𝜇* (𝐴 ∩ (𝐸 ∖ 𝐶)) + 𝜇* (𝐴 ∩ 𝐶 𝑐 ) + 𝜇* (𝐴 ∩ (𝐶 ∖ 𝐸)) ⩽

⩽ 𝜇* (𝐴 ∩ 𝐶) + 𝜇* (𝐴 ∩ 𝐶 𝑐 ) + 𝜇* (𝐸 ∖ 𝐶) + 𝜇* (𝐶 ∖ 𝐸) < 𝜇* (𝐴) + 2𝜀.


Следовательно, 𝜇* (𝐴 ∩ 𝐸) + 𝜇* (𝐴 ∩ 𝐸 𝑐 ) ⩽ 𝜇* (𝐴). Значит, 𝐸 измеримо.

Построим пример неизмеримого множества.


Пример ⨆︀ (множество Витали). На [0, 1] введём отношение эквивалентности 𝑥 ∼ 𝑦 ⇔ 𝑥 − 𝑦 ∈ Q ⇒
[0, 1] = 𝛼 𝐻𝛼 , 𝐻𝛼 – классы эквивалетности.
𝑉 – множество, содержащее ровно один элемент из каждого 𝐻𝛼 и только такие элементы
(такое множество существует по аксиоме выбора).
Пусть {𝑟𝑛 }+∞
𝑛=0 – некоторая нумерация Q ∩ [−1, 1], 𝑟0 = 0. Рассмотрим 𝑉𝑛 = 𝑉 + 𝑟𝑛 .

1. 𝑉𝑛 попарно не пересекаются, так как

𝑥 ∈ 𝑉𝑖 ∩ 𝑉𝑗 ⇒ 𝑥𝑖 + 𝑟𝑖 = 𝑥𝑗 + 𝑟𝑗 ⇒ 𝑥𝑗 − 𝑥𝑖 ∈ Q.

ФПМИ МФТИ, весна 2023 46


Математический анализ Назад к содержанию

⨆︀∞
2. [0, 1] ⊂ 𝑛=0 𝑉𝑛 ⊂ [−1, 2].
(левое включение: 𝑦 ∈ [0, 1] ⇒ 𝑦 ∈ 𝐻𝛼 ⇒ 𝑦 = 𝑥𝛼 + 𝑟, 𝑟 = 𝑦 − 𝑥𝛼∈[−1,1] ⇒ ∃𝑛(𝑟 − 𝑟𝑛 ))
(правое включение: 𝑉 ⊂ [0, 1] и 𝑟𝑛 ∈ [−1, 1])

3. Пусть 𝐴𝑛 ⊂ 𝑉𝑛 ⇒ 𝜇(𝐴𝑛 ) = 0.
⏟ ⏞
изм.
(𝐴𝑚 = 𝐴𝑛 − 𝑟𝑛 + 𝑟𝑚 ⊂ 𝑉𝑚 , 𝜇(𝐴𝑚 ) = 𝜇(𝐴𝑛 ) = 0, 𝜇( ∞
⨆︀ ∑︀∞ ∑︀∞
𝑛=0 𝐴𝑛 ) = 𝑛=0 𝜇(𝐴𝑛 ) = 𝑛=0 𝑎 ⩽
𝜇([−1, 2]) = 3 ⇒ 𝑎 = 0)

ФПМИ МФТИ, весна 2023 47


Математический анализ Назад к содержанию

9. Измеримые функции. Согласованность измеримости функций


с арифметическими операциями. Измеримость точных граней и
предела последовательности измеримых функций. Сходимость по-
чти всюду. Простые функции. Теорема о приближении измеримой
функции простыми.
Пусть 𝐸 измеримо и 𝑓 : 𝐸 → R.

Определение 55. Функция 𝑓 называется измеримой (по Лебегу), если {𝑥 ∈ 𝐸 : 𝑓 (𝑥) < 𝑎} =
𝑓 −1 ([−∞, 𝑎)) измеримо для всех 𝑎 ∈ R.

Лемма 27. Пусть 𝑓 : 𝐸 → R. Тогда следующие условия эквивалентны:

1. 𝑓 измеримо;

2. 𝑓 −1 (𝑈 ) измеримо для любого открытого 𝑈 в R;

3. 𝑓 −1 (Ω) для любого борелевского Ω в R.

Доказательство. Рассмотрим 𝒜 = {𝐴 ∈ 𝐵(R) : 𝑓 −1 (𝐴) измеримо}. Так как ∅ ∈ 𝒜, 𝐸 ∖ 𝑓 −1 (𝐴) =


∞ ∞ ∞
𝑓 −1 (R ∖ 𝐴) ⇒ (𝐴 ∈ 𝒜 ⇒ R ∖ 𝐴 ∈ 𝒜) и 𝑓 −1 ( 𝑖=1 𝐴𝑖 ) = 𝑖=1 𝑓 −1 (𝐴𝑖 ) ⇒ 𝑖=1 𝐴𝑖 ∈ 𝒜, то 𝒜 образует
⋃︀ ⋃︀ ⋃︀
𝜎-алгебру. 𝒜 содержит все лучи [−∞, 𝑎). Следовательно, 𝐵(R) = 𝒜, то есть (1 ⇒ 3).
Импликации (3 ⇒ 2 ⇒ 1) очевидны.

Теорема 50. Если 𝑓, 𝑔 : 𝐸 → R измеримые и 𝜆 ∈ R, то 𝑓 + 𝑔, 𝜆𝑓, |𝑓 |, 𝑓 𝑔 также измеримы.

Доказательство. 1. Докажем измеримость суммы. Поскольку 𝛼 < 𝛽 ⇔ ∃𝑟 ∈ Q(𝛼 < 𝑟 < 𝛽),
Q = {𝑟𝑘 }∞
𝑘=1 , то


⋃︁
{𝑥 ∈ 𝐸 : 𝑓 (𝑥) + 𝑔(𝑥) < 𝑎} = {𝑥 ∈ 𝐸 : 𝑓 (𝑥) < 𝑎 − 𝑔(𝑥)} = {𝑥 ∈ 𝐸 : 𝑓 (𝑥) < 𝑟𝑘 < 𝑎 − 𝑔(𝑥)} =
𝑘=1


⋃︁
= {𝑥 ∈ 𝐸 : 𝑓 (𝑥) < 𝑟𝑟 } ∩ {𝑥 ∈ 𝐸 : 𝑔(𝑥) < 𝑎 − 𝑟𝑘 }.
𝑘=1

Следовательно, {𝑥 ∈ 𝐸 : 𝑓 (𝑥) + 𝑔(𝑥) < 𝑎} измеримо.

2. Пусть 𝜆 > 0, тогда {𝑥 ∈ 𝐸 : 𝜆𝑓 (𝑥) < 𝑎} = {𝑥 ∈ 𝐸 : 𝑓 (𝑥) < 𝜆𝑎 } измеримо.


Если 𝜆 = 0, то тривиально. Если 𝜆 < 0, то аналогично.
√ √
3. Так как {𝑥 ∈ 𝐸 : 𝑓 2 (𝑥) < 𝑎} = {𝑥 ∈ 𝐸 : 𝑓 (𝑥) < 𝑎} ∩ {𝑥 ∈ 𝐸 : 𝑓 (𝑥) > − 𝑎} измеримо ∀𝑎 > 0.
Если 𝑎 ⩽ 0, то {𝑥 ∈ 𝐸 : 𝑓 2 (𝑥) < 𝑎} = ∅ – измеримо.
Следовательно, 𝑓 2 – измеримая функция. Аналогично для |𝑓 |.
Так как 𝑓 𝑔 = 12 (𝑓 + 𝑔)2 − 𝑓 2 − 𝑔 2 , то 𝑓 𝑔 измерима.
(︀ )︀

Теорема 51. Если 𝑓𝑘 : 𝐸 → R – измеримы, то sup𝑓𝑘 , inf 𝑓𝑘 , lim𝑘→+∞ 𝑓𝑘 , lim𝑘→+∞ 𝑓𝑘 также


𝑘 𝑘
измеримы на 𝐸.

ФПМИ МФТИ, весна 2023 48


Математический анализ Назад к содержанию

Доказательство. Измеримость 𝑔 = sup𝑓𝑘 следует из равенства:


𝑘

+∞
⋂︁
{𝑥 ∈ 𝐸 : 𝑔(𝑥) ⩽ 𝑎} = {𝑥 ∈ 𝐸 : 𝑓𝑘 (𝑥) ⩽ 𝑎}
𝑘=1

Измеримость ℎ = inf 𝑓𝑘 следует из inf 𝑓𝑘 = −sup(−𝑓𝑘 ).


𝑘 𝑘 𝑘
Далее, поскольку lim𝑘→+∞ 𝑓𝑘 = inf sup 𝑓𝑚 , lim𝑘→+∞ 𝑓𝑘 = sup inf 𝑓𝑚 , то оба предела измеримы.
𝑘 𝑚⩾𝑘 𝑘 𝑚⩾𝑘

Следствие 24. Если 𝑓𝑘 : 𝐸 → R измеримы, и 𝑓 (𝑥) = lim𝑘→+∞ 𝑓𝑘 (𝑥) для всех 𝑥 ∈ 𝐸, то 𝑓


измерима на 𝐸.

Доказательство. Вытекает из предыдущей теоремы, но докажем непосредственно.


Имеем 𝑓 (𝑥) < 𝑎 ⇔ ∃𝑗 ∈ N ∃𝑁 ∀𝑘 ⩾ 𝑁 (𝑓𝑘 (𝑥) < 𝑎 − 1𝑗 ).
{𝑥 : 𝑓 (𝑥) < 𝑎} = +∞
⋃︀ ⋃︀+∞ ⋂︀+∞ 1
𝑗=1 𝑁 =1 𝑘=𝑁 {𝑥 : 𝑓𝑘 (𝑥) < 𝑎− 𝑗 } – измеримо как операции над измеримыми
множествами.

Определение 56. Пусть 𝐸 ⊂ R𝑛 , 𝑄 – формула на 𝐸.


Говорят, что 𝑄 верна почти везде на 𝐸, если 𝜇(𝑥 ∈ 𝐸 : 𝑄(𝑥) ложно) = 0.

Лемма 28. Пусть 𝑓, 𝑔 : 𝐸 → R. Если 𝑓 = 𝑔 почти везде и 𝑓 измерима, то 𝑔 измерима.

Доказательство. По условию, 𝑍 = {𝑥 ∈ 𝐸 : 𝑓 (𝑥) ̸= 𝑔(𝑥)} имеет меру нуль. Тогда для любого
𝑎 ∈ R имеем {𝑥 ∈ 𝐸 : 𝑔(𝑥) < 𝑎} = ({𝑥 ∈ 𝐸 : 𝑓 (𝑥) < 𝑎} ∩ 𝑍 𝑐 ) ∪ ({𝑥 ∈ 𝐸 : 𝑔(𝑥) < 𝑎} ∩ 𝑍) –
измеримо.

Следствие 25. Если 𝑓𝑘 : 𝐸 → R измеримы и 𝑓𝑘 → 𝑓 почти везде на 𝐸, где 𝑓 : 𝐸 → R, то 𝑓


измерима.

Доказательство. 𝑔 = lim𝑘→+∞ 𝑓𝑘 измерима на 𝐸, 𝑓 = 𝑔 почти везде на 𝐸, значит 𝑓 измерима


(по лемме).

Определение 57. Функция 𝜙 : R𝑛 → R называется простой, если 𝜙 измерима и множество её


значений конечно.

Пример. Пусть 𝐴 ⊂ R𝑛 , Определим индикатор (характеристическую функцию) 𝐴:


{︃
1, 𝑥 ∈ 𝐴;
I𝐴 : R𝑛 → R𝑛 , I𝐴 (𝑥) =
0, 𝑥 ̸∈ 𝐴.

Поскольку {𝑥 : I𝐴 (𝑥) < 𝑎} пусто при 𝑎 ⩽ 0, совпадает с 𝐴𝑐 при 𝑎 ∈ (0, 1] и совпадает с R𝑛 при
𝑎 > 1, то функция I𝐴 является измеримой ⇔ 𝐴 измеримо.

Теорема 52. Если 𝑓 : 𝐸 → [0, +∞] – неотрицательная измеримая функция, то существует


последовательность {𝜙𝑘 } неотрицательных простых функций, таких что ∀𝑥 ∈ 𝐸 выполняется

1. 0 ⩽ 𝜙1 (𝑥) ⩽ 𝜙2 (𝑥) ⩽ . . .

2. lim𝑘→+∞ 𝜙𝑘 (𝑥) = 𝑓 (𝑥)

ФПМИ МФТИ, весна 2023 49


Математический анализ Назад к содержанию

Доказательство. Для 𝑘 ∈ N определим множества:


{︂ }︂
𝑗−1 𝑗
𝐸𝑘,𝑗 = 𝑥 ∈ 𝐸 : 𝑘 ⩽ 𝑓 (𝑥) < 𝑘 , 𝑗 = 1, . . . , 𝑘 · 2𝑘 ,
2 2

𝐹𝑘 = {𝑥 ∈ 𝐸 : 𝑓 (𝑥) ⩾ 𝑘}.
Множества 𝐸𝑘,𝑗 и 𝐹𝑘 измеримы и в объединении дают 𝐸.
∑︀𝑘·2𝑘 𝑗−1
Определим 𝜙𝑘 = 𝑗=1 2𝑘 I𝐸𝑘,𝑗 + 𝑘 · I𝐹𝑘 . Пусть 𝑥 ∈ 𝐸. Покажем, что {𝜙𝑘 (𝑥)}, возрастая,
стремится к 𝑓 (𝑥).
Если 𝑓 (𝑥) = +∞, то 𝜙𝑘 (𝑥) = 𝑘 для всех 𝑘 и утверждение верно.
Пусть 𝑓 (𝑥) ∈ R и 𝑘 ∈ N. Если 𝑓 (𝑥) ⩾ 𝑘+1, то 𝜙𝑘+1 (𝑥) = 𝑘+1 > 𝑘 = 𝜙𝑘 (𝑥). Если 𝑘 ⩽ 𝑓 (𝑥) < 𝑘+1,
то 𝜙𝑘+1 (𝑥) ⩾ 𝑘 = 𝜙𝑘 (𝑥).
Пусть 𝑓 (𝑥) < 𝑘, тогда 𝑗−1
2𝑘
⩽ 𝑓 (𝑥) < 2𝑗𝑘 для некоторого 𝑗, 1 ⩽ 𝑗 ⩽ 𝑘·2𝑘 . Возможны два варианта:
2𝑗−2
2𝑘+1
⩽ 𝑓 (𝑥) < 2𝑗−1
2𝑘+1
или 2𝑗−1
2𝑘+1
2𝑗
⩽ 𝑓 (𝑥) < 2𝑘+1 . В обоих случаях 𝜙𝑘+1 (𝑥) ⩾ 2𝑗−2
2𝑘+1
= 𝑗−1
2𝑘
= 𝜙𝑘 (𝑥) и
−𝑘
возрастание установлено. Кроме того, 0 ⩽ 𝑓 (𝑥) − 𝜙𝑘 (𝑥) < 2 при всех 𝑘 ⩾ [𝑓 (𝑥)] + 1, откуда
следует, что 𝜙𝑘 (𝑥) → 𝑓 (𝑥).

ФПМИ МФТИ, весна 2023 50


Математический анализ Назад к содержанию

10. Интеграл от неотрицательной простой функции и его свойства.


Интеграл от неотрицательной измеримой функции. Монотонность
интеграла по функциям и по множествам. Теорема Леви о монотон-
ной сходимости. Аддитивность интеграла по функциям. Счетная
аддитивность интеграла по множествам. Неравенство Чебышева.
Интеграл Лебега от произвольной измеримой функции. Интегри-
руемые функции. Одновременная интегрируемость функции и ее
модуля. Конечность почти всюду интегрируемой функции. Прене-
брежение при интегрировании множествами меры нуль. Монотон-
ность и линейность интеграла. Теорема Лебега о мажорированной
сходимости. Связь интеграла Лебега и определенного интеграла
Римана. Формула суммирования Эйлера (б/д). Формула Стирлин-
га.
Определение 58. Пусть 𝜙 – простая функция, 𝜙 = 𝑚 𝑚
∑︀
𝑖=1 𝑎𝑖 I𝐴𝑖 , где {𝐴𝑖 }𝑖=1 – допустимое разло-
жение.
Интегралом от 𝜙 по измеримому множеству 𝐸 называется
∫︁ 𝑚
∑︁
𝜙𝑑𝜇 = 𝑎𝑖 𝜇(𝐸 ∩ 𝐴𝑖 ).
𝐸 𝑖=1

Лемма 29. Пусть 𝜙, 𝜓 – простые функции. Тогда:


∫︀ ∫︀
1. Если 𝜙 ⩽ 𝜓 на 𝐸, то 𝐸 𝜙𝑑𝜇 ⩽ 𝐸 𝜓𝑑𝜇 (монотонность).
∫︀ ∫︀
2. Если 𝛼 ∈ [0, +∞), то 𝐸 𝛼𝜙𝑑𝜇 = 𝛼 𝐸 𝜙𝑑𝜇 (положительная однородность).
∫︀ ∫︀ ∫︀
3. 𝐸 (𝜙 + 𝜓)𝑑𝜇 = 𝐸 𝜙𝑑𝜇 + 𝐸 𝜓𝑑𝜇 (аддитивность по функциям).
Доказательство. Пусть {𝐴𝑖 }𝑚 𝑘
𝑖=1 , {𝐵𝑗 }𝑗=1 – допустимые разбиения 𝜙 и 𝜓 соответственно (𝜙|𝐴𝑖 =
𝑎𝑖 , 𝜙|𝐵𝑗 = 𝑏𝑗 ). Положим 𝐶𝑖𝑗 = 𝐴𝑖 ∩ 𝐵𝑗 .
Тогда {𝐶𝑖𝑗 } – общее допустимое разбиение для 𝜙 и 𝜓. Поскольку 𝐴𝑖 = 𝐴𝑖 ∩R𝑛 = 𝐴𝑖 ∩( 𝑘𝑗=1 𝐵𝑗 ) =
⋃︀
⋃︀𝑘 ∫︀ ∑︀𝑚 ∑︀𝑚 ⋃︀𝑘
𝑗=1 𝐶𝑖𝑗 , то по свойству аддитивности меры 𝐸 𝜙𝑑𝜇 = 𝑖=1 𝑎𝑖 𝜇(𝐸 ∩ 𝐴𝑖 ) = 𝑖=1 𝑎𝑖 𝜇( 𝑗=1 (𝐸 ∩
𝐶𝑖𝑗 )) = 𝑚
∑︀ ∑︀𝑘
𝑖=1 𝑗=1 𝑎𝑖 𝜇(𝐸 ∩ 𝐶𝑖𝑗 ).
Аналогично, 𝐸 𝜓𝑑𝜇 = 𝑘𝑗=1 𝑚
∫︀ ∑︀ ∑︀
𝑖=1 𝑏𝑗 𝜇(𝐸 ∩ 𝐶𝑖𝑗 ). Если 𝐸 ∩ 𝐶𝑖𝑗 ̸= ∅, то для любого 𝑥 ∈ 𝐸 ∩ 𝐶𝑖𝑗
имеем 𝑎𝑖 = 𝜙(𝑥) ⩽ 𝜓(𝑥) = 𝑏𝑗 , что завершает доказательство.
Доказательство пункта 2 очевидно.
Доказательство пункта 3 аналогично пункту 1.

Определение 59. Пусть 𝑓 : 𝐸 → [0, +∞] – неотрицательная измеримая функция. Тогда:


∫︁ {︂∫︁ }︂
𝑓 𝑑𝜇 = sup 𝜙𝑑𝜇, 0 ⩽ 𝜙 ⩽ 𝑓, 𝜙 – простая .
𝐸 𝐸

Замечание. Покажем, что определение согласуется с интегралом от простой функции. Чтобы


их различить, перед знаком введенного ранее интеграла поставим (𝑠).
Пусть 𝑓 – простая
∫︀ неотрицательная
∫︀ функция. Если 0 ⩽ 𝜙 ⩽ 𝑓 и 𝜙 – простая,
∫︀ то по свойству
∫︀
монотонности (𝑠) 𝐸 𝜙𝑑𝜇 ⩽ (𝑠) 𝐸 𝑓 𝑑𝜇. Переходя к супремуму по 𝜙, получим 𝐸 𝜙𝑑𝜇 ⩽ (𝑠) 𝐸 𝑓 𝑑𝜇.
Противоположное неравенство очевидно, так как 𝑓 сама является простой функцией.

ФПМИ МФТИ, весна 2023 51


Математический анализ Назад к содержанию

Пусть 𝑓, 𝑔 : 𝐸 → [0, +∞] — неотрицательные измеримые функции.


∫︀ ∫︀
Свойство 14 (монотонность). Если 𝑓 ⩽ 𝑔 на 𝐸, то 𝐸 𝑓 𝑑𝜇 ⩽ 𝐸 𝑔 𝑑𝜇.
∫︀ ∫︀
Свойство 15 (однородность). Если 𝜆 ∈ [0, +∞), то 𝐸 𝜆𝑓 𝑑𝜇 = 𝜆 𝐸 𝑓 𝑑𝜇.
∫︀ ∫︀
Свойство 16. Если 𝐸0 ⊂ 𝐸 измеримо, то 𝐸0 𝑓 𝑑𝜇 = 𝐸 𝑓 · I𝐸0 𝑑𝜇.

Доказательство. Пусть 0 ⩽ 𝜙 ⩽ 𝑓 на 𝐸0 , тогда


⏟ ⏞
прост.
∫︁ ∫︁ ∫︁
𝜙 𝑑𝜇 = 𝜙 · I𝐸0 𝑑𝜇 ⩽ 𝑓 · I𝐸0 𝑑𝜇,
𝐸0 𝐸 𝐸
∫︁ ∫︁
𝑓 𝑑𝜇 ⩽ 𝑓 · I𝐸0 𝑑𝜇 ⩽ .
𝐸0 𝐸

Обратно, пусть 0 ⩽ 𝜓 ⩽ 𝑓 · I𝐸0 на 𝐸. Тогда 𝜓 = 0 на 𝐸 ∖ 𝐸0 и, значит, 𝜓 = 𝜓 · I𝐸0 на 𝐸.


⏟ ⏞
прост.
Следовательно, ∫︁ ∫︁ ∫︁ ∫︁
𝜓 𝑑𝜇 = 𝜓 · I𝐸0 𝑑𝜇 = 𝜓 𝑑𝜇 ⩽ 𝑓 𝑑𝜇.
𝐸 𝐸 𝐸0 𝐸0
∫︀ ∫︀
и, значит, 𝐸 𝑓 · I𝐸0 𝑑𝜇 ⩽ 𝐸0 𝑓 𝑑𝜇.
∫︀ ∫︀
Свойство 17. Если 𝐸0 ⊂ 𝐸 измеримо, то 𝐸0 𝑓 𝑑𝜇 ⩽ 𝐸 𝑓 𝑑𝜇.

Доказательство. По свойствам (14) и (16) имеем


∫︁ ∫︁ ∫︁
𝑓 𝑑𝜇 = 𝑓 · I𝐸0 𝑑𝜇 ⩽ 𝑓 𝑑𝜇.
𝐸0 𝐸 𝐸

Теорема 53 (Беппо Леви). Пусть 𝑓𝑘 : 𝐸 → [0, +∞] измеримы, и 𝑓𝑘 → 𝑓 на 𝐸. Если 0 ⩽ 𝑓𝑘 (𝑥) ⩽


𝑓𝑘+1 (𝑥) для всех 𝑥 ∈ 𝐸 и 𝑘 ∈ N, то
∫︁ ∫︁
lim 𝑓𝑘 𝑑𝜇 = 𝑓 𝑑𝜇.
𝑘→∞ 𝐸 𝐸

Доказательство. Интегрируя 𝑓𝑘 ⩽ 𝑓𝑘+1 ⩽ 𝑓 на 𝐸, получим


∫︁ ∫︁ ∫︁
𝑓𝑘 𝑑𝜇 ⩽ 𝑓𝑘+1 𝑑𝜇 ⩽ 𝑓 𝑑𝜇.
𝐸 𝐸 𝐸
{︀∫︀ }︀
Следовательно, 𝐸 𝑓 𝑑𝜇 нестрого возрастает (в R) и, значит, существует
∫︁ ∫︁
lim 𝑓𝑘 𝑑𝜇 ⩽ 𝑓 𝑑𝜇.
𝑘→∞ 𝐸 𝐸
∫︀
∫︀Докажем противоположное неравенство. Для этого достаточно доказать, что lim𝑘→∞ 𝐸 𝑓𝑘 𝑑𝜇 ⩾
𝐸 𝜙 𝑑𝜇 для всех простых 𝜙, 0 ⩽ 𝜙 ⩽ 𝑓 на 𝐸.
Рассмотрим такую функцию 𝜙. Зафиксируем 𝑡 ∈ (0,⋃︀1). Положим 𝐸𝑘 = {𝑥 ∈ 𝐸 : 𝑓𝑘 (𝑥) ⩾ 𝑡𝜙(𝑥)}.
Ввиду монотонности ∀𝑘 𝐸𝑘 ⊂ 𝐸𝑘+1 . Докажем, что ∞ 𝑘=1 𝐸𝑘 = 𝐸. Включение «⊂» очевидно.
Пусть 𝑥 ∈ 𝐸. Если 𝜙(𝑥) = 0, то ∀𝑘 𝑥 ∈ 𝐸𝑘 .
Если 𝜙(𝑥) > 0, то 𝑓 (𝑥) ⩾ 𝜙(𝑥) > 𝑡𝜙(𝑥). Тогда ∃𝑚 ∈ N (𝑓𝑚 (𝑥) ⩾ 𝑡𝜙(𝑥)), то есть 𝑥 ∈ 𝐸𝑚 .

ФПМИ МФТИ, весна 2023 52


Математический анализ Назад к содержанию

По монотонности ∫︁ ∫︁ ∫︁
𝑓𝑘 𝑑𝜇 ⩾ 𝑓𝑘 𝑑𝜇 ⩾ 𝑡 𝜙 𝑑𝜇.
𝐸 𝐸𝑘 𝐸𝑘

Пусть 𝜙 = 𝑁 𝑁
∑︀
𝑖=1 𝑎𝑖 · I𝐴𝑖 , где {𝐴𝑖 }1 — допустимое разбиение.
Тогда по свойству монотонности меры:
∫︁ 𝑁
∑︁ 𝑁
∑︁ ∫︁
𝜙 𝑑𝜇 = 𝑎𝑖 𝜇(𝐴𝑖 ∩ 𝐸𝑘 ) → 𝑎𝑖 𝜇(𝐴𝑖 ∩ 𝐸) = 𝜙 𝑑𝜇.
𝐸𝑘 𝑘→∞ 𝐸
𝑖=1 𝑖=1

Переходя к пределу в неравенстве (53)


∫︁ ∫︁
lim 𝑓𝑘 𝑑𝜇 ⩾ 𝑡 𝜙 𝑑𝜇, 𝑡 → 1 − 0.
𝑘→∞ 𝐸 𝐸

∫︀ ∫︀ ∫︀
Свойство 18 (аддитивность). Если 𝑓, 𝑔 ⩾ 0 измеримы на 𝐸, то 𝐸 (𝑓 + 𝑔) 𝑑𝜇 = 𝐸 𝑓 𝑑𝜇 + 𝐸 𝑔 𝑑𝜇.

Доказательство. Пусть 𝜙𝑘 ↑ ( возрастает и стремится к )𝑓, 𝜓𝑘 ↑ 𝑔 на 𝐸. Тогда 𝜙𝑘 + 𝜓𝑘 ↑ 𝑓 + 𝑔 на


𝐸 и, значит, по теореме Леви
∫︁ ∫︁
(𝑓 + 𝑔) 𝑑𝜇 = lim (𝜙𝑘 + 𝜓𝑘 ) 𝑑𝜇 =
𝐸 𝑘→∞ 𝐸
∫︁ ∫︁ ∫︁ ∫︁
= lim 𝜙𝑘 𝑑𝜇 + lim 𝜓𝑘 𝑑𝜇 = 𝑓 𝑑𝜇 + 𝑔 𝑑𝜇.
𝑘→∞ 𝐸 𝑘→∞ 𝐸 𝐸 𝐸

Теорема 54 (счётная аддитивность интеграла). Пусть 𝐸𝑘 измеримы и попарно не пересекают-


⨆︀∞
ся, 𝐸 = 𝑘=1 𝐸𝑘 . Если 𝑓 ⩾ 0 на 𝐸, то
∫︁ ∞ ∫︁
∑︁
𝑓 𝑑𝜇 = 𝑓 𝑑𝜇.
𝐸 𝑘=1 𝐸𝑘

∑︀∞ ∑︀∞
Доказательство. Поскольку {𝐸𝑘 } образуют разбиение 𝐸, то I𝐸 = 𝑘=1 I𝐸𝑘 , 𝑓 = 𝑓 · I𝐸 = 𝑘=1 𝑓 ·
I𝐸𝑘 на 𝐸. Следовательно, по теореме Леви для рядов и
∫︁ ∞ ∫︁
∑︁ ∞ ∫︁
∑︁
𝑓 𝑑𝜇 = 𝑓 · I𝐸𝑘 𝑑𝜇 = 𝑓 𝑑𝜇.
𝐸 𝑘=1 𝐸 𝑘=1 𝐸𝑘

Теорема 55 (неравенство Чебышёва). Если 𝑓 ⩾ 0 измерима на 𝐸, то ∀𝑡 ∈ (0, +∞)


∫︁
1
𝜇{𝑥 ∈ 𝐸 : 𝑓 (𝑥) ⩾ 𝑡} ⩽ 𝑓 𝑑𝜇.
𝑡 𝐸

Доказательство. Рассмотрим 𝐸𝑡 = {𝑥 : 𝑓 (𝑥) ⩾ 𝑡}, тогда


∫︁ ∫︁ ∫︁
𝑓 𝑑𝜇 ⩾ 𝑓 𝑑𝜇 ⩾ 𝑡 𝑑𝜇 = 𝑡 · 𝜇(𝐸𝑡 ).
𝐸 𝐸𝑡 𝐸𝑡

ФПМИ МФТИ, весна 2023 53


Математический анализ Назад к содержанию

Определение 60. Функции 𝑓 + = max{𝑓, 0} и 𝑓 − = max{−𝑓, 0} называются положительной и


отрицательной частями 𝑓 соответственно.

Замечание. Из определения следует, что 𝑓 = 𝑓 + − 𝑓 − , |𝑓 | = 𝑓 + + 𝑓 − и 0 ⩽ 𝑓 ± ⩽ |𝑓 |.

Определение 61. Пусть 𝑓 : 𝐸 → R измерима, тогда


∫︁ ∫︁ ∫︁
𝑓 𝑑𝜇 := +
𝑓 𝑑𝜇 − 𝑓 − 𝑑𝜇,
𝐸 𝐸 𝐸

при условии, что хотя бы один из 𝐸 𝑓 ± 𝑑𝜇 конечен.


∫︀

Функция 𝑓 называется интегрируемой (по Лебегу), если оба интеграла 𝐸 𝑓 ± 𝑑𝜇 конечны.


∫︀

Замечание. Если⃒∫︀𝑓 измерима ⃒ ∫︀ на 𝐸, то условия интегрируемости 𝑓 и |𝑓 | равносильны. В случае


интегрируемости ⃒ 𝐸 𝑓 𝑑𝜇⃒ ⩽ 𝐸 |𝑓 | 𝑑𝜇.
∫︀ ±
Доказательство. Если 𝑓 интегрируема на 𝐸, то 𝐸 𝑓 𝑑𝜇 < +∞. Тогда в силу оценки |𝑓 | =
𝑓 + + 𝑓 − интеграл на 𝐸, то в силу оценки 0 ⩽ 𝑓 ± ⩽ |𝑓 |
∫︀
∫︀ ±𝐸 |𝑓 |𝑑𝜇 < +∞. Если |𝑓 | интегрируема
получаем, что 𝐸 𝑓 𝑑𝜇 < +∞, то есть 𝑓 интегрируема на 𝐸.
Имеем ⃒∫︁ ⃒ ⃒∫︁ ∫︁ ⃒ ∫︁ ∫︁ ∫︁
⃒ 𝑓 𝑑𝜇⃒ = ⃒ 𝑓 + 𝑑𝜇 − − + −
⃒ ⃒ ⃒ ⃒
⃒ ⃒ ⃒ 𝑓 𝑑𝜇⃒ ⩽⃒ 𝑓 𝑑𝜇 + 𝑓 𝑑𝜇 = |𝑓 |𝑑𝜇.
𝐸 𝐸 𝐸 𝐸 𝐸 𝐸

Замечание. Если 𝑓 интегрируема на 𝐸, то 𝑓 конечна почти всюду на 𝐸.

Доказательство. Определим 𝐴 = {𝑥 ∈ 𝐸 : |𝑓 (𝑥)| =∫︀ +∞}. Тогда по неравенству Чебышева для


любого 𝑡 ∈ (0; +∞) : 𝜇(𝐴) ⩽ 𝜇{𝑥 : |𝑓 (𝑥)| ⩾ 𝑡} ⩽ 1𝑡 𝐸 |𝑓 |𝑑𝜇. Устремляя 𝑡 → +∞, получаем, что
𝜇(𝐴) = 0.
∫︀ ∫︀
Лемма 30. Если 𝐸0 ⊂ 𝐸 и 𝜇(𝐸 ∖ 𝐸0 ) = 0, то интегралы 𝐸 𝑓 𝑑𝜇 и 𝐸0 𝑓 𝑑𝜇 существуют
⏟ ⏞
изм.
одновременно и в случае существования совпадают.

Доказательство. Отметим, что 𝑓 на 𝐸 и 𝑓 на 𝐸0 измеримы одновременно. По свойству адди-


тивности по множествам:
∫︁ ∫︁ ∫︁ ∫︁
± ± ±
𝑓 𝑑𝜇 = 𝑓 𝑑𝜇 + 𝑓 𝑑𝜇 = 𝑓 ± 𝑑𝜇.
𝐸 𝐸0 𝐸∖𝐸0 𝐸0

Учтем, что интеграл по множеству меры 0 от произведения измеримых функций равен 0. Это
вытекает из определения интеграла, для простых функций также следует учесть, что она огра-
ничена.

Следствие 26. Пусть 𝑓, 𝑔 : ⏟ 𝐸⏞ → R. Если 𝑓 интегрируема на 𝐸 и 𝑓 = 𝑔 почти всюду на 𝐸, то


∫︀ изм. ∫︀
𝑔 интегрируема на 𝐸 и 𝐸 𝑔𝑑𝜇 = 𝐸 𝑓 𝑑𝜇.

Теорема 56. Пусть 𝑓, 𝑔 : 𝐸 → R интегрируема и 𝛼 ∈ R. Тогда:


∫︀ ∫︀
1. Если 𝑓 ⩽ 𝑔 на 𝐸, то 𝐸 𝑓 𝑑𝜇 ⩽ 𝐸 𝑔𝑑𝜇;
∫︀ ∫︀
2. 𝐸 𝛼𝑓 𝑑𝜇 = 𝛼 𝐸 𝑓 𝑑𝜇;
∫︀ ∫︀ ∫︀
3. 𝐸 (𝑓 + 𝑔)𝑑𝜇 = 𝐸 𝑓 𝑑𝜇 + 𝐸 𝑔𝑑𝜇.

Доказательство.

ФПМИ МФТИ, весна 2023 54


Математический анализ Назад к содержанию

1. Пусть 𝑓 ⩽ 𝑔 на 𝐸. Тогда 𝑓 + ⩽ 𝑔 + , 𝑓 − ⩾ 𝑔 − и, значит,


∫︀ + ∫︀ + ∫︀ − ∫︀ −
∫︀ 𝐸 𝑓 𝑑𝜇∫︀ ⩽ 𝐸 𝑔 𝑑𝜇 и 𝐸 𝑓 𝑑𝜇 ⩾ 𝐸 𝑔 𝑑𝜇.
Вычтем одно неравенство из другого, получаем 𝐸 𝑓 𝑑𝜇 ⩽ 𝐸 𝑔𝑑𝜇.
+ + − − значит, 𝐸 𝛼𝑓 𝑑𝜇 = 𝐸 (𝛼𝑓 )+ 𝑑𝜇− 𝐸 (𝛼𝑓 )− 𝑑𝜇 =
∫︀ ∫︀ ∫︀
∫︀ +𝛼 ⩾ 0. Тогда
2. Пусть ∫︀ −(𝛼𝑓 ) =∫︀𝛼𝑓 , (𝛼𝑓 ) = 𝛼𝑓 и, +
𝛼 𝐸 𝑓 𝑑𝜇 − 𝛼 𝐸 𝑓 𝑑𝜇 = 𝛼 𝐸 𝑓 𝑑𝜇. Так как (−𝑓 ) = max{−𝑓, 0} = 𝑓 − , (−𝑓 )− = max{𝑓, 0} =
𝑓 + , то:
∫︁ ∫︁ ∫︁ ∫︁ ∫︁ ∫︁
(−𝑓 )𝑑𝜇 = (−𝑓 )+ 𝑑𝜇 − (−𝑓 )− 𝑑𝜇 = 𝑓 − 𝑑𝜇 − 𝑓 + 𝑑𝜇 = − 𝑓 𝑑𝜇.
𝐸 𝐸 𝐸 𝐸 𝐸 𝐸

Случай 𝛼 < 0 сводится к рассмотренному, так как 𝛼 = (−1)|𝛼|.

3. Так как 𝑓 и 𝑔 конечны почти всюду на 𝐸 (из интегрируемости), то ∃𝐸0 ⊂ 𝐸 и 𝜇(𝐸 ∖ 𝐸0 ) = 0,


на котором определена функция ℎ = 𝑓 + 𝑔. Функция ℎ = 𝑓 + 𝑔 интегрируема на 𝐸0 (так как
|ℎ| ⩽ |𝑓 | + |𝑔|) и ℎ+ − ℎ−∫︀ = ℎ = 𝑓 + 𝑔∫︀ = (𝑓 + − 𝑓 − ) + (𝑔 + − 𝑔 − ) или ℎ+ +∫︀𝑓 − + 𝑔 − = ℎ∫︀− + 𝑓 + + 𝑔 +
на 𝐸0 . Следовательно, 𝐸0 ℎ+ 𝑑𝜇 + 𝐸0 𝑓 − 𝑑𝜇 + 𝐸0 𝑔 − 𝑑𝜇 = 𝐸0 ℎ− 𝑑𝜇 + 𝐸0 𝑓 + 𝑑𝜇 + 𝐸0 𝑔 + 𝑑𝜇.
∫︀ ∫︀
∫︀ ∫︀
Все интегралы в предыдущем равенстве конечны, их перегруппировка дает 𝐸0 ℎ𝑑𝜇 = 𝐸0 𝑓 𝑑𝜇+
∫︀
𝐸 𝑔𝑑𝜇.
Так как 𝜇(𝐸∖𝐸0 ), то доопределим на 𝐸0 ∪(𝐸∖𝐸0 ) произвольным образом. Получаем равенство
для интегралов из 3 пункта.

Теорема 57 (Лебег). Пусть 𝑓𝑘 : 𝐸 → R измеримы и 𝑓𝑘 → 𝑓 почти всюду∫︀на 𝐸. Если


∫︀ существу-
ет интегрируемая на 𝐸 функция 𝑔, такая что |𝑓𝑘 | ⩽ 𝑔 ∀𝑘, то lim𝑘→+∞ 𝐸 𝑓𝑘 𝑑𝜇 = 𝐸 𝑓 𝑑𝜇

Доказательство. Посколько при интегрируемости можно пренебрегать множествами меры 0,


будем считать, что 𝑓𝑘 → 𝑓 всюду на 𝐸 и 𝑔 конечна на 𝐸. Так как |𝑓𝑘 | ⩽ 𝑔 на 𝐸, то все 𝑓𝑘
интегрируемы на 𝐸. Переходя к пределу при 𝑘 → +∞, получаем |𝑓 | ⩽ 𝑔 на 𝐸. Следовательно, 𝑓
интегрируема.
Определим ℎ𝑘 = sup𝑚⩾𝑘 |𝑓𝑚 −𝑓 | на 𝐸, тогда имеем 0 ⩽ ℎ𝑘+1 (𝑥) ⩽ ℎ𝑘 (𝑥) на 𝐸 и lim𝑘→+∞ ℎ𝑘 (𝑥) =
inf 𝑘 𝑠𝑢𝑝𝑚⩾𝑘 |𝑓𝑚 (𝑥) − 𝑓 (𝑥)| = lim𝑘→+∞ |𝑓𝑘 (𝑥) − 𝑓 (𝑥)| = 0. Функция ℎ𝑘 интегрируема на 𝐸 и |ℎ𝑘 | ⩽ 2𝑔
(|𝑓𝑘 | ⩽ 𝑔, |𝑓 | ⩽ 𝑔). Применим теорему Леви к последовательности {2𝑔 − ℎ𝑘 }:
∫︁ ∫︁
lim (2𝑔 − ℎ𝑘 )𝑑𝜇 = 2𝑔𝑑𝜇,
𝑘→+∞ 𝐸 𝐸
∫︀ ∫︀ ∫︀
откуда lim𝑘→+∞ 𝐸 ℎ
⃒∫︀𝑘 𝑑𝜇 = 0. Для
∫︀ завершения
⃒ ∫︀ доказательства 𝐸 |𝑓𝑘 − 𝑓 |𝑑𝜇 ⩽ 𝐸 ℎ𝑘 𝑑𝜇 → 0 при
𝑘 → +∞ и, значит, 𝐸 𝑓𝑘 𝑑𝜇 − 𝐸 𝑓 𝑑𝜇 ⩽ 𝐸 |𝑓𝑘 − 𝑓 |𝑑𝜇 → 0.
⃒ ⃒

Теорема 58. Пусть 𝑓 ограничена на [𝑎, 𝑏]. 𝑓 интегрируема по Риману на [𝑎, 𝑏] ⇔ 𝑓 непрерывна
почти всюду на [𝑎, 𝑏]. В этом случае функция интегрируема по Лебегу и оба интеграла совпа-
дают.
∫︀ 𝑏
Доказательство.
∫︀ 1. Пусть 𝑓 ∈ ℛ[𝑎, 𝑏], 𝐽 = 𝑎 𝑓 (𝑥)𝑑𝑥. Покажем, что 𝑓 непрерывна почти всюду
на [𝑎, 𝑏] и [𝑎,𝑏] 𝑓 𝑑𝜇 = 𝐽.
Для разбиения 𝑇 = {𝑥𝑘 }𝑚
𝑘=0 открытого на [𝑎, 𝑏] положим 𝑀𝑖 = sup[𝑥𝑖−1 ,𝑥𝑖 ] 𝑓 , 𝑚𝑖 = inf [𝑥𝑖−1 ,𝑥𝑖 ] 𝑓
и определим простые функции
𝑚
∑︁ 𝑚
∑︁
𝜙𝑇 = 𝑚𝑖 · I[𝑥𝑖−1 ,𝑥𝑖 ) , 𝜓𝑇 = I[𝑥𝑖−1 ,𝑥𝑖 ) · 𝑀𝑖 .
𝑖=1 𝑖=1

ФПМИ МФТИ, весна 2023 55


Математический анализ Назад к содержанию

∫︀ ∫︀
В последний промежуток включим точку 𝑏 = 𝑥𝑛 . Очевидно, что [𝑎,𝑏] 𝜙𝑇 𝑑𝜇 = 𝑠𝑇 , [𝑎,𝑏] 𝜓𝑇 𝑑𝜇 =
𝑆𝑇 (сумма Дарбу).
Рассмотрим последовательность разбиений {𝑇𝑘 }, 𝑇𝑘 ⊂ 𝑇𝑘+1 и |𝑇 | → 0. Положим 𝜙𝑘 = 𝜙𝑇𝑘 ,
𝜓𝑘 = 𝜓𝑇𝑘 . Имеем 𝜙𝑘 (𝑥) ⩽ 𝜙𝑘+1 (𝑥) ⩽ 𝑓 (𝑥) ⩽ 𝜓𝑘+1 (𝑥) ⩽ 𝜓𝑘 (𝑥) для всех 𝑥 ∈ [𝑎, 𝑏]. Следователь-
но, существуют 𝜙(𝑥) = lim𝑘→+∞ 𝜙𝑘 (𝑥), 𝜓(𝑥) = lim𝑘→+∞ 𝜓𝑘 (𝑥).
Функции 𝜙, 𝜓 измеримы (как предел измеримых функций) и если |𝑓 | ⩽ 𝑀 , то |𝜙|, |𝜓| ⩽ 𝑀 и,
значит, по теореме Лебега о мажорированной сходимости
∫︁ ∫︁
(𝜓 − 𝜙)𝑑𝜇 = lim (𝜓𝑘 − 𝜙𝑘 )𝑑𝜇 = lim (𝑆𝑇𝑘 − 𝑠𝑇𝑘 ) = 0,
[𝑎,𝑏] 𝑘→+∞ [𝑎,𝑏] 𝑘→+∞

откуда следует, что 𝜓 − 𝜙 = 0 почти всюду на [𝑎, 𝑏].


Пусть 𝑍 = {𝑥 : 𝜙(𝑥) ̸= 𝜓(𝑥)}. Рассмотрим 𝑥 ̸∈ 𝑍 ∪ ( +∞
⋃︀
𝑘=1 𝑇𝑘 ) и 𝜀 > 0. Выберем 𝑘, так что
𝜓𝑘 (𝑥) − 𝜙𝑘 (𝑥) < 𝜀 и рассмотрим соотвествующее 𝑇𝑘 . Выберем (𝑥 − 𝛿, 𝑥 + 𝛿), лежащий в одном
отрезке разбиения 𝑇𝑘 . Тогда |𝑓 (𝑡) − 𝑓 (𝑥)| < 𝜓𝑘 (𝑥) − 𝜙𝑘 (𝑥) < 𝜀 ∀𝑡 ∈ (𝑥 − 𝛿, 𝑥 + 𝛿). Это означает,
что 𝑓 непрерывна в точке 𝑥. Следовательно, 𝑓 непрерывна почти всюду на [𝑎, 𝑏]. По теореме
Лебега
∫︁ ∫︁
𝐽 = lim 𝑆𝑇𝑘 = lim 𝜙𝑘 𝑑𝜇 = 𝑓 𝑑𝜇.
𝑘→+∞ 𝑘→+∞ [𝑎,𝑏] [𝑎,𝑏]

2. Пусть 𝑓 непрерывна почти всюду на [𝑎, 𝑏] и 𝜀 > 0. Рассмотрим {𝑇𝑘 } – разбиение 𝑘


⋃︀∞ [𝑎, 𝑏] на 2
равных отрезка, тогда 𝑇𝑘+1 ⊂ 𝑇𝑘 . Пусть 𝑥 не является точкой разрыва 𝑓 и 𝑥 ̸∈ 𝑖=1 𝑇𝑘 . Тогда,
как и первом пункте ∫︀, имеем 𝜙𝑘 (𝑥)∫︀ ↑ 𝑓 (𝑥) и 𝜓𝑘 ↓ ∫︀𝑓 (𝑥) (учли непрерывность
∫︀ в точке 𝑥). По
теорме Лебега 𝑆𝑇𝑘 = [𝑎,𝑏] 𝜓𝑘 𝑑𝜇 → [𝑎,𝑏] 𝑓 𝑑𝜇, 𝑠𝑇𝑘 = [𝑎,𝑏] 𝜙𝑘 𝑑𝜇 = [𝑎,𝑏] 𝑓 𝑑𝜇. Тогда, по критерию
Дарбу 𝑓 ∈ ℛ[𝑎, 𝑏].

Теорема 59 (Эйлер). Пусть 𝑓 : [1, +∞) → R дифференцируема и 𝑓 ′ локально интегрируема на


[1, +∞). Тогда для любого 𝑛 ∈ N справедливо равенство
𝑛 ∫︁ 𝑛 ∫︁ 𝑛 (︂ )︂
∑︁ 𝑓 (1) + 𝑓 (𝑛) 1
𝑓 (𝑘) = 𝑓 (𝑡)𝑑𝑡 + + {𝑡} − 𝑓 ′ (𝑡)𝑑𝑡.
1 2 1 2
𝑘=1

Пример (формула Стирлинга). При 𝑛 → +∞ справедлива оценка


√ (︁ 𝑛 )︁𝑛
𝑛! ∼ 2𝜋𝑛 .
𝑒
Доказательство. Применим следствие к функции 𝑓 (𝑡) = ln 𝑡. Тогда
𝑛
∑︁ ln 𝑛
ln 𝑘 = 𝑛 ln 𝑛 − 𝑛 + 1 + 𝐶 + + 𝜀𝑛 ,
2
𝑘=1

ln 𝑛! = ln(𝑛𝑛 𝑒−𝑛 𝑛𝑒𝐶+1 𝑒𝜀𝑛 ),
√ (︁ 𝑛 )︁𝑛
𝑛! = 𝑐 𝑛 (1 + 𝑜(1)), 𝑛 → +∞.
𝑒
Для нахождения константы 𝑐 воспользуемся формулой Валлиса:
(︂ )︂2
1 (2𝑛)!!
𝜋 = lim
𝑛→+∞ 𝑛 (2𝑛 − 1)!!

ФПМИ МФТИ, весна 2023 56


Математический анализ Назад к содержанию

Имеем (︀ )︀2𝑛 √
(2𝑛)!! 22𝑛 (𝑛!)2 22𝑛 𝑐2 𝑛 𝑛𝑒 (1 + 𝑜(1))2 𝑐 𝑛
= = √ (︀ )︀2𝑛 = √ (1 + 𝑜(1)),
(2𝑛 − 1)!! (2𝑛)! 𝑐 2𝑛 2𝑛 (1 + 𝑜(1)) 2
𝑒
значит,
1 𝑐2 𝑛 𝑐2 √
𝜋 = lim (1 + 𝑜(1))2 = ⇒ 𝑐 = 2𝜋.
𝑛→+∞ 𝑛 2 2

ФПМИ МФТИ, весна 2023 57

Вам также может понравиться